You are on page 1of 555

******Created by ebook converter - www.ebook-converter.

com******
ABSITE KILLER 2014
Do you want a great review source for the Absite 2014? Would you like this source to
be modeled after the original Absite Killer? If you do, then this book is for you. It
covers high yield Absite topics in over 300 pages. All of the topics are hyperlinked
from the table of contents to make reviewing specific topics easier.
New updates will be uploaded monthly and will be available for free after a purchase
by sending an email to absitereviewquestions@gmail.com.
Last updated: 12/31/13
TABLE OF CONTENTS (WITH LINKS)
ABDOMINAL COMPARTMENT SYNDROME:
ACALCULOUS CHOLECYSTITIS:
ACHALASIA:
ACTIVATED PROTEIN C: XIGRIS:
ACUTE PHASE RESPONSE:
ACUTE RENAL FAILURE:
ADRENAL MASS:
AIR EMBOLISM:
AMASTIA:
AMINOCAPROIC ACID: AMICAR:
ANAL CANCER:
ANAL SPHINCTER DYSFUNCTION:
ANEMIA:
ANOVA:
ANTIBODIES:
APPENDICEAL MUCOCELE:
BANKED BLOOD:
BARRETTS ESOPHAGUS:
BASE DEFICIT:

******ebook converter DEMO - www.ebook-converter.com*******

******Created by ebook converter - www.ebook-converter.com******


BILIARY ATRESIA:
BIOTECHNOLOGY:
BLEOMYCIN:
BLOOD PRESSURE CUFF:
BREAST DEVELOPMENT:
BUDD CHIARI SYNDROME:
BURNS:
CARBON MONOXIDE POISONING:
CARDIAC MYXOMAS:
CARDIAC OUTPUT: CARDIAC INDEX: CO: CI:
CHOLECYSTOKININ: CCK:
CHOLEDOCHAL CYSTS:
CHOLESTEROL:
CLOSTRIDIUM DIFFICILE: C. DIFFICILE:
COARCTATION OF THE AORTA:
COHORT STUDY:
COLLAGEN:
COLON CANCER:
COMPARTMENT SYNDROME:
CONDYLOMA ACCUMINATA:
CO2 EMBOLISM:
COX INHIBITORS:
CRYOPRECIPITATE:
DDAVP: DESMOPRESSIN:
DESMOID TUMORS:
DIABETES INSIPIDUS:
DIAPHRAGMATIC INJURIES:
DIFFUSE ESOPHAGEAL SPASMS:
DISSEMINATED INTRAVASCULAR COAGULOPATHY: (DIC:)
DIVERTICULITIS:
DNA:
DUODENAL ATRESIA:
ECMO:

******ebook converter DEMO - www.ebook-converter.com*******

******Created by ebook converter - www.ebook-converter.com******


ENDOPLASMIC RETICULUM:
ERRORS: TYPE I ERRORS: TYPE II ERRORS: TYPE III ERRORS:
ERYTHROMYCIN:
ESOPHAGEAL PERFORATION: ESOPHAGEAL RUPTURE:
EXTRINSIC PATHWAY:
FACTORS:
FACTOR V LEIDEN:
FACTOR VIII:
FAMILIAL ADENOMATOUS POLYPOSIS: FAP:
FATTY ACIDS:
FIBRIN:
FLAIL CHEST:
FLUIDS: NORMAL SALINE: LACTATED RINGERS:
FLUIDS: MAINTENANCE FLUIDS:
FOCAL NODULAR HYPERPLASIA: FNH:
FORMIC ACID BURNS:
FUNCTIONAL GENOMICS:
GALLSTONE ILEUS:
GASTRIC CANCER: GASTRIC ADENOCARCINOMA:
GASTRIC LYMPHOMA:
GAStRIC VARICES:
GASTRIN:
GASTROESOPHAGEAL REFLUX DISEASE: (GERD:)
GASTROINTESTINAL BLEEDING:
GASTROINTESTINAL STROMAL TUMORS: GIST:
GASTROSCHISIS:
GLASGOW COMA SCALE: GCS:
GLEEVEC:
GLYCOLYSIS:
GLUCONEOGENESIS:
GYNECOMASTIA:
HEMOGLOBIN OXYGEN BINDING CURVE:
HEMOPHILIA:

******ebook converter DEMO - www.ebook-converter.com*******

******Created by ebook converter - www.ebook-converter.com******


HEMORRHAGE:
HEMORRHOIDS:
HEMOSTASIS:
HEPARIN:
HEPARIN INDUCED THROMBOCYTOPENIA: HIT:
HEPATIC ADENOMA:
HERNIAS:
HIATAL HERNIA:
HISTAMINE:
HYPERCOAGULABLE STATES:
HYPERHOMOCYSTEINEMIA:
HYPERKALEMIA:
HYPERPARATHYROIDISM:
HYPOCALCEMIA:
HYPOKALEMIA:
INCIDENCE:
INHALED ANESTHETICS:
INR:
INTESTINAL ATRESIA:
INTRA-AORTIC BALLOON PUMP: IABP:
INTRINSIC PATHWAY:
INTUSSUSCEPTION:
LACTIFEROUS DUCTS:
LAPAROSCOPY:
LIVER ABSCESS: PYOGENIC:
LIVER CANCER: HEPATOCELLULAR CARCINOMA:
LUNG ABSCESS:
LUNG CANCER:
LUPUS ANTICOAGULANT:
MAINTENANCE FLUIDS:
MALIGNANT HYPERTHERMIA:
MAMMOGRAPHY:
MASTITIS:

******ebook converter DEMO - www.ebook-converter.com*******

******Created by ebook converter - www.ebook-converter.com******


MEAN ARTERIAL PRESSURE: MAP:
MECONIUM ILEUS:
MELANOMA:
META-ANALYSIS:
MITOCHONDRIA:
MOTILIN:
MUCOSA ASSOCIATED LYMPHOID TISSUE: MALT:
NECROTIZING FASCIITIS:
NECROTIZING PANCREATITIS:
NEUROBLASTOMA:
NITRIC OXIDE: NO:
NITROUS OXIDE PNEUMOPERITONEUM: N2O PNEUMOPERITONEUM:
NUCLEOLUS:
NUCLEUS:
OMEPRAZOLE:
OMPHALOCELE:
OXALATE STONES:
PALLIATIVE CARE:
PARADOXICAL ACIDURIA:
PARAESOPHAGEAL HERNIA:
PARALYTICS:
PARATHYROID GLANDS:
PARATHYROID HORMONES: PTH:
PARATHYROIDECTOMY:
PHEOCHROMOCYTOMA:
PLASMA MEMBRANE:
PLASMIN:
PORT WINE STAIN: CAPILLARY HEMANGIOMA:
POSTSPLENECTOMY SEPSIS:
POSTTRANSPLANT LYMPHOPROLIFERATIVE DISORDER: PTLD:
PREGNANCY:
PREVALENCE:
PROSTACYCLIN:

******ebook converter DEMO - www.ebook-converter.com*******

******Created by ebook converter - www.ebook-converter.com******


PROTAMINE:
PROTEIN C:
PROTEIN S:
PRUNE BELLY SYNDROME:
PSEUDOMYXOMA PERITONEI OF APPENDICEAL ORIGIN:
PT:
PTT:
PULMONARY EMBOLISM:
PULMONARY LYMPH NODES:
PYLORIC STENOSIS:
RADIATION THERAPY:
RECOMBINANT FACTOR VIIa: NOVOSEVEN:
RECTUS SHEATH HEMATOMA:
REFEEDING SYNDROME:
REJECTION:
RENIN:
RESPIRATORY QUOTIENT: RQ:
RHABDOMYOLYSIS:
RNA:
SARCOMAS:
SECRETIN:
SENSITIVITY:
SHORT GUT SYNDROME: SHORT BOWEL SYNDROME:
SILVADENE:
SILVER NITRATE:
SKIN GRAFTS:
SOMATOSTATIN:
SPECIFICITY:
SULFAMYLON:
SUPERIOR VENA CAVA SYNDROME: SVC SYNDROME:
SURGICAL SITE INFECTIONS: SSI: SSIs:
SYSTEMIC VASCULAR RESISTANCE: SVR:
TENSION PNEUMOTHORAX:

******ebook converter DEMO - www.ebook-converter.com*******

******Created by ebook converter - www.ebook-converter.com******


TESTICULAR CANCER:
TETRALOGY OF FALLOT:
THROMBIN:
THROMBOXANE:
THYROGLOSSAL CYSTS:
THYROID STORM:
TRACHEA:
TRACHEOESOPHAGEAL FISTULAS: TE FISTULAS:
TRACHEOSTOMY:
TRANSFUSION REACTIONS:
TRANSLATION:
ULCERATIVE COLITIS:
UMBILICAL HERNIAS:
UREMIA:
URINARY TRACT INFECTIONS: UTI: UTIs:
VASODILATION:
VENTRICULAR SEPTAL DEFECTS: VSDs:
VITAMIN D:
VITAMIN K:
VON WILLEBRAND DISEASE: VON WILLEBRANDS DISEASE: VWD:
VON WILLEBRAND FACTOR: VWF:
WARFARIN: COUMADIN:
WOUND HEALING:
ZENKERS DIVERTICULUM:

******ebook converter DEMO - www.ebook-converter.com*******

******Created by ebook converter - www.ebook-converter.com******

TABLE OF CONTENTS (LIST, NO LINK)


- Abdominal compartment syndrome
- Acalculous cholecystitis
- Achalasia
- Activated protein C (Xigris)
- Acute phase response
- Acute renal failure
- Adrenal mass
- Air embolism
- Amastia
- Aminocaproic acid (Amicar)
- Anal cancer
- Anal sphincter dysfunction
- Anemia
- ANOVA
- Antibodies
- Appendiceal mucocele
- Banked blood
- Barretts esophagus
- Base deficit
- Biliary atresia
- Biotechnology
- Bleomycin
- Blood pressure cuff
******ebook converter DEMO - www.ebook-converter.com*******

******Created by ebook converter - www.ebook-converter.com******


- Breast development
- Budd Chiari syndrome
- Burns
- Carbon monoxide poisoning
- Cardiac myxomas
- Cardiac output, cardiac index
- Cholecystokinin (CCK)
- Choledochal cysts
- Cholesterol
- Clostridium difficile (C. difficile)
- Coarctation of the aorta
- Cohort study
- Collagen
- Colon cancer
- Compartment syndrome
- Condyloma accuminata
- CO2 embolism
- Coumadin (see Warfarin)
- COX inhibitors
- Cryoprecipitate
- DDAVP (Desmopressin)
- Desmoid tumors
- Diabetes insipidus
- Diaphragmatic injuries
- Diffuse esophageal spasms
- Disseminated intravascular coagulopathy (DIC)
******ebook converter DEMO - www.ebook-converter.com*******

******Created by ebook converter - www.ebook-converter.com******


- Diverticulitis
- DNA
- Duodenal atresia
- ECMO
- Endoplasmic reticulum
- Errors (Type I, Type II, and Type III errors)
- Erythromycin
- Esophageal perforation / Esophageal rupture
- Extrinsic pathway
- Factors
- Factor V Leiden
- Factor VIII
- Familial adenomatous polyposis (FAP)
- Fatty acids
- Fibrin
- Flail chest
- Fluids: Normal saline, lactated ringers
- Fluids: Maintenance fluids
- Focal nodular hyperplasia (FNH)
- Formic acid burns
- Functional genomics
- Gallstone ileus
- Gastric cancer (Gastric adenocarcinoma)
- Gastric lymphoma
- Gastric varices
******ebook converter DEMO - www.ebook-converter.com*******

******Created by ebook converter - www.ebook-converter.com******


- Gastrin
- Gastroesophageal reflux disease (GERD)
- Gastrointestinal bleeding
- Gastrointestinal stromal tumors (GIST)
- Gastroschisis
- Glasgow coma scale (GCS)
- Gleevec
- Glycolysis
- Gluconeogenesis
- Gynecomastia
- Hemoglobin oxygen binding curve
- Hemophilia
- Hemorrhage
- Hemorrhoids
- Hemostasis
- Heparin
- Heparin induced thrombocytopenia (HIT)
- Hepatic adenoma
- Hernias
- Hiatal hernia
- Histamine
- Hypercoagulable states
- Hyperhomocysteinemia
- Hyperkalemia
- Hyperparathyroidism
- Hypocalcemia
******ebook converter DEMO - www.ebook-converter.com*******

******Created by ebook converter - www.ebook-converter.com******


- Hypokalemia
- Incidence
- Inhaled anesthetics
- INR
- Intestinal atresia
- Intra-aortic balloon pump (IABP)
- Intrinsic pathway
- Intussusception

- Lactiferous ducts
- Laparoscopy
- Liver abscess: pyogenic
- Liver cancer (hepatocellular carcinoma)
- Lung abscess
- Lung cancer
- Lupus anticoagulant
- Maintenance fluids
- Malignant hyperthermia
- Malrotation and midgut volvulus
- Mammography
- Mastitis
- Mean arterial pressure (MAP)
- Meconium ileus
- Melanoma
- Meta-analysis
******ebook converter DEMO - www.ebook-converter.com*******

******Created by ebook converter - www.ebook-converter.com******


- Mitochondria
- Motilin
- Mucosa associated lymphoid tissue (MALT)
- Necrotizing fasciitis
- Necrotizing pancreatitis
- Neuroblastoma
- Nitric oxide (NO)
- Nitrous oxide pneumoperitoneum (N2O pneumoperitoneum)
- Nucleolus
- Nucleus
- Omeprazole
- Omphalocele
- Oxalate stones
- Palliative care
- Paradoxical aciduria
- Paraesophageal hernia
- Paralytics
- Parathyroid glands
- Parathyroid hormones (PTH)
- Parathyroidectomy
- Pheochromocytoma
- Plasma membrane
- Plasmin
- Port wine stain (capillary hemangioma)
- Postsplenectomy sepsis
******ebook converter DEMO - www.ebook-converter.com*******

******Created by ebook converter - www.ebook-converter.com******


- Posttransplant lymphoproliferative disorder (PTLD)
- Pregnancy
- Prevalence
- Prostacyclin
- Protamine
- Protein C
- Protein S
- Prune belly syndrome
- Pseudomyxoma peritonei of appendiceal origin
- PT
- PTT
- Pulmonary embolism
- Pulmonary lymph nodes
- Pyloric stenosis
- Radiation therapy
- Recombinant factor VIIa (Novoseven)
- Rectus sheath hematoma
- Refeeding syndrome
- Rejection
- Renin
- Respiratory quotient (RQ)
- Rhabdomyolysis
- RNA
- Sarcomas
- Secretin
- Sensitivity
******ebook converter DEMO - www.ebook-converter.com*******

******Created by ebook converter - www.ebook-converter.com******


- Short gut syndrome / Short bowel syndrome
- Silvadene
- Silver nitrate
- Skin grafts
- Somatostatin
- Specificity
- Sulfamylon
- Superior vena cava syndrome (SVC syndrome)
- Surgical site infections (SSIs)
- Systemic vascular resistance (SVR)
- Tension pneumothorax
- Testicular cancer
- Tetralogy of Fallot
- Thrombin
- Thromboxane
- Thyroglossal cysts
- Thyroid storm
- Trachea
- Tracheoesophageal fistulas (TE fistulas)
- Tracheostomy
- Transfusion reactions
- Translation
- Ulcerative colitis
- Umbilical hernias
- Uremia
- Urinary tract infections (UTIs)
******ebook converter DEMO - www.ebook-converter.com*******

******Created by ebook converter - www.ebook-converter.com******

- Vasodilation
- Ventricular septal defects (VSDs)
- Vitamin D
- Vitamin K
- von Willebrand Disease (vWD)
- von Willebrand Factor (vWF)
- Warfarin
- Wound healing
- Xigris (see Activated protein C)
- Zenkers diverticulum

******ebook converter DEMO - www.ebook-converter.com*******

******Created by ebook converter - www.ebook-converter.com******

******ebook converter DEMO - www.ebook-converter.com*******

******Created by ebook converter - www.ebook-converter.com******

ABDOMINAL COMPARTMENT SYNDROME:


QUESTION: A 27 year old man is in the ICU 6 hours after splenectomy following a
motor vehicle accident. The patient had a prolonged transport time and received 20
units of blood prior to arrival. Currently, his peak airway pressures are 65 (plateaus
50), his abdomen is distended, he is not making any urine, and his bladder pressure is
40. His CVP is 18.
What is the diagnosis and what is the treatment for the diagnosis?
.
.
.
.
.
ANSWER:
- This patient has classic signs of abdominal compartment syndrome.
- Objectively, a bladder pressure >25-30 suggests abdominal compartment syndrome.
- Decreased urine output (from compression of the IVC resulting in decreased cardiac
output) and elevated ventilation pressures are part of the syndrome.
- Treatment of abdominal compartment syndrome is decompressive laparotomy.
.
.
.
.

******ebook converter DEMO - www.ebook-converter.com*******

******Created by ebook converter - www.ebook-converter.com******

QUESTION: What is the following bladder pressures is an absolute indication for a


decompressive laparotomy?
.
.
.
.
.
.
ANSWER:
- Generally, no specific bladder pressure prompts therapeutic intervention, except when
the pressure is >35 mmHg.
- Rather, emergent decompression is carried out when intra-abdominal hypertension
reaches a level at which end-organ dysfunction occurs.
- Mortality is directly affected by decompression, with 60% mortality in patients
undergoing presumptive decompression, 70% mortality in patients with a delay in
decompression, and nearly uniform mortality in those not undergoing decompression.
- Abdominal hypertension is classified by grade, with Grade I (mild) being >10 mmHg
(13mmH20).
- Grade IV hypertension or >35 mmHg ( 48cmH2) is an absolute indication for
decompressive laparotomy..

******ebook converter DEMO - www.ebook-converter.com*******

******Created by ebook converter - www.ebook-converter.com******

******ebook converter DEMO - www.ebook-converter.com*******

******Created by ebook converter - www.ebook-converter.com******

ACALCULOUS CHOLECYSTITIS:
QUESTION: A 45-year-old man has a 50% total body surface area third-degree burn.
On hospital day 7, fever, marked leukocytosis, and right upper quadrant pain develop.
His blood pressure is 130/80 mmHg and his heart rate is 110 beats per minute.
Ultrasonography shows a distended gallbladder but is negative for gallstones.
Antibiotics are initiated. What is the next step in management?
.
.
.
.
.
.
ANSWER: Hepatoiminodiacetic acid (HIDA) scan
- The presentation is most consistent with acalculous cholecystitis (see response to
question 13). The initial study of choice is US, which can be performed at the bedside.
- If the US findings are negative and the patient is not critically ill, the next study would
be an HIDA scan with morphine.
.
.
.
.
.
QUESTION: What ultrasound findings support a diagnosis of acalculous cholecystitis?
.
.
.
.
.
******ebook converter DEMO - www.ebook-converter.com*******

******Created by ebook converter - www.ebook-converter.com******


ANSWER:
- Findings that would confirm the diagnosis would include thickening of the gallbladder
wall, sludge, and pericholecystic fluid.
- If the US findings are negative and the patient is not critically ill, the next study would
be an HIDA scan with morphine.
.
QUESTION: What is a positive finding of acalculous cholecystitis on HIDA scan?.
.
.
.
.
ANSWER:
- A positive study finding would demonstrate nonfilling of the gallbladder with
visualization of the tracer in the liver and small bowel.
- Morphine decreases the rate of false-positive HIDA scan results because it leads to
sphincter of Oddi contraction and thus increases the likelihood of filling of the
gallbladder in the absence of cholecystitis.
.
.
.
.
.
QUESTION: True or false: The incidence of perforation with acalculous cholecystitis
is higher than with calculous cholecystitis.
.
.
.
.
******ebook converter DEMO - www.ebook-converter.com*******

******Created by ebook converter - www.ebook-converter.com******


.
ANSWER: True

******ebook converter DEMO - www.ebook-converter.com*******

******Created by ebook converter - www.ebook-converter.com******

QUESTION: Acalculous cholecystitis is seen in association with severe burns.


.
.
.
.
.
ANSWER: True
- Acalculous cholecystitis typically occurs in critically ill, highly stressed patients, such
as those who have experienced severe trauma, sepsis, burns, or multisystem organ
failure.
- The etiology is unclear but is likely related to a combination of ischemia to the
gallbladder from a low flow state and marked gallbladder distention, as well as bile
stasis due to a lack of gallbladder contraction (many patients are on total parenteral
nutrition).
.
.
.
.
.
QUESTION: True or false: The US findings in acalculous cholecystitis may be normal.
.
.
.
.
.
ANSWER: True
******ebook converter DEMO - www.ebook-converter.com*******

******Created by ebook converter - www.ebook-converter.com******

******ebook converter DEMO - www.ebook-converter.com*******

******Created by ebook converter - www.ebook-converter.com******

QUESTION: True or false: In severely ill patients, emergency laparoscopic


cholecystectomy is the treatment of choice.
.
.
.
.
.
ANSWER: False.
- Acalculous cholecystitis requires urgent intervention, preferably cholecystectomy.
- However, if the patient is too ill for surgery, percutaneous US or computed
tomography guided cholecystostomy is the treatment option of choice.
.
.
.
.
.
QUESTION: True or false: Acalculous cholecystitis may be caused by cystic artery
thrombosis.
.
.
.
.
.
ANSWER: True
******ebook converter DEMO - www.ebook-converter.com*******

******Created by ebook converter - www.ebook-converter.com******

******ebook converter DEMO - www.ebook-converter.com*******

******Created by ebook converter - www.ebook-converter.com******

QUESTION: How do you diagnose acalculous cholecystitis?


.
.
.
.
.
ANSWER:
- The diagnosis can be very difficult to make because (1) patients are often intubated
and sedated, thus not allowing a thorough physical examination; (2) US findings are not
pathognomonic; and (3) HIDA scanning can have false-positive results due to the fact
that the patients are fasting.
- Thus, the diagnosis of acalculous cholecystitis requires a high index of suspicion and
may need to be a diagnosis of exclusion.
- The classic findings are fever, leukocytosis, right upper quadrant pain and tenderness,
elevation of liver function test results (bilirubin and alkaline phosphatase), a thickened
gallbladder wall on US without stones, and nonvisualization of the gallbladder on
HIDA scanning.

******ebook converter DEMO - www.ebook-converter.com*******

******Created by ebook converter - www.ebook-converter.com******

QUESTION: What is the initial diagnostic study of choice for acalculous cholecystitis?
.
.
.
.
.
ANSWER: Ultrasound
- US is the initial diagnostic study of choice and can be performed at the bedside in the
intensive care unit.
- In a recent prospective study, US findings were considered positive if three major
criteria were present: wall thickness greater than 4 mm, hydrops, and sludge.
- The sensitivity was only 50% and the specificity 94%.
- In stable patients in whom the diagnosis is unclear after US, an HIDA scan can be
performed with morphine (67% sensitivity and 100% specificity).
- Failure to visualize the gallbladder is the most sensitive and specific finding.
- Leakage into the pericholecystic space suggests perforation.
- An HIDA scan is not recommended in critically ill patients in whom a delay in therapy
can be potentially fatal.

******ebook converter DEMO - www.ebook-converter.com*******

******Created by ebook converter - www.ebook-converter.com******

******ebook converter DEMO - www.ebook-converter.com*******

******Created by ebook converter - www.ebook-converter.com******

ACHALASIA:
QUESTION: Is achalasia due to increased or decreased ganglion cells?
.
.
.
ANSWER: Achalasia is due to decreased ganglion cells
.
.
.
.
QUESTION: Achalasia results from a problem with ganglion cells in ________'s
plexus
.
.
.
ANSWER: Achalasia results from decreased ganglion cells in Auerbachs plexus
.
.
.
QUESTION: Does achalasia result in absence of peristalsis or increased peristalsis?
.
.
.
ANSWER: Achalasia results in the absence of peristalsis
.
.
.
QUESTION: Does achalasia result in esophageal constriction or dilation?
.
.
.
ANSWER: Achalasia results in esophageal dilation
.
.
.
QUESTION: What image does achalasia look like on Barium swallow?
.
******ebook converter DEMO - www.ebook-converter.com*******

******Created by ebook converter - www.ebook-converter.com******


.
.
ANSWER: Achalasia looks like a bird's beak on Barium swallow
.
.
.
QUESTION: In terms of peristalsis, what do you expect to see on manometry of
someone with achalasia?
.
.
.
ANSWER: In achalasia, manometry shows no peristalsis
.
.
.
QUESTION: Are the LES pressures high or low in achalasia?
.
.
.
ANSWER: In achalasia, there are high LES pressures
.
.
.
QUESTION: Is there too much LES relaxation or does the LES fail to relax in
achalasia?
.
.
.
ANSWER: Achalasia is failure of the LES to relax
.
.
.
QUESTION: What is the treatment for achalasia?
.
ANSWER: The treatment for achalasia is laparoscopic or thoracoscopic Heller
myotomy
******ebook converter DEMO - www.ebook-converter.com*******

******Created by ebook converter - www.ebook-converter.com******

ACTIVATED PROTEIN C: XIGRIS:


QUESTION: What is the mechanism of Xigris (activated protein C)?
.
.
.
.
.
ANSWER:
- The mechanism of action of Xigris (activated protein C) is fibrinolysis.

.
.
.
.
.
QUESTION: What is an important side effect of Xigris?
.
.
.
.
.
ANSWER:
- The side effects of Xigris is bleeding. (QUESTION)

******ebook converter DEMO - www.ebook-converter.com*******

******Created by ebook converter - www.ebook-converter.com******

******ebook converter DEMO - www.ebook-converter.com*******

******Created by ebook converter - www.ebook-converter.com******

ACUTE PHASE RESPONSE:


QUESTION: What factors are increased during the acute phase response?
.
.
.
.
.
ANSWER:
- The factors that increase during the acute phase response are:

INCREASED:
- C-reactive protein (an opsonin, activates complement)
- amyloid A and P
- Fibrinogen
- Haptoglobin
- Ceruloplasmin
- Alpha-1 antitrypsin
- Alpha-1 antichymotrypsin
- C3 (complement)
.
.
.
.
.
QUESTION: What factors are decreased during the acute phase response?
.
.
.
******ebook converter DEMO - www.ebook-converter.com*******

******Created by ebook converter - www.ebook-converter.com******


.
.
ANSWER:
- The factors that decrease during the acute phase response are:

DECREASED
- Albumin
- Transferrin
- Fibronectin

******ebook converter DEMO - www.ebook-converter.com*******

******Created by ebook converter - www.ebook-converter.com******

******ebook converter DEMO - www.ebook-converter.com*******

******Created by ebook converter - www.ebook-converter.com******

ACUTE RENAL FAILURE:


QUESTION: An FeNA of less than what % suggests a low volume, pre-renal picture?
.
.
.
ANSWER: A FeNA<1% is indicative of a low volume, pre-renal picture
.
.
.
QUESTION: A BUN/Cr ratio greater than what is indicative of low volume, pre-renal
picture?
.
.
.
ANSWER: A BUN/Cr > 30 is indicative of a low volume, pre-renal picture
.
.

******ebook converter DEMO - www.ebook-converter.com*******

******Created by ebook converter - www.ebook-converter.com******

******ebook converter DEMO - www.ebook-converter.com*******

******Created by ebook converter - www.ebook-converter.com******

ADRENAL MASS:
QUESTION: What finding on an adrenal mass on CT scan is the most suggestive of
adrenal cancer?
.
.
.
.
.
.
ANSWER: Size > 6cm.
- The size of the adrenal mass on imaging studies is the single most important criterion
to help diagnose malignancy.
- In the series reported by Copeland, 92% of adrenal cancers were >6 cm in diameter.
- Other CT imaging characteristics suggesting malignancy include tumor heterogeneity,
irregular margins, and the presence of hemorrhage and adjacent lymphadenopathy or
liver metastases.

******ebook converter DEMO - www.ebook-converter.com*******

******Created by ebook converter - www.ebook-converter.com******

******ebook converter DEMO - www.ebook-converter.com*******

******Created by ebook converter - www.ebook-converter.com******

AIR EMBOLISM:
QUESTION: How do you treat an air embolism?
.
.
.
.
.
ANSWER:
- Although estimated to occur in only 0.2 to 1% of patients, an air embolism can be
dramatic and fatal.
- Treatment may prove futile if the air embolus is larger than 50ml.
- If an embolus is suspected, the patient should immediately be placed into a left lateral
decubitus Trendelenburg position, so the entrapped air can be stabilized within the right
ventricle.
- Aspiration via a central venous line accessing the heart may decrease the volume of
gas in the right side of the heart, and minimize the amount traversing into the pulmonary
circulation.
- Subsequent recovery of intracardiac and intrapulmonary air may require open surgical
or angiographic techniques.
- Prevention requires careful attention to technique.
.
.
.

******ebook converter DEMO - www.ebook-converter.com*******

******Created by ebook converter - www.ebook-converter.com******

******ebook converter DEMO - www.ebook-converter.com*******

******Created by ebook converter - www.ebook-converter.com******

AMASTIA:
QUESTION: Absence of the breast (amastia) is associated with which genetic
disorder?
.
.
.
.
.
ANSWER: Poland syndrome.
POLAND SYNDROME
- Absence of the breast (amastia) is rare and results from an arrest in mammary ridge
development that occurs during the sixth fetal week.
- Poland's syndrome consists of hypoplasia or complete absence of the breast, costal
cartilage and rib defects, hypoplasia of the subcutaneous tissues of the chest wall, and
brachysyndactyly.
TURNER'S SYNDROME
- Turner's syndrome (ovarian agenesis and dysgenesis) may have polymastia as a
component.
FLEISCHER'S SYNDROME
- Fleischer's syndrome (displacement of the nipples and bilateral renal hypoplasia) may
also have polymastia as a component.
KLINEFELTER'S SYNDROME
- Klinefelter's syndrome (XXY) is manifested by gynecomastia, hypergonadotropic
hypogonadism, and azoospermia.
- There is an increased risk of breast cancer in men with Klinefelter's syndrome.
******ebook converter DEMO - www.ebook-converter.com*******

******Created by ebook converter - www.ebook-converter.com******

******ebook converter DEMO - www.ebook-converter.com*******

******Created by ebook converter - www.ebook-converter.com******

******ebook converter DEMO - www.ebook-converter.com*******

******Created by ebook converter - www.ebook-converter.com******

AMINOCAPROIC ACID: AMICAR:


QUESTION: What is the mechanism of action of Amicar?
.
.
.
.
.
ANSWER:
- The mechanism of action of aminocaproic acid (Amicar) is inhibition of plasmin.

******ebook converter DEMO - www.ebook-converter.com*******

******Created by ebook converter - www.ebook-converter.com******

******ebook converter DEMO - www.ebook-converter.com*******

******Created by ebook converter - www.ebook-converter.com******

ANAL CANCER:
QUESTION: What is the treatment for squamous cell carcinoma of the anal canal?
.
.
.
ANSWER: The treatment for squamous cell carcinoma of the anal canal is the Nigro
protocol
.
.
.
QUESTION: True or false: The treatment of squamous cell carcinoma of the anal canal
is NOT surgery
.
.
.
ANSWER: True. The treatment of squamous cell carcinoma of the anal canal is NOT
surgery. The treatment is the Nigro protocol
.
.
.
QUESTION: The Nigro protocol is made up of which treatment modalities?
.
.
.
ANSWER: The Nigro protocol consists of chemotherapy and radiation therapy
.
.

******ebook converter DEMO - www.ebook-converter.com*******

******Created by ebook converter - www.ebook-converter.com******

******ebook converter DEMO - www.ebook-converter.com*******

******Created by ebook converter - www.ebook-converter.com******

ANAL SPHINCTER DYSFUNCTION:


QUESTION: Is a resting pressure of 20 mmHg indicative of normal internal sphincter
function or dysfunction of the internal sphincter?
.
.
.
.
.
ANSWER: A resting pressure of 20 mmHg indicates internal sphincter dysfunction.
- The resting pressure in the anal canal reflects the function of the internal anal
sphincter (normal: 40 to 80 mmHg)
- The squeeze pressure, defined as the maximum voluntary contraction pressure minus
the resting pressure, reflects function of the external anal sphincter (normal: 40 to 80
mmHg above resting pressure)
- The high-pressure zone estimates the length of the anal canal (normal: 2.0 to 4.0 cm)
- The rectoanal inhibitory reflex can be detected by inflating a balloon in the distal
rectum.
-

******ebook converter DEMO - www.ebook-converter.com*******

******Created by ebook converter - www.ebook-converter.com******

******ebook converter DEMO - www.ebook-converter.com*******

******Created by ebook converter - www.ebook-converter.com******

ANEMIA:
QUESTION: What are the laboratory findings in iron deficiency anemia?
.
.
.
.
.
ANSWER:
- The laboratory findings in iron deficiency anemia are low MCV, low ferritin, high transferrin and high TIBC.

.
.
.
.
.
QUESTION: What are the laboratory findings in anemia of chronic disease?
.
.
.
.
.
ANSWER:
- The laboratory findings in anemia of chronic disease are low to normal MCV, normal or high ferritin, and low
TIBC. (TOPIC)

******ebook converter DEMO - www.ebook-converter.com*******

******Created by ebook converter - www.ebook-converter.com******

******ebook converter DEMO - www.ebook-converter.com*******

******Created by ebook converter - www.ebook-converter.com******

ANOVA:
QUESTION: True or false: ANOVA compares means for more than 2 groups.
.
.
.
ANSWER: True
.
.
.
QUESTION: Does ANOVA use quantitative or qualitative data when comparing the
means of more than 2 groups?
.
.
.
ANSWER: ANOVA uses quantitative data when comparing the means of more than 2
groups
.
.
.
QUESTION: True or false: ANOVA is a t-test for more than 2 groups
.
.
.
ANSWER: True

******ebook converter DEMO - www.ebook-converter.com*******

******Created by ebook converter - www.ebook-converter.com******

******ebook converter DEMO - www.ebook-converter.com*******

******Created by ebook converter - www.ebook-converter.com******

ANTIBODIES:
QUESTION: Which antibodies can act as opsonins?
.
.
.
ANSWER: IgG and IgM are opsonins
.
.
.
QUESTION: Which antibodies are able to fix complement?
.
.
.
ANSWER: IgG and IgM are able to fix complement. 2 IgGs or 1 IgM is needed to do
this.
.
.
.
QUESTION: Which antibody level decreases after splenectomy?
.
.
.
ANSWER: After a splenectomy, you will have decreased levels of IgM
.
.
.
QUESTION: Which antibody is made first during an immune reaction?
.
.
.
ANSWER: IgM is made first during an immune reaction
.
.
.

******ebook converter DEMO - www.ebook-converter.com*******

******Created by ebook converter - www.ebook-converter.com******

QUESTION: Which antibody is found in secretions?


.
.
.
ANSWER: IgA is found in secretions
.
.
.
QUESTION: True or false: IgD is found on the surface of B cells and its function is
unclear
.
.
.
ANSWER: True
.
.
.
QUESTION: What antibody is the most abundant antibody in serum?
.
.
.
ANSWER: IgG
.
.
.
QUESTION: Which antibody cross the placenta?
.
.
.
ANSWER: IgG
.
.
.

******ebook converter DEMO - www.ebook-converter.com*******

******Created by ebook converter - www.ebook-converter.com******


QUESTION: Which antibody is involved in allergic reactions?
.
.
.
ANSWER: IgE is involved in allergic reactions
QUESTION: Which antibody is involved in type I hypersensitivity reactions?
.
.
.
ANSWER: IgE is involved in type I hypersensitivity reactions
.
.
.
QUESTION: Which antibody causes histamine release from mast cells and basophils?
.
.
.
ANSWER: IgE causes histamine release from mast cells and basophils
.
.
.
QUESTION: Which antibodies is secreted during a parasitic infection?
.
.
.
ANSWER: IgE is secreted in response to parasitic infections
.
.
.
QUESTION: Which portion of an antibody is responsible for antigen recognition? The
variable region or the constant region?
.
.
.
ANSWER: The variable region of antibody is responsible for antigen recognition
.
******ebook converter DEMO - www.ebook-converter.com*******

******Created by ebook converter - www.ebook-converter.com******

******ebook converter DEMO - www.ebook-converter.com*******

******Created by ebook converter - www.ebook-converter.com******

******ebook converter DEMO - www.ebook-converter.com*******

******Created by ebook converter - www.ebook-converter.com******

APPENDICEAL MUCOCELE:
QUESTION: At the time of laparoscopic surgery for presumed appendicitis, the patient
is noted to have a mucous-filled, distended appendix measuring 3 cm in diameter. There
is no acute inflammation or signs of perforation. What is the best next step in
management?
.
.
.
.
.
ANSWER: Conversion to open appendectomy with pathologic confirmation of a
negative margin at the base of the appendix
- An intact mucocele presents no future risk for the patient.
- However, the opposite is true if the mucocele has rupture and epithelial cells have
escaped into the peritoneal cavity.
- As a result, when a mucocele is visualized at the time of laparoscopic examination,
conversion to open laparotomy is recommended.
- Conversion from a laparoscopic approach to a laparotomy ensures that a benign
process will not be converted to a malignant one through mucocele rupture.
- In addition, laparotomy allows for thorough abdominal exploration to rule out the
presence of mucoid fluid accumulations.

******ebook converter DEMO - www.ebook-converter.com*******

******Created by ebook converter - www.ebook-converter.com******

QUESTION: True or false: The presence of a mucocele of the appendix does not
mandate performance of a right hemicolectomy.
.
.
.
.
.
ANSWER: True.
- The principles of surgery include resection of the appendix, wide resection of the
mesoappendix to include all the appendiceal lymph nodes, collection and cytologic
examination of all intraperitoneal mucus, and careful inspection of the base of the
appendix.

******ebook converter DEMO - www.ebook-converter.com*******

******Created by ebook converter - www.ebook-converter.com******

QUESTION: For appendiceal mucocele, when is a right hemicolectomy indicated?


.
.
.
.
.
ANSWER: Positive margin at base of appendix or positive lymph nodes
- Right hemicolectomy, or preferably cecectomy, is reserved for patients with a positive
margin at the base of the appendix or positive periappendiceal lymph nodes.

******ebook converter DEMO - www.ebook-converter.com*******

******Created by ebook converter - www.ebook-converter.com******

******ebook converter DEMO - www.ebook-converter.com*******

******Created by ebook converter - www.ebook-converter.com******

BANKED BLOOD:
QUESTION: Which factors are low in banked blood?
.
.
.
.
.
ANSWER:
- Factors that are low in banked blood are 2,3-DPG, and factors V and VIII (labile factors)
.
.
.
.
.

QUESTION: Banked blood results in a shift in the oxygen dissociation curve in which
direction?
.
.
.
.
.
ANSWER:
- Shift in the O2 dissociation curve in banked blood is a left shift. (higher affinity of Hgb to O2).
.
.
.
.
.

QUESTION: What is the effect of 2,3 DPG on the oxygen dissociation curve?
******ebook converter DEMO - www.ebook-converter.com*******

******Created by ebook converter - www.ebook-converter.com******


.
.
.
.
.
ANSWER:
- The effect of 2,3 DPG on the oxygen dissociation curve is that it causes a RIGHT shift by decreasing Hgb
affinity to O2.

******ebook converter DEMO - www.ebook-converter.com*******

******Created by ebook converter - www.ebook-converter.com******

BARRETTS ESOPHAGUS:
QUESTION: .Barrett's esophagus refers to what type of mucosal change?
.
.
.
ANSWER: Barrett's refers to the change in esophageal epithelium from squamous to
columnar.
.

******ebook converter DEMO - www.ebook-converter.com*******

******Created by ebook converter - www.ebook-converter.com******

******ebook converter DEMO - www.ebook-converter.com*******

******Created by ebook converter - www.ebook-converter.com******

BASE DEFICIT:
QUESTION: What is the probability of death for a patient with a base deficit of -6?
.
.
.
.
.
ANSWER: 25%.

******ebook converter DEMO - www.ebook-converter.com*******

******Created by ebook converter - www.ebook-converter.com******

******ebook converter DEMO - www.ebook-converter.com*******

******Created by ebook converter - www.ebook-converter.com******

BILIARY ATRESIA:
QUESTION: What is the most important surgical cause of jaundice in the newborn?
.
.
.
.
.
ANSWER: Biliary atresia
- The most important surgical cause of jaundice in the newborn is biliary atresia, which
is an obliterative process of the extrahepatic bile ducts and is associated with hepatic
fibrosis.
- The infant produces acholic stools and demonstrates a failure to thrive.
- Left untreated, it will progress to liver failure and portal hypertension.
.
.
.
.
.
QUESTION: How is biliary atresia diagnosed?
.
.
..
.
ANSEWR: Nuclear scanning.
- Nuclear scanning after pretreatment with phenobarbital is a useful study.
- One is specifically looking to see whether the radionuclide appears in the intestine,
******ebook converter DEMO - www.ebook-converter.com*******

******Created by ebook converter - www.ebook-converter.com******


which would confirm that the extrahepatic bile ducts are patent.
- This finding excludes biliary atresia.
- If the radionuclide is normally concentrated in the liver but not excreted and the
metabolic screen results are normal, this is highly suggestive of biliary atresia.
- The presence of a gallbladder does not exclude the diagnosis of biliary atresia.
.
.
QUESTION: What is the treatment for biliary atresia?
.
.
.
.
.
ANSWER: Kasai procedure (hepatoportoenterostomy)
- The most effective treatment of biliary atresia is portoenterostomy, as described by
Kasai.
- The procedure involves anastomosing an isolated limb of jejunum to the transected
ducts at the liver plate.
- The likelihood of surgical success is increased if the procedure is performed before
the infant reaches the age of 8 weeks.
.
.
.
.
.
QUESTION: What is the indication for liver transplantation in biliary atresia?
.
.
******ebook converter DEMO - www.ebook-converter.com*******

******Created by ebook converter - www.ebook-converter.com******


.
.
.
ANSWER: Sympatomatic after Kasai procedure
- If the patient remains symptomatic after the Kasai operation, he or she will require
liver transplantation.
- Independent risk factors that predict failure of the procedure include bridging liver
fibrosis at the time of surgery and postoperative cholangitis episodes.
.
.
.
.
.

******ebook converter DEMO - www.ebook-converter.com*******

******Created by ebook converter - www.ebook-converter.com******

QUESTION: Optimal management for an 8-month-old girl with biliary atresia in whom
a Kasai operation (hepatoportoenterostomy) failed would be what?
.
.
.
.
.
ANSWER: liver transplantation.
- The Kasai operation, in which an isolated limb of the jejunum is anastomosed to the
transected ducts at the liver plate, is the operation of choice for biliary atresia.
- However, in a subset of patients, the Kasai operation is unsuccessful, and they
eventually require liver transplantation for progressive liver failure and recurrent bouts
of cholangitis.

******ebook converter DEMO - www.ebook-converter.com*******

******Created by ebook converter - www.ebook-converter.com******

******ebook converter DEMO - www.ebook-converter.com*******

******Created by ebook converter - www.ebook-converter.com******

BIOTECHNOLOGY:
Q: What is the difference between southern blotting, northern blotting, and western
blotting? .
.
.
.
A:
- Southern blotting refers to the technique of transferring DNA fragments from an
electrophoresis gel to a membrane support, and the subsequent analysis of the fragments
by hybridization with a radioactively labeled probe.
- Southern blotting is named after E. M. Southern, who in 1975 first described the
technique of DNA analysis.
- It enables reliable and efficient analysis of size-fractionated DNA fragments in an
immobilized membrane support.
- Northern blotting refers to the technique of size fractionation of RNA in a gel and the
transferring of an RNA sample to a solid support (membrane) in such a manner that the
relative positions of the RNA molecules are maintained.
- The resulting membrane then is hybridized with a labeled probe complementary to the
mRNA of interest.
- Signals generated from detection of the membrane can be used to determine the size
and abundance of the target RNA.
- In principle, Northern blot hybridization is similar to Southern blot hybridization (and
hence its name), with the exception that RNA, not DNA is on the membrane.
- Analyses of proteins are primarily carried out by antibody-directed immunologic
techniques.
- For example, Western blotting, also called immunoblotting, is performed to detect
protein levels in a population of cells or tissues, whereas immunoprecipitation is used
to concentrate proteins from a larger pool.
- There is no technique known as Eastern blotting.
******ebook converter DEMO - www.ebook-converter.com*******

******Created by ebook converter - www.ebook-converter.com******

******ebook converter DEMO - www.ebook-converter.com*******

******Created by ebook converter - www.ebook-converter.com******

BLEOMYCIN:
QUESTION: What is the main side effect of Bleomycin?
.
.
.
ANSWER: The main side effect of Bleomycin is pulmonary fibrosis

******ebook converter DEMO - www.ebook-converter.com*******

******Created by ebook converter - www.ebook-converter.com******

******ebook converter DEMO - www.ebook-converter.com*******

******Created by ebook converter - www.ebook-converter.com******

BLOOD PRESSURE CUFF:


QUESTION: The width of a blood pressure cuff should be what percentage of the
circumference of the patient's arm?
.
.
.
.
.
ANSWER: 40%.
- If the cuff is too narrow (relative to the extremity), the measured pressure will be
artificially elevated.
- Therefore, the width of the cuff should be approximately 40% of its circumference.

******ebook converter DEMO - www.ebook-converter.com*******

******Created by ebook converter - www.ebook-converter.com******

******ebook converter DEMO - www.ebook-converter.com*******

******Created by ebook converter - www.ebook-converter.com******

BREAST DEVELOPMENT:
QUESTION: Which of the hormone is primarily responsible for differentiation of the
breast ductal epithelium?
.
.
.
.
.
ANSWER: Progesterone
- Estrogen initiates ductal development, whereas progesterone is responsible for
differentiation of epithelium and for lobular development.
- Prolactin is the primary hormonal stimulus for lactogenesis in late pregnancy and the
postpartum period.
- It upregulates hormone receptors and stimulates epithelial development.

******ebook converter DEMO - www.ebook-converter.com*******

******Created by ebook converter - www.ebook-converter.com******

******ebook converter DEMO - www.ebook-converter.com*******

******Created by ebook converter - www.ebook-converter.com******

BUDD CHIARI SYNDROME:


QUESTION: 25 year old woman develops jaundice and ascites 2 weeks after
childbirth. This is most likely related to what type of venous thrombosis?
.
.
.
.
ANSWER:
- Postpartum hepatic vein thrombosis (Post-partum Budd Chiari Syndrome) is rare and
related to the relative hypercoagulable state that occurs following pregnancy.
- Patients with hypercoagulable syndromes seem more susceptible to the problem.
- The best test for post partum hepatic vein thrombosis is a mesenteric angiogram with
venous phase contrast.
- MRI would also be a useful test but is not as sensitive as an angiogram.

******ebook converter DEMO - www.ebook-converter.com*******

******Created by ebook converter - www.ebook-converter.com******

******ebook converter DEMO - www.ebook-converter.com*******

******Created by ebook converter - www.ebook-converter.com******

BURNS:
QUESTION: What is the most common infection in patients with large (>35%) severe
burns ?
.
.
.
.
.
ANSWER: Although urinary tract infection is the most common infection in surgery
patients, pneumonia is the leading cause of infection in patients with severe burn
injuries.
- Inhalation injury, decreased immunity, fluid resuscitation causing pulmonary edema,
and requirement for mechanical ventilation all lead to increased risk for pneumonia.
- In some series, 60-70% of all patients with large severe burns get pneumonia.
.

******ebook converter DEMO - www.ebook-converter.com*******

******Created by ebook converter - www.ebook-converter.com******

******ebook converter DEMO - www.ebook-converter.com*******

******Created by ebook converter - www.ebook-converter.com******

CARBON MONOXIDE POISONING:


QUESTION: Shock following severe carbon monoxide poisoning is most commonly
what type of shock?
.
.
.
.
.
ANSWER: Vasodilatory shock. Causes of vasodilatory shock include hypoxic lactic
acidosis, carbon monoxide poisoning, decompensated and irreversible hemorrhagic
shock, terminal cardiogenic shock, and postcardiotomy shock.
.
.
.
.
QUESTION: Treatment for carbon monoxide poisoning is primarily what?
.
.
.
.
.
ANSWER:
- The treatment of choice for carbon monoxide poisoning is oxygen.
- For severe poisoning, hyperbaric oxygen therapy can be used.
.
.
******ebook converter DEMO - www.ebook-converter.com*******

******Created by ebook converter - www.ebook-converter.com******


.

******ebook converter DEMO - www.ebook-converter.com*******

******Created by ebook converter - www.ebook-converter.com******

QUESTION: The affinity of carbon monoxide for hemoglobin is how many times greater
than oxygen?
.
.
.
.
.
ANSWER:
- Another important contributor to early mortality in burns is carbon monoxide (CO)
poisoning resulting from smoke inhalation.
- The affinity of CO for hemoglobin is approximately 200-250 times more than that of
oxygen, which decreases the levels of normal oxygenated hemoglobin and can quickly
lead to anoxia and death.
- Unexpected neurologic symptoms should raise the level of suspicion, and an arterial
carboxyhemoglobin level must be obtained because pulse oximetry is falsely elevated.
.
.
.

******ebook converter DEMO - www.ebook-converter.com*******

******Created by ebook converter - www.ebook-converter.com******

******ebook converter DEMO - www.ebook-converter.com*******

******Created by ebook converter - www.ebook-converter.com******

******ebook converter DEMO - www.ebook-converter.com*******

******Created by ebook converter - www.ebook-converter.com******

CARDIAC MYXOMAS:
QUESTION: What is the most common location for cardiac myxomas ?
.
.
.
.
.
ANSWER: Left atrium..
- Myxomas are the most common primary tumor of the heart.
- The majority (60%75%) arise in the left atrium, with most of the remaining in the
right atrium.
- Very few are found in the right or left ventricle.
- Myxomas are gelatinous tumors with a propensity for embolization.
- They can also obstruct or damage the mitral valve, leading to symptoms of heart
failure.
- Diagnosis is made by transthoracic or transesophageal echocardiography.
- Treatment is surgical excision.
- They are benign but have a tendency to recur.

******ebook converter DEMO - www.ebook-converter.com*******

******Created by ebook converter - www.ebook-converter.com******

******ebook converter DEMO - www.ebook-converter.com*******

******Created by ebook converter - www.ebook-converter.com******

CARDIAC OUTPUT: CARDIAC INDEX: CO: CI:


QUESTION: What is the difference between cardiac output and cardiac index? (write
down equations).
.
.
.
.
ANSWER:
- The equation for the cardiac output is CO = SV X SR.
- The equation for the cardiac index is CI = CO / BSA
- [Cl = CO/BSA; CO=SV X HR; therefore, CI = (SV X HR)/BSA]
- The cardiac index is computed by dividing the cardiac output by the body surface area.

******ebook converter DEMO - www.ebook-converter.com*******

******Created by ebook converter - www.ebook-converter.com******

******ebook converter DEMO - www.ebook-converter.com*******

******Created by ebook converter - www.ebook-converter.com******

CHOLECYSTOKININ: CCK:
QUESTION: Where is cholecystokinin produced?
.
.
.
ANSWER: Cholecystokinin (CCK) is released by I-cells of the small intestines
.
.
.
QUESTION: What is the effect of cholecystokinin on the gallbladder?
.
.
.
ANSWER: Cholecystokinin stimulates contraction of the gallbladder
.
.
.
QUESTION: What is the effect of cholecystokinin on the sphincter of Oddi?
.
.
.
ANSWER: Cholecystokinin relaxes the Sphincter of Oddi
.
.
.
QUESTION: What is the effect of cholecystokinin on pancreatic enzyme secretion?
.
.
.
ANSWER: Cholecystokinin increases pancreatic enzyme secretion
.
.
.

******ebook converter DEMO - www.ebook-converter.com*******

******Created by ebook converter - www.ebook-converter.com******

CHOLEDOCHAL CYSTS:
QUESTION: What is the most common type of choledocha cyst?
.
.
.
.
.
ANSWER: Type I
- Choledochal cysts have been classified into five types.
- The most common is type I, which is fusiform dilatation of the bile duct.
.
.
.
.
.
QUESTION: Multiple diffuse dilatations of the intrahepatic ducts are known associated
with which type choledochal cysts?
.
.
.
.
.
ANSWER: Type IV choledochal cysts = Carolis disease
- Caroli disease is a type V choledochal cyst that causes multiple bile duct dilatations
that are limited to the intrahepatic bile ducts.
- The cysts lead to recurrent bouts of cholangitis and have a risk of malignancy.
******ebook converter DEMO - www.ebook-converter.com*******

******Created by ebook converter - www.ebook-converter.com******


- If the Caroli disease is limited to one lobe, partial hepatectomy is potentially curative.
In patients with diffuse Caroli disease, liver transplantation can provide satisfying longterm results.

******ebook converter DEMO - www.ebook-converter.com*******

******Created by ebook converter - www.ebook-converter.com******

******ebook converter DEMO - www.ebook-converter.com*******

******Created by ebook converter - www.ebook-converter.com******

CHOLESTEROL:
QUESTION: What molecule serves as the plasma carrier of cholesterol?
.
.
.
.
ANSWER:
- The plasma carrier of cholesterol is VLDL.
.

.
.
.
.
QUESTION: In which organ is cholesterol formed?
.
.
.
.
.
ANSWER:
- The organ where cholesterol is formed is the liver.
.
.
.
.
.
QUESTION: What enzyme is the rate limiting step in cholesterol formation?
.

******ebook converter DEMO - www.ebook-converter.com*******

******Created by ebook converter - www.ebook-converter.com******


.
.
.
.
ANSWER:
- The rate limiting step in cholesterol formation is HMG CoA reductase.

******ebook converter DEMO - www.ebook-converter.com*******

******Created by ebook converter - www.ebook-converter.com******

******ebook converter DEMO - www.ebook-converter.com*******

******Created by ebook converter - www.ebook-converter.com******

CLOSTRIDIUM DIFFICILE: C. DIFFICILE:


QUESTION: Treatment of severe C. difficile proctosigmoiditis which is unresponsive
to intravenous antibiotics may include what type of enema?
.
.
.
.
.
.
.
- Management of C. difficile colitis should include immediate cessation of the offending
antimicrobial agent.
- Patients with mild disease (diarrhea but no fever or abdominal pain) may be treated as
outpatients with a 10-day course of oral metronidazole.
- Oral vancomycin is a second-line agent used in patients allergic to metronidazole or in
patients with recurrent disease.
- More severe diarrhea associated with dehydration and/or fever and abdominal pain is
best treated with bowel rest, IV hydration, and oral metronidazole or vancomycin.
- Proctosigmoiditis may respond to vancomycin enemas.
- Recurrent colitis occurs in up to 20% of patients and may be treated with a longer
course of oral metronidazole or vancomycin (up to 1 month).
- Reintroduction of normal flora by ingestion of probiotics has been suggested as a
possible treatment for recurrent or refractory disease.
- Fulminant colitis, characterized by septicemia and/or evidence of perforation, requires
emergent laparotomy.
- A total abdominal colectomy with end ileostomy may be lifesaving.

******ebook converter DEMO - www.ebook-converter.com*******

******Created by ebook converter - www.ebook-converter.com******

******ebook converter DEMO - www.ebook-converter.com*******

******Created by ebook converter - www.ebook-converter.com******

COARCTATION OF THE AORTA:


QUESTION: Coarctation of the aorta most commonly occurs in what anatomic location?
.
.
.
.
.
ANSWER: Thoracic aorta just distal to the left subclavian artery.
- Aortic coarctation is a congenital narrowing of the thoracic aorta, usually occurring
distal to the left subclavian artery, at the point of insertion of the ductus arteriosus.
- Coarctation may go undetected until adolescence or young adulthood.
- Patients may present with severe upper extremity hypertension in association with
weak or absent lower extremity pulses.
- Conventional treatment has been with open thoracic aortic surgery.
- However, it more recently has been treated successfully with endovascular techniques.

******ebook converter DEMO - www.ebook-converter.com*******

******Created by ebook converter - www.ebook-converter.com******

******ebook converter DEMO - www.ebook-converter.com*******

******Created by ebook converter - www.ebook-converter.com******

COHORT STUDY:
QUESTION: Is cohort study a prospective study or a retrospective study?
.
.
.
ANSWER: A cohort study is a prospective study
.
.
.
QUESTION Is a cohort study randomized or nonrandomized?
.
.
.
ANSWER: A cohort study is nonrandomized
.
.
.
QUESTION: True or false: A prospective cohort study is a non-random assignment to a
treatment group
.
.
.
ANSWER: True
.
.
.

******ebook converter DEMO - www.ebook-converter.com*******

******Created by ebook converter - www.ebook-converter.com******

******ebook converter DEMO - www.ebook-converter.com*******

******Created by ebook converter - www.ebook-converter.com******

COLLAGEN:
QUESTION: True or false. Tensile strength is never equal to prewound strength. It is
only 80% of what it used to be.
.
.
.
ANSWER: True. The tensile strength is never equal to the pre-wound strength
.
.
.
QUESTION: Which type collage is the most abundant?
.
.
.
ANSWER Collagen Type I is the most abundant throughout
.
.
.
QUESTION: Which type collagen is the principal collagen in scar?
.
.
.
ANSWER: Type I collagen is the principal collagen in scar
.
.
.
QUESTION: Which type collagen is the most abundant in a healing wound?
.
.
.
ANSWER: Type III collagen is the principal collagen in healing wound
.
.

******ebook converter DEMO - www.ebook-converter.com*******

******Created by ebook converter - www.ebook-converter.com******

******ebook converter DEMO - www.ebook-converter.com*******

******Created by ebook converter - www.ebook-converter.com******

COLON CANCER:
QUESTION: What gene mutations are associated with colon cancer?
.
.
.
.
.
ANSWER: The genetic defects and molecular abnormalities associated with the
development and progression of colorectal adenomas and carcinoma are as follows:
- Mutations may cause activation of oncogenes (K-ras) and/or inactivation of tumorsuppressor genes [APC, DCC (deleted in colorectal carcinoma), p53].
- Colorectal carcinoma is thought to develop from adenomatous polyps by accumulation
of these mutations.
.
.

******ebook converter DEMO - www.ebook-converter.com*******

******Created by ebook converter - www.ebook-converter.com******

******ebook converter DEMO - www.ebook-converter.com*******

******Created by ebook converter - www.ebook-converter.com******

COMPARTMENT SYNDROME:
QUESTION: When performing a four-compartment fasciotomy for compartment
syndrome, medial and lateral incisions are created. Which compartments is opened
through the medial incision? Through the lateral incision?
.
.
.
.
.
.
ANSWER:
- Compartment pressures are relieved in the leg by medial and lateral incisions.
- Through the medial incision, long openings are then made in the fascia of the
superficial and deep posterior compartments.
- Through the lateral incision, the anterior and peroneal compartments are opened.
.

******ebook converter DEMO - www.ebook-converter.com*******

******Created by ebook converter - www.ebook-converter.com******

******ebook converter DEMO - www.ebook-converter.com*******

******Created by ebook converter - www.ebook-converter.com******

CONDYLOMA ACCUMINATA:
QUESTION: How is the treatment for extensive perianal condyloma accuminata (anal
warts)?.
.
.
.
.
ANSWER:
- Treatment of anal condyloma depends on the location and extent of disease.
- Small warts on the perianal skin and distal anal canal may be treated in the office with
topical application of bichloracetic acid or podophyllin.
- Although 60 to 80% of patients will respond to these agents, recurrence and
reinfection are common.
- Imiquimod (Aldara) is an immunomodulator that recently was introduced for topical
treatment of several viral infections, including anogenital condyloma.
- Initial reports suggest that this agent is highly effective in treating condyloma located
on the perianal skin and distal anal canal.
- Larger and/or more numerous warts require excision and/or fulguration in the
operating room.
- Excised warts should be sent for pathologic examination to rule out dysplasia or
malignancy.
- It is important to note that prior use of podophyllin may induce histological changes
that mimic dysplasia.
.
.

******ebook converter DEMO - www.ebook-converter.com*******

******Created by ebook converter - www.ebook-converter.com******

******ebook converter DEMO - www.ebook-converter.com*******

******Created by ebook converter - www.ebook-converter.com******

CO2 EMBOLISM:
QUESTION: A 35 year old woman undergoing routine laparoscopic bilateral tubal
ligation develops severe hypotension, tachycardia, and drop in her end tidal CO2. The
anesthesiologist states the patient still has bilateral breath sounds. What is the most
likely diagnosis?
.
.
.
.
.
ANSWER:
- The most likely diagnosis in this patient is CO2 embolism.
- End tidal CO2 specifically reflects the exchange of CO2 from blood to the alveolus.
- A gradual rise in ETCO2 usually reflects impaired exchange from lung collapse or
atelectasis.
- A sudden drop in ETCO2 can be from something simple like disconnection from the
ventilator or something more serious such as an embolus.
- The abrupt drop in ETCO2 following an embolus is from the interruption of CO2
exchange at the alveolar level.
- Because of the hypotension associated with a drop in ETCO2 in the above patient, the
most likely diagnosis is CO2 embolism.
.
.

******ebook converter DEMO - www.ebook-converter.com*******

******Created by ebook converter - www.ebook-converter.com******

******ebook converter DEMO - www.ebook-converter.com*******

******Created by ebook converter - www.ebook-converter.com******

COX INHIBITORS:
QUESTION: Which drugs irreversibly inhibits platelet COX (cyclooxygenase)? What is
the mechanism of action of ibuprofen, celebrex, clopidogrel, and aspirin.
.
.
.
.
..
ANSWER:
- Arachidonic acid released from the platelet membranes is converted by COX to
prostaglandin G2 (PGG2) and then to prostaglandin H2 (PGH2), which, in turn, is
converted to TXA2.
- TXA2 has potent vasoconstriction and platelet aggregation effects.
- Arachidonic acid may also be shuttled to adjacent endothelial cells and converted to
prostacyclin (PGI2), which is a vasodilator and acts to inhibit platelet aggregation.
- Platelet COX is irreversibly inhibited by aspirin and reversibly blocked by NSAIDs
but is not affected by COX-2 inhibitors.
- Ibuprofen is a nonsteroidal anti-inflammatory drug (NSAID) and reversibly affects
platelet COX.
- Both aspirin and clopidogrel irreversibly inhibit platelet function, clopidogrel through
selective irreversible inhibition of ADP-induced platelet aggregation and aspirin
through irreversible acetylation of platelet prostaglandin synthase.
- Celebrex is a COX-2 inhibitor and therefore does not affect platelet COX.
.
.

******ebook converter DEMO - www.ebook-converter.com*******

******Created by ebook converter - www.ebook-converter.com******

******ebook converter DEMO - www.ebook-converter.com*******

******Created by ebook converter - www.ebook-converter.com******

CRYOPRECIPITATE:
QUESTION:When should cryoprecipitate be given to a patient needing a massive
transfusion of packed RBCs?
.
.
.
.
.
.
.
TABLE: COMPONENT THERAPY ADMINISTRATION DURING MASSIVE
TRANSFUSION
FRESH FROZEN PLASMA (FFP)
- As soon as the need for massive transfusion is recognized.
- For every 6 units of red blood cells (RBCs), give 6 units of FFP (1:1 ratio)
PLATELETS
- For every 6 units of RBCs and plasma, give one 6-pack of platelets.
- Six random-donor platelet packs = 1 apheresis platelet unit
- Keep platelet counts >100,000 ul/ during active hemorrhage control
- After first 6 units of RBCs, check fibrinogen level.
- If 100 mg/dL, give 20 units of cryoprecipitate (2g fibrinogen)
- Repeat as needed, depending on fibrinogen level.

QUESTION: True or false: cryoprecipitate contains vWF-VIII


.
******ebook converter DEMO - www.ebook-converter.com*******

******Created by ebook converter - www.ebook-converter.com******


.
.
ANSWER: True. Cryoprecipitate contains fibrinogen and vWF-VIII
.
.
.

******ebook converter DEMO - www.ebook-converter.com*******

******Created by ebook converter - www.ebook-converter.com******

QUESTION: Cryoprecipitate is used in patients with what disease?


.
.
.
ANSWER: Cryoprecipitate is used in von Willebrand's disease and hemophilia A
(factor VIII deficiency). It is also used in DIC if fibrinogen is low.

******ebook converter DEMO - www.ebook-converter.com*******

******Created by ebook converter - www.ebook-converter.com******

******ebook converter DEMO - www.ebook-converter.com*******

******Created by ebook converter - www.ebook-converter.com******

DDAVP: DESMOPRESSIN:
QUESTION: What is the mechanism of action of DDAVP?
.
.
.
.
.
ANSWER:
- The mechanism of action DDAVP is that it causes release of vWF.

.
.
.
.
.
QUESTION: What conditions are usually treated with DDAVP?
.
.
.
.
.
ANSWER:
- Conditions treated with DDAVP include uremia, von Willebrand disease, and Aspirin.

******ebook converter DEMO - www.ebook-converter.com*******

******Created by ebook converter - www.ebook-converter.com******

******ebook converter DEMO - www.ebook-converter.com*******

******Created by ebook converter - www.ebook-converter.com******

DESMOID TUMORS:
QUESTION: What gene is associated with desmoid tumors of the chest wall?
.
.
..
.
.
ANSWER: Adenomatous polyposis coli (APC)
- Desmoid tumors have recently been shown to possess alterations in the adenomatous
polyposis coli / Beta-catenin pathway, and cyclin D1 dysregulation is thought to play a
significant role in their pathogenesis.
- Associations with other diseases and conditions are well documented, especially
those with similar alterations in the adenomatous polyposis coli pathway, such as
familial adenomatous polyposis (Gardner's syndrome).

******ebook converter DEMO - www.ebook-converter.com*******

******Created by ebook converter - www.ebook-converter.com******

******ebook converter DEMO - www.ebook-converter.com*******

******Created by ebook converter - www.ebook-converter.com******

DIABETES INSIPIDUS:
QUESTION: Is diabetes insipidus due to elevated ADH or low ADH?
.
.
.
ANSWER: Diabetes insipidus is due to low ADH
.
.
.
QUESTION Do patients with diabetes insipidus have high urine output or low urine
output?
.
.
.
ANSWER: Patients with diabetes insipidus have high urine output
.
.
.
QUESTION: Do patients with diabetes insipidus have a low urine specific gravity or a
high urine specific gravity?
.
.
.
ANSWER: Patients with diabetes insipidus have a low urine specific gravity
.
.
.
QUESTION: Do patients diabetes insipidus have high serum osmolarity or low serum
osmolarity?
.
.
.
ANSWER: Patients with diabetes insipidus have a high serum osmolarity
.
.
.
******ebook converter DEMO - www.ebook-converter.com*******

******Created by ebook converter - www.ebook-converter.com******

QUESTION: Do patients with diabetes insipidus have high serum Na or low serum
Na?
.
.
.
ANSWER: Patients with diabetes insipidus have a high serum sodium concentration
.
.
.
QUESTION. Do patients with diabetes insipidus have a high urine sodium or a low
urine sodium?
.
.
.
ANSWER: Patients with diabetes insipidus have a low urine sodium

******ebook converter DEMO - www.ebook-converter.com*******

******Created by ebook converter - www.ebook-converter.com******

******ebook converter DEMO - www.ebook-converter.com*******

******Created by ebook converter - www.ebook-converter.com******

DIAPHRAGMATIC INJURIES:
QUESTION: Diaphragm injuries involve which side of the chest more commonly, right
or left? Are diaphragmatic injuries easy are hard to detect on CT scan?
.
.
.
.
ANSWER:
- Diaphragm injuries are more common with blunt trauma and are more common on the
left (liver projects the right side)
- Diaphragm injuries are hard to find on CT scan unless there is gross herniation

******ebook converter DEMO - www.ebook-converter.com*******

******Created by ebook converter - www.ebook-converter.com******

******ebook converter DEMO - www.ebook-converter.com*******

******Created by ebook converter - www.ebook-converter.com******

DIFFUSE ESOPHAGEAL SPASMS:


QUESTION: What is the medical treatment of diffuse esophageal spasm?
.
.
.
ANSWER: The medical treatment of diffuse esophageal spasm includes calcium
channel blockers, nitrates, and antispasmodics

******ebook converter DEMO - www.ebook-converter.com*******

******Created by ebook converter - www.ebook-converter.com******

******ebook converter DEMO - www.ebook-converter.com*******

******Created by ebook converter - www.ebook-converter.com******

DISSEMINATED INTRAVASCULAR COAGULOPATHY:


(DIC:)
QUESTION: What hematologic factors do you take into account when diagnosing
someone with DIC?
.
.
.
ANSWER: Platelets, INR, PTT, fibrinogen, and fibrin split products.
.
.
.
QUESTION: Are platelets high or low in DIC?
.
.
.
ANSWER: Platelets are low in DIC.
.
.
.
QUESTION: Is the INR normal or prolonged in DIC?
.
.
.
ANSWER: The INR is prolonged in DIC.
.
.
.
QUESTION: Is the PTT normal or prolonged in DIC?
******ebook converter DEMO - www.ebook-converter.com*******

******Created by ebook converter - www.ebook-converter.com******


.
.
.
ANSWER: The PTT is prolonged in DIC.
.
.
.

******ebook converter DEMO - www.ebook-converter.com*******

******Created by ebook converter - www.ebook-converter.com******

QUESTION: Is the fibrinogen level high or low in DIC?


.
.
.
ANSWER: The fibrinogen level is low in DIC.
.
.
.
QUESTION: Is the fibrin split products high or low in DIC?
.
.
.
ANSWER: Fibrin split products are elevated in DIC.

******ebook converter DEMO - www.ebook-converter.com*******

******Created by ebook converter - www.ebook-converter.com******

******ebook converter DEMO - www.ebook-converter.com*******

******Created by ebook converter - www.ebook-converter.com******

DIVERTICULITIS:
QUESTION: Approximately 5% of patients with complicated diverticulitis develop a
fistula to an adjacent organ. The most commonly involved organ is what?
.
.
.
.
ANSWER: The most commonly involved organ is the bladder.
- Approximately 5% of patients with complicated diverticulitis develop fistulas
between the colon and an adjacent organ.
- Colovesical fistulas are most common, followed by colovaginal and colo-enteric
fistulas.
- Colocutaneous fistulas are a rare complication of diverticulitis.
.
.
.
.
.

******ebook converter DEMO - www.ebook-converter.com*******

******Created by ebook converter - www.ebook-converter.com******

******ebook converter DEMO - www.ebook-converter.com*******

******Created by ebook converter - www.ebook-converter.com******

DNA:
Q: Approximately how many genes are present in the human genome?
.
.
.
.
.
A:
- The human genome has an estimated 25,000 to 30,000 genes, and overall it is 99.9%
identical in all people.

******ebook converter DEMO - www.ebook-converter.com*******

******Created by ebook converter - www.ebook-converter.com******

******ebook converter DEMO - www.ebook-converter.com*******

******Created by ebook converter - www.ebook-converter.com******

DUODENAL ATRESIA:
QUESTION: A newborn has bilious vomiting. Plain films reveal a distended gastric air
bubble and a markedly dilated proximal duodenum. What is the most likely diagnosis?
.
.
.
.
.
ANSWER: Duodenal atresia
- The history and radiograph findings are consistent with duodenal atresia.
- Duodenal atresia occurs because of failure of vacuolization of the duodenum from its
solid core state.
.
.
.
.
.
QUESTION: What conditions are associated with duodenal atresia?
.
.
.
.
.
ANSWER: Down syndrome, biliary atresia
- It is associated with prematurity, Down syndrome, maternal polyhydramnios,
malrotation, annular pancreas, and biliary atresia.
.
******ebook converter DEMO - www.ebook-converter.com*******

******Created by ebook converter - www.ebook-converter.com******


.
.
.
.
.

******ebook converter DEMO - www.ebook-converter.com*******

******Created by ebook converter - www.ebook-converter.com******

QUESTION: What is the typical presentation of an infant with duodenal atresia?


.
.
.
.
.
ANSWER:
- In most cases, the duodenal obstruction is distal to the ampulla of Vater, and infants
present with bilious emesis in the neonatal period.
.
.
.
.
.
.
QUESTION: What is the classic radiographic finding suggesting duodenal atresia?
.
.
.
.
.
ANSWER: double-bubble sign
- The classic radiographic finding is the "double-bubble sign" (an air-filled stomach, a
functioning pylorus, and a distended proximal duodenal bulb).
- If there is no distal bowel gas, complete atresia is confirmed and no further studies are
******ebook converter DEMO - www.ebook-converter.com*******

******Created by ebook converter - www.ebook-converter.com******


necessary.
- Conversely, if distal air is present, an upper gastrointestinal contrast study should be
done because other diagnoses are possible.
- This is important to exclude midgut volvulus.
- The finding of distal air in association with a double bubble could also indicate a
duodenal stenosis or web or an annular pancreas that does not cause a complete
obstruction.
.
.
.

******ebook converter DEMO - www.ebook-converter.com*******

******Created by ebook converter - www.ebook-converter.com******

QUESTION: What is the treatment for duodenal atresia?


.
.
.
.
.
ANSWER:
- The treatment of duodenal atresia is surgical bypass of the obstruction as either a sideto-side or proximal transverse-to-distal longitudinal duodenoduodenostomy or a
duodenojejunostomy.
- When the proximal duodenum is markedly dilated, a tapering duodenoplasty may be
performed.

******ebook converter DEMO - www.ebook-converter.com*******

******Created by ebook converter - www.ebook-converter.com******

******ebook converter DEMO - www.ebook-converter.com*******

******Created by ebook converter - www.ebook-converter.com******

ECMO:
QUESTION: What are the indications for ECMO in neonates?
.
.
.
.
.
ANSWER:
- In neonates with respiratory distress syndrome, management includes high-frequency
ventilation surfactant and inhaled nitric oxide.
- When those interventions fail, ECLS is used.
- ECLS can be performed by either venovenous or venoarterial cannulation.
- The major indications for ECLS include meconium aspiration, respiratory distress
syndrome, persistent pulmonary hypertension, sepsis, and CDH.

******ebook converter DEMO - www.ebook-converter.com*******

******Created by ebook converter - www.ebook-converter.com******

******ebook converter DEMO - www.ebook-converter.com*******

******Created by ebook converter - www.ebook-converter.com******

ENDOPLASMIC RETICULUM:
QUESTION: Which type of ER makes cytoplasmic proteins? Which type of ER makes
proteins for export?
.
.
.
.
.
ANSWER:
- The endoplasmic reticulum that makes cytoplasmic proteins is the smooth endoplasmic reticulum.
- The endoplasmic reticulum that makes proteins for export is the rough endoplasmic reticulum.

******ebook converter DEMO - www.ebook-converter.com*******

******Created by ebook converter - www.ebook-converter.com******

******ebook converter DEMO - www.ebook-converter.com*******

******Created by ebook converter - www.ebook-converter.com******

ERRORS: TYPE I ERRORS: TYPE II ERRORS: TYPE III


ERRORS:
QUESTION: In terms of the null hypothesis, what kind of an error is a Type I error?
.
.
.
ANSWER: A Type I error rejects the null hypothesis incorrectly. It falsely assumes
there was a difference when no difference exists.
.
.
.
QUESTION: In terms of the null hypothesis, what kind of error is a Type II error?
.
.
.
ANSWER: A Type II error accepts the null hypothesis incorrectly. This can be due to
small sample size. For example, the treatments are interpreted as equal when there is
actually a difference
.
.
.
QUESTION: What is a Type III error?
.
.
.
ANSWER: A Type III error is made when the conclusion is not supported by the data
from the study

******ebook converter DEMO - www.ebook-converter.com*******

******Created by ebook converter - www.ebook-converter.com******

******ebook converter DEMO - www.ebook-converter.com*******

******Created by ebook converter - www.ebook-converter.com******

ERYTHROMYCIN:
QUESTION: Erythromycin increases gastrointestinal motility by which mechanism?
.
.
.
ANSWER: Erythromycin binds the motilin receptor, and activates it to increase
gastrointestinal motility

******ebook converter DEMO - www.ebook-converter.com*******

******Created by ebook converter - www.ebook-converter.com******

******ebook converter DEMO - www.ebook-converter.com*******

******Created by ebook converter - www.ebook-converter.com******

ESOPHAGEAL PERFORATION: ESOPHAGEAL


RUPTURE:
QUESTION: What is the key to survival in esophageal rupture?
.
.
.
ANSWER: In esophageal rupture, the key to survival is early diagnosis
.
.
.
.
QUESTION: What percentage of patients with esophageal rupture die if the diagnosis is
made after 36 hours?
.
.
.
ANSWER: 85% of patients who are diagnosed with esophageal perforation more than
36 hours after the incident die.
.
.
.
.

******ebook converter DEMO - www.ebook-converter.com*******

******Created by ebook converter - www.ebook-converter.com******

******ebook converter DEMO - www.ebook-converter.com*******

******Created by ebook converter - www.ebook-converter.com******

EXTRINSIC PATHWAY:
QUESTION: Which two factors are involved in the initiation of the extrinsic pathway
(PT)?
.
.
.
.
.
ANSWER: In the extrinsic pathway, tissue factor reacts with factor VII to initiate the
coagulation process leading up to fibrin production
.
.
.
.
.
QUESTION: Summarize the extrinsic pathway: include the lab test associated with it,
the factors involved, and the final product
.
.
.
.
.
ANSWER: Extrinsic path tested with PT. Tissue factor + VII --> activated X --> fibrin
.
.
.

******ebook converter DEMO - www.ebook-converter.com*******

******Created by ebook converter - www.ebook-converter.com******

******ebook converter DEMO - www.ebook-converter.com*******

******Created by ebook converter - www.ebook-converter.com******

FACTORS:
QUESTION: Which factors are present at low levels in banked blood?
.
.
.
.
.
ANSWER:
- The factors that are at low levels in banked blood are factor V and factor VIII.

.
.
.
.
.
QUESTION: Which factor is not made by the liver?
.
.
.
.
.
ANSWER:
- The only factor that is not made in the liver (but instead is made by the reticuloendothelial cells) is Factor VIII.

.
.
.
.
******ebook converter DEMO - www.ebook-converter.com*******

******Created by ebook converter - www.ebook-converter.com******


QUESTION: Which factor is activated during cardiopulmonary bypass?
.
.
.
.
.
ANSWER:
- The factor that is activated during cardiopulmonary bypass is Factor XII (Hageman factor) necessitating a need
for Heparin.

******ebook converter DEMO - www.ebook-converter.com*******

******Created by ebook converter - www.ebook-converter.com******

******ebook converter DEMO - www.ebook-converter.com*******

******Created by ebook converter - www.ebook-converter.com******

FACTOR V LEIDEN:
QUESTION: True or false: The most common risk factor for spontaneous venous thromboembolism is factor
V leiden.

.
.
.
.
.
ANSWER: TRUE
.
.
.
.
.

QUESTION: What exactly is the problem in Factor V Leiden?


.
.
.
.
.
ANSWER:
- The problem in Factor V Leiden is a defect on factor V, which leads to resistance to activated protein C.

.
.
.
.
******ebook converter DEMO - www.ebook-converter.com*******

******Created by ebook converter - www.ebook-converter.com******


.

******ebook converter DEMO - www.ebook-converter.com*******

******Created by ebook converter - www.ebook-converter.com******

QUESTION: Patients with factor V Leiden are predisposed to thrombosis because they
have a genetic mutation in factor V which is what effect made on factor V?
.
.
.
.
.
.
ANSWER:
- A third major mechanism of inhibition of thrombin formation is the protein C system.
- On its formation, thrombin binds to thrombomodulin and activates protein C to
activated protein C (APC), which then forms a complex with its cofactor, protein S, on
a phospholipid surface.
- The APC-protein S complex cleaves factors Va and VIIIa so they are no longer able to
participate in the formation of tissue factor-VIIa or prothrombinase complexes.
- Of interest is an inherited form of factor V that carries a genetic mutation, called
factor V Leiden, that is resistant to cleavage by APC and thus remains active
(procoagulant).
- Patients with factor V Leiden are predisposed to venous thromboembolic events.
.
.

******ebook converter DEMO - www.ebook-converter.com*******

******Created by ebook converter - www.ebook-converter.com******

******ebook converter DEMO - www.ebook-converter.com*******

******Created by ebook converter - www.ebook-converter.com******

FACTOR VIII:
QUESTION: Factor VIII is the only factor not made in the liver. Where is it made?
.
.
.
ANSWER: Factor VIII is the only factor not made in the liver. It is made by
reticuloendothelial system
.

******ebook converter DEMO - www.ebook-converter.com*******

******Created by ebook converter - www.ebook-converter.com******

******ebook converter DEMO - www.ebook-converter.com*******

******Created by ebook converter - www.ebook-converter.com******

FAMILIAL ADENOMATOUS POLYPOSIS: FAP:


QUESTION: In patients who have undergone total proctocolectomy for FAP, what is the
most common cause of death?
.
.
.
.
ANSWER:
- The most common cause of death in these patients is duodenal cancer.
- Patients with FAP can also develop duodenal polyps which should be followed and
removed.
- These polyps have malignant potential.
.
.
.
.
QUESTION: Is familial adenomatous polyposis autosomal dominant or recessive?
.
.
.
ANSWER: FAP is autosomal dominant
.
.
.
QUESTION:: At what age do FAP patients get cancer?
.
.
.
ANSWER: Patients with FAP develop cancer by age 40
.
.
******ebook converter DEMO - www.ebook-converter.com*******

******Created by ebook converter - www.ebook-converter.com******


.
QUESTION: What is the gene mutated in FAP?
.
.
ANSWER: The APC gene is mutated

******ebook converter DEMO - www.ebook-converter.com*******

******Created by ebook converter - www.ebook-converter.com******

FATTY ACIDS:
QUESTION: How do short and medium chain fatty acids enter the bloodstream?
.
.
.
.
ANSWER:
- The metabolism of short chain fatty acids and medium chain fatty acids is that they are absorbed by the
intestines by simple diffusion and enter the portal system. (TRICKY)

.
.
.
.
.
QUESTION: How do long chain fatty acids enter the bloodstream?
.
.
.
.
.
ANSWER:
- The metabolism of long chain fatty acids is that they are in the form of micelles in the intestines where they fuse
with enterocytes, then they are carried with chylomicrons, and then enter the bloodstream via the lymphatics.
(Easy, 3)

******ebook converter DEMO - www.ebook-converter.com*******

******Created by ebook converter - www.ebook-converter.com******

******ebook converter DEMO - www.ebook-converter.com*******

******Created by ebook converter - www.ebook-converter.com******

FIBRIN:
QUESTION: Which factor is involved in cross-linking fibrin to form a fibrin plug?
.
.
.
.
.
ANSWER:
- The factor involved in cross-linking fibrin to form a fibrin plug is Factor XIII (factor 13).

******ebook converter DEMO - www.ebook-converter.com*******

******Created by ebook converter - www.ebook-converter.com******

******ebook converter DEMO - www.ebook-converter.com*******

******Created by ebook converter - www.ebook-converter.com******

FLAIL CHEST:
QUESTION: In the adult, the hypoxemia seen immediately after a flail chest injury is
due to what?
.
.
.
.
.
ANSWER: The underlying pulmonary contusion.
- In those patients in whom early respiratory failure develops, it is the result of an
underlying pulmonary contusion.
- Pain control is another important element of management. Most patients do not need
fixation of the flail segment.

******ebook converter DEMO - www.ebook-converter.com*******

******Created by ebook converter - www.ebook-converter.com******

******ebook converter DEMO - www.ebook-converter.com*******

******Created by ebook converter - www.ebook-converter.com******

FLUIDS: NORMAL SALINE: LACTATED RINGERS:


QUESTION: Normal saline has how many mEq of Na? How many of Cl?
.
.
.
.
.
ANSWER: Normal saline has 154 mEq Na and 154 mEq K.
.
.
.
.
.
.
QUESTION: What electrolytes are in lactated ringers, but not in normal saline?
.
.
.
ANSWER: K, Ca, and bicarb are in lactated ringers, but not in normal saline.
.
.
.
.
.
.
QUESTION: Is the mEq of Na higher in normal saline or higher in lactated ringers?
.
.
.
ANSWER: Lactated ringers has 130 mEq of Na, while normal saline has 154 mEq of
Na.
******ebook converter DEMO - www.ebook-converter.com*******

******Created by ebook converter - www.ebook-converter.com******


.
.
QUESTION: Lactated ringers has how many mEq of Na, K, Ca, Cl, and bicarb?
.
.
ANSWER: Lactated ringers has: Na 130, K 4, Ca 2.7, Cl 109, bicarb 28 mEqs.

******ebook converter DEMO - www.ebook-converter.com*******

******Created by ebook converter - www.ebook-converter.com******

******ebook converter DEMO - www.ebook-converter.com*******

******Created by ebook converter - www.ebook-converter.com******

FLUIDS: MAINTENANCE FLUIDS:


The approximate IV rate for maintenance fluids for a 50 kg patient would be
.
a. 75 ml/hr
b. 90 ml/hr
c. 105 ml/hr
d. 120 ml/hr
.
.
.
.
.
- The answer is B
- Once the daily total is established, dividing by 24 will give an approximate hourly
rate.
- Alternatively, dividing by 25 (instead of 24) gives a rapid approximate rate.
- In other words, the hourly IV rate will be
.
- 4ml/kg/hour for the 1st 10kg
- 2ml/kg/hour for the 2nd 10kg
- 1ml/kg/hour for each kg>20kg
.
.
.
- In this example
- 4 X 10 = 40 (for the 1st 10kg)
- 2 X 10 = 20 (for the 2nd 10kg)
******ebook converter DEMO - www.ebook-converter.com*******

******Created by ebook converter - www.ebook-converter.com******


- and 1X30=30 (for the remaining kg).
- 40 + 20 + 30 = 90 ml/hr
- The number if one divides by 24 instead of 25 is 87.5ml/hr.
- For the first 0 to 10 kg, give 100cc/kg per day
- For the next 10-20 kg, give an additional 50cc/kg per day
- For weight >20kg, give an additional 20cc/kg per day

******ebook converter DEMO - www.ebook-converter.com*******

******Created by ebook converter - www.ebook-converter.com******

FOCAL NODULAR HYPERPLASIA: FNH:


QUESTION: A 40 year old woman on no medications or past medical problems
presents with intermittent abdominal pain. She undergoes a contrasted abdominal CT
scan which shows a 4 cm liver lesion with a definite central stellate scar. Sulfur
colloid scan shows increased uptake. AFP is normal. There is no peripheral to central
enhancement on the contrasted CT scan. The most appropriate next step is to do what?
.
..
.
.
.
ANSWER:
- Because of the central stellate scar and the positive sulfur colloid scan, this lesion is
most consistent with focal nodular hyperplasia and does not need resection.
- You should follow the lesion to make sure it's not getting any bigger.
- Finding the central stellate scar and having uptake on sulfur colloid scan are not
100% so you should follow the lesion for awhile.
- Kupffer cells are what light up on the sulfur colloid scan.
.
.

******ebook converter DEMO - www.ebook-converter.com*******

******Created by ebook converter - www.ebook-converter.com******

******ebook converter DEMO - www.ebook-converter.com*******

******Created by ebook converter - www.ebook-converter.com******

FORMIC ACID BURNS:


QUESTION: Formic acid burns are associated with what hematologic abnormality?
.
.
.
.
.
ANSWER: Hemoglobinuria
- Formic acid has been known to cause hemolysis and hemoglobinuria.

******ebook converter DEMO - www.ebook-converter.com*******

******Created by ebook converter - www.ebook-converter.com******

******ebook converter DEMO - www.ebook-converter.com*******

******Created by ebook converter - www.ebook-converter.com******

FUNCTIONAL GENOMICS:
QUESTION: What is functional genomics?
.
.
.
.
.
ANSWER: Functional genomics is a term used to describe a) Transcription of DNA, b)
Translation of RNA, and c) Proteomics. .
- Functional genomics seeks to assign a biochemical, physiologic, cell biologic, and/or
developmental function to each predicted gene.
- Genomics and proteomics are the study of the genetic composition of a living
organism at the DNA and protein level, respectively.
- The study of the relationship between genes and their cellular functions is called
functional genomics.

******ebook converter DEMO - www.ebook-converter.com*******

******Created by ebook converter - www.ebook-converter.com******

******ebook converter DEMO - www.ebook-converter.com*******

******Created by ebook converter - www.ebook-converter.com******

GALLSTONE ILEUS:
QUESTION: What is the primary surgery for patients with gallstone ileus?
.
.
.
.
.
ANSWER: The primary surgery in this patient is to relieve the small bowel obstruction.
That involves feeling for the gallstone, opening the ileum, and removing the stone. The
secondary procedure, if the patient can tolerate it, is cholecystectomy and closure of the
hole in the duodenum.

******ebook converter DEMO - www.ebook-converter.com*******

******Created by ebook converter - www.ebook-converter.com******

******ebook converter DEMO - www.ebook-converter.com*******

******Created by ebook converter - www.ebook-converter.com******

GASTRIC CANCER: GASTRIC ADENOCARCINOMA:


QUESTION: In resected gastric adenocarcinoma, what is considered a minimum gross
margin?
a. 2cm
b. 3cm
c. 4cm
d. 5cm
.
.
.
.
.
- The answer is D.
GASTRIC CANCER REVIEW
Statistics and Epidemiology
- The incidence of stomach cancer varies significantly among different regions of the
world.
- The age-adjusted incidence is highest in Japan (62.1 per 100,000 men, 26.1 per
100,000 women).
- In comparison, the rates are much lower in North America (7.4 per 100,000 (4.4 to 3.4
per 100,000 men, 2.5 to 3.6 per 100,000 women).
- The difference in risk by country is presumed to be primarily due to differences in
dietary factors.
- Fortunately, a steady decline is being observed in the incidence and mortality rates of
gastric cancer.
- This may be related to improvements in preservation and storage of foods as well as
due to changes in the prevalence of H. pylori.
******ebook converter DEMO - www.ebook-converter.com*******

******Created by ebook converter - www.ebook-converter.com******


Pathophysiology
- Some gastric tumors, particularly the diffuse variety, are quite infiltrative and tumor
cells can extend well beyond the tumor mass; thus, gross margins beyond 5 cm may be
desirable.
Risk Factors
- The risk is increased by high consumption of preserved salted foods such as meats and
pickles, and decreases by high intake of fruits and vegetables.
- There also is some international variation in the incidence of infection with
Helicobacter pylori, which is known to play a major role in gastric cancer
development.

- Surgical considerations: FROZEN SECTIONS


- Frozen section confirmation of negative margins is important when performing
operation for cure, but it is less important in patients with nodal metastases beyond the
N1 nodal basin.
- Surgical considerations: MARGIN
- The goal of curative surgical treatment is resection of all tumor (i.e. R0 resection).
- Thus, all margins (proximal, distal, and radial) should be negative and an adequate
lymphadenectomy performed.
- Generally, the surgeon strives for a grossly negative margin of at least 5 cm.
- Some gastric tumors, particularly the diffuse variety, are quite infiltrative and tumor
cells can extend well beyond the tumor mass; thus, gross margins beyond 5 cm may be
desirable.
- Surgical considerations: LYMPHADENECTOMY
-An adequate lymphadenectomy should be performed.

******ebook converter DEMO - www.ebook-converter.com*******

******Created by ebook converter - www.ebook-converter.com******

******ebook converter DEMO - www.ebook-converter.com*******

******Created by ebook converter - www.ebook-converter.com******

GASTRIC LYMPHOMA:
QUESTION: A 50 year old man presents with epigastric pain unrelieved with proton
pump inhibitors. You perform an upper endoscopy and a mass is found in the stomach.
You biopsy it and it comes back lymphoma. Abdominal CT scan shows that the mass is
full thickness and 3/4 of the stomach is involved. What is the most appropriate therapy?
.
.
.
.
.
.
.
.
ANSWER.
- Treatment or gastric lymphoma is chemo-XRT
- Surgery is indicated only for complications of the disease (bleeding, perforation,
obstruction) or possibly for stage I disease limited to the mucosa.

******ebook converter DEMO - www.ebook-converter.com*******

******Created by ebook converter - www.ebook-converter.com******

******ebook converter DEMO - www.ebook-converter.com*******

******Created by ebook converter - www.ebook-converter.com******

GAStRIC VARICES:
QUESTION: A patient with a history of pancreatitis develops severe bleeding from
gastric varices. You do not see any esophageal varices on EGD. What is the best
treatment for this condition?
.
.
.
.
ANSWER:
- Gastric varices without esophageal varices are most likely from a thrombosed splenic
vein related to pancreatitis and not cirrhosis. Treatment is splenectomy.
- Less than 2-3% of patients with cirrhosis get gastric varices without esophageal
varices.
- Treatment in this patient is splenectomy.

******ebook converter DEMO - www.ebook-converter.com*******

******Created by ebook converter - www.ebook-converter.com******

******ebook converter DEMO - www.ebook-converter.com*******

******Created by ebook converter - www.ebook-converter.com******

GASTRIN:
QUESTION: What is the molecular mechanism of action of how Acetylcholine and
gastrin cause HCl production?
.
.
.
ANSWER: Acetylcholine and gastrin activate PIP,DAG to increase Ca, which activates
protein kinase C which increases HCl production
ACh or Gastrin --> PIP, DAG, Ca, Protein Kinase C --> HCl production
.
.
.
.
.
QUESTION: Which cells produce Gastrin?
.
.
.
ANSWER: Gastrin is produced by gastric antrum G cells
Mnemonic: Gastric is made by G cells
.
.
.
QUESTION: What molecules stimulate the production of Gastrin?
.
.
.
ANSWER: Peptides and amino acids in the stomach, vagal input with acetylcholine, and
calcium all stimulate gastrin release
.
.
.
******ebook converter DEMO - www.ebook-converter.com*******

******Created by ebook converter - www.ebook-converter.com******

QUESTION: What molecules inhibit the production of Gastrin?


.
.
.
ANSWER: H+ in the duodenum, a pH less than 3.0, and somatostatin inhibit gastrin
release
.
.
.
QUESTION: What does gastrin have to do with antrectomy?
.
.
.
ANSWER: If a patient has high levels of gastrin, an antrectomy would remove the
antrum G cells that produce gastrin
.
.
.

******ebook converter DEMO - www.ebook-converter.com*******

******Created by ebook converter - www.ebook-converter.com******

******ebook converter DEMO - www.ebook-converter.com*******

******Created by ebook converter - www.ebook-converter.com******

GASTROESOPHAGEAL REFLUX DISEASE: (GERD:)


QUESTION: What is the most sensitive test for gastroesophageal reflux disease?
.
.
.
ANSWER: The most sensitive test for GERD is the 24 hour pH study.
.
.
.
.
.
QUESTION: Does the esophagus have a serosal layer?
.
.
.
ANSWER: Unlike the rest of the alimentary tract the esophagus lacks a true serosal
layer
.
.
.
QUESTION: Which layer of the esophagus is the strongest layer? How is that different
in the small bowel?
.
.
.
ANSWER: The strongest layer in the esophagus is the mucosa. In the small bowel, the
strongest layer is the submucosa.
.
.
.
QUESTION: Are the esophageal sphincters relaxed or contracted at rest?
******ebook converter DEMO - www.ebook-converter.com*******

******Created by ebook converter - www.ebook-converter.com******


.
.
.
ANSWER: Esophageal sphincters are contracted at rest

******ebook converter DEMO - www.ebook-converter.com*******

******Created by ebook converter - www.ebook-converter.com******

GASTROINTESTINAL BLEEDING:
QUESTION: A 50 year old man has severe hematochezia requiring 6 units of blood in
12 hours. The bleeding is ongoing. His blood pressure in 110/60 and his HR is 110.
You do not find anything on anoscopy. You place an NG tube and get back bile but no
blood. How was the most appropriate next step in your work-up?
.
.
.
.
.
ANSWER:
- This would be considered massive bleeding because of the transfusion requirement.
- You are unlikely to find anything on colonoscopy because there is too much blood.
- You should not go directly to the OR for a subtotal colectomy unless the patient was
hypotensive despite aggressive resuscitation attempts.
- Try to localize the area first. Angiography is the best choice, which will localize the
lesion (and possibly treat with embolization).
- Then take the patient to the OR for colectomy at the appropriate location.
.

******ebook converter DEMO - www.ebook-converter.com*******

******Created by ebook converter - www.ebook-converter.com******

******ebook converter DEMO - www.ebook-converter.com*******

******Created by ebook converter - www.ebook-converter.com******

GASTROINTESTINAL STROMAL TUMORS: GIST:


QUESTION: What is the treatment for gastrointestinal stromal tumor of the stomach that
has metastasized to the liver after resection?
.
.
.
.
.
ANSWER: The next step in treatment is metastatic GIST is Gleevec, a tyrosine kinase
inhibitor. Gleevec has been found to be extremely effective for metastatic
gastrointestinal stromal tumors.
.

******ebook converter DEMO - www.ebook-converter.com*******

******Created by ebook converter - www.ebook-converter.com******

******ebook converter DEMO - www.ebook-converter.com*******

******Created by ebook converter - www.ebook-converter.com******

GASTROSCHISIS:
QUESTION: The most common anomaly associated with gastroschisis is what?
.
.
.
.
.
ANSWER: Intestina atresia
- Gastroschisis, unlike omphalocele, is not typically associated with systemic or
chromosomal abnormalities.
- It is most commonly associated with intestinal atresia in as many as 15% of cases.
- For this reason, it is imperative that the small bowel be carefully explored at the time
of reduction and repair of the abdominal wall defect.
.
.
.
.
.
.
QUESTION: On what side is the abdominal wall defect in gastroschisis?
.
.
.
.
.
ANSWER: Right side.
- There is an abdominal wall defect to the right of the umbilicus, and the bowel
******ebook converter DEMO - www.ebook-converter.com*******

******Created by ebook converter - www.ebook-converter.com******


herniates through without a peritoneal covering.
- Because the bowel is eviscerated and exposed, this condition is a surgical emergency.
- The bowel can be thickened and covered with an exudate.
- If the defect cannot be closed, a silo can be used..

******ebook converter DEMO - www.ebook-converter.com*******

******Created by ebook converter - www.ebook-converter.com******

******ebook converter DEMO - www.ebook-converter.com*******

******Created by ebook converter - www.ebook-converter.com******

GLASGOW COMA SCALE: GCS:


QUESTION: A 25 year old man involved in a motor vehicle accident will open his eyes
only to painful stimuli, does not form words but mumbles, and withdraws to pain. This
patient's Glasgow Coma Score is what?
.
.
.
.
ANSWER:
- Open eyes to painful stimuli is 2
- Incomprehensible sounds is 2
- Withdraws from pain is 4
- 2+2+4 = 8
.
.
.
.
.
QUESTION: A 27 year old man presents to the ED after receiving blows to the head.
He opens his eyes with painful stimuli, is confused, and localizes to pain. What is his
Glasgow Coma Score?
.
.
.
.
.
.
ANSWER:
******ebook converter DEMO - www.ebook-converter.com*******

******Created by ebook converter - www.ebook-converter.com******


- His is is 2(eye) + 4(verbal) + 5(motor) = 11
.
.

******ebook converter DEMO - www.ebook-converter.com*******

******Created by ebook converter - www.ebook-converter.com******

******ebook converter DEMO - www.ebook-converter.com*******

******Created by ebook converter - www.ebook-converter.com******

GLEEVEC:
QUESTION: What type of leukemia is Gleevec used to treat?
.
.
.
.
.
ANSWER: - The primary function of anticancer chemicals is to block different steps
involved in cell growth and replication.
- These chemicals often block a critical chemical reaction in a signal transduction
pathway or during DNA replication or gene expression.
- For example, STI571, also known as Gleevec, is one of the first molecularly targeted
drugs based on the changes that cancer causes in cells.
- STI571 offers promise for the treatment of chronic myeloid leukemia (CML) and may
soon surpass interferon-y as the standard treatment for the disease.
- In CML, STI571 is targeted at the Bcr-Abl kinase, an activated oncogene product in
CML .
- Bcr-Abl is an overly activated protein kinase resulting from a specific genetic
abnormality generated by chromosomal translocation that is found in the cells of patients
with CmL.
- STI571-mediated inhibition of Bcr-Abl-kinase activity not only prevents cell growth
of Bcr-Abl-transformed leukemic cells, but also induces apoptosis.
- Clinically, the drug quickly corrects the blood cell abnormalities caused by the
leukemia in a majority of patients, achieving a complete disappearance of the leukemic
blood cells and the return of normal blood cells.

******ebook converter DEMO - www.ebook-converter.com*******

******Created by ebook converter - www.ebook-converter.com******

******ebook converter DEMO - www.ebook-converter.com*******

******Created by ebook converter - www.ebook-converter.com******

GLYCOLYSIS:
QUESTION: What is the net ATP made during anaerobic glycolysis of 1 glucose
molecule?
.
.
.
.
.
ANSWER:
- The net ATP made during anaerobic glycolysis is 2 ATPs and lactate.
.
.
.
.
.
.

QUESTION: What is the total ATP made from 1 glucose molecule in aerobic glycolysis
(including Kreb cycle, and oxidative phosphorylation)?
.
.
.
.
.
ANSWER:
- The amount of ATP formed from 1 glucose molecule by the Kreb cycle (plus glycolysis and oxidative
phosphorylation) is 38 ATP.

******ebook converter DEMO - www.ebook-converter.com*******

******Created by ebook converter - www.ebook-converter.com******

******ebook converter DEMO - www.ebook-converter.com*******

******Created by ebook converter - www.ebook-converter.com******

GLUCONEOGENESIS:
QUESTION: During gluconeogenesis, what is the intermediate molecule in the
conversion of lactate to glucose in the liver?
.
.
.
.
.
ANSWER:
- When lactate from muscle is converted to glucose in the liver via the Cori cycle during gluconeogenesis, the
intermediate is pyruvate. Lactate Pyruvate Glucose.

.
.
.
.
.
QUESTION: Which molecule can act as a substrate for either gluconeogenesis or the
Krebs cycle?
.
.
.
.
.
ANSWER:
- The molecule that is used in gluconeogenesis and also used in the Krebs cycle is pyruvate. (TRICKY)

.
.
.
******ebook converter DEMO - www.ebook-converter.com*******

******Created by ebook converter - www.ebook-converter.com******


.
.

******ebook converter DEMO - www.ebook-converter.com*******

******Created by ebook converter - www.ebook-converter.com******

QUESTION: What is the role of fatty acids in gluconeogenesis?


.
.
.
ANSWER:
- The role that fat molecules play in gluconeogenesis is that they contribute glycerol backbones.
.
.
.
.
.
QUESTION:. Gluconeogenesis normally occurs in the liver. However, where does it occur in late starvation?
.
.
.
.
.
ANSWER:
- In late starvation, gluconeogenesis occurs in the kidney. (TRICKY)

******ebook converter DEMO - www.ebook-converter.com*******

******Created by ebook converter - www.ebook-converter.com******

******ebook converter DEMO - www.ebook-converter.com*******

******Created by ebook converter - www.ebook-converter.com******

******ebook converter DEMO - www.ebook-converter.com*******

******Created by ebook converter - www.ebook-converter.com******

GYNECOMASTIA:
QUESTION: Which gastrointestinal condition leads to gynecomastia due to an
increased production of estrogen?
.
.
.
.
.
ANSWER: Hepatocellular carcinoma
GYNECOMASTIA DUE TO ESTROGEN EXCESS
- Estrogen excess results from an increase in the secretion of estradiol by the testicles or
by non-testicular tumors, nutritional alterations such as protein and fat deprivation,
endocrine disorders (hyperthyroidism, hypothyroidism), and hepatic disease
(nonalcoholic and alcoholic cirrhosis).
GYNECOMASTIA DUE TO TESTOSTERONE DEFICIENCY
- Klinefelter's syndrome, aging, and renal failure all cause gynecomastia by a decrease
in testosterone production.

******ebook converter DEMO - www.ebook-converter.com*******

******Created by ebook converter - www.ebook-converter.com******

******ebook converter DEMO - www.ebook-converter.com*******

******Created by ebook converter - www.ebook-converter.com******

HEMOGLOBIN OXYGEN BINDING CURVE:


QUESTION: Is stored blood low or high in 2,3-DPG?
.
.
.
ANSWER: Stored blood is low in 2,3-DPG
.
.
.
QUESTION: Does a low 2,3-DPG in stored blood lead to a left or a right shift?
.
.
.
ANSWER: Stored blood has low 2,3-DPG. This results in a left shift of the oxygen
binding curve of hemoglobin.
.
.
.
QUESTION: Does a left shift result in a higher or a lower affinity for oxygen?
.
.
.
ANSWER: A left shift means that the blood has a higher affinity for oxygen
.
.
.
QUESTION: Explain the physiology behind why stored blood has increased affinity for
oxygen
.
.
.
ANSWER: Banked blood is low in 2,3-DPG which leads to a left shift on the oxygen
binding curve of hemoglobin, resulting in increaseD hemoglobin affinity for oxygen

******ebook converter DEMO - www.ebook-converter.com*******

******Created by ebook converter - www.ebook-converter.com******

HEMOPHILIA:
QUESTION: What is the treatment of hemophilia A?
.
.
.
.
.
ANSWER:
- The treatment for hemophilia A is factor VIII or cryoprecipitate.

.
.
.
.
.
QUESTION: What is the treatment a duodenal hematoma in a patient with hemophilia
A?
.
.
.
.
.
ANSWER:
- The treatment of duodenal hematoma in a patient with hemophilia A is factor VIII and nonoperative
management.

.
.
.
******ebook converter DEMO - www.ebook-converter.com*******

******Created by ebook converter - www.ebook-converter.com******


.
.

******ebook converter DEMO - www.ebook-converter.com*******

******Created by ebook converter - www.ebook-converter.com******

QUESTION: What is the treatment for hemophilia B?


.
.
.
ANSWER:
- The treatment for hemophilia B is factor IX or FFP

.
.
.
.
.
QUESTION: How do you treat a hemophiliac joint effusion?
.
.
.
.
.
ANSWER: Do not aspirate. Treat with ice, range of motion therapy. Give actor VIII.

******ebook converter DEMO - www.ebook-converter.com*******

******Created by ebook converter - www.ebook-converter.com******

******ebook converter DEMO - www.ebook-converter.com*******

******Created by ebook converter - www.ebook-converter.com******

HEMORRHAGE:
QUESTION: In a patient with ongoing hemorrhage, the risk of death increases 1% every
______ minutes in the ER.
.
.
.
.
.
ANSWER: Every 3 minutes in the ER.

******ebook converter DEMO - www.ebook-converter.com*******

******Created by ebook converter - www.ebook-converter.com******

******ebook converter DEMO - www.ebook-converter.com*******

******Created by ebook converter - www.ebook-converter.com******

HEMORRHOIDS:
QUESTION: An internal hemorrhoid that prolapses past the dentate line with straining
is what degree hemorrhoid?
.
.
.
.
ANSWER:
- It is a first degree internal hemorrhoid.
- Internal hemorrhoids are located proximal to the dentate line and covered by insensate
anorectal mucosa.
- Internal hemorrhoids may prolapse or bleed, but rarely become painful unless they
develop thrombosis and necrosis (usually related to severe prolapse, incarceration,
and/or strangulation).
- Internal hemorrhoids are graded according to the extent of prolapse.
- First degree hemorrhoids bulge into the anal canal and may prolapse beyond the
dentate line on straining
- Second degree hemorrhoids prolapse through the anus but reduce spontaneously.
- Third degree hemorrhoids prolapse through the anal canal and require manual
reduction.
- Fourth degree hemorrhoids prolapse but cannot be reduced and are at risk for
strangulation.
.

******ebook converter DEMO - www.ebook-converter.com*******

******Created by ebook converter - www.ebook-converter.com******

******ebook converter DEMO - www.ebook-converter.com*******

******Created by ebook converter - www.ebook-converter.com******

HEMOSTASIS:
QUESTION: What is the first step in the process of hemostasis?
.
.
.
.
.
ANSWER:
- The first step in the process of hemostasis after vessel injury is: vasoconstriction.

******ebook converter DEMO - www.ebook-converter.com*******

******Created by ebook converter - www.ebook-converter.com******

******ebook converter DEMO - www.ebook-converter.com*******

******Created by ebook converter - www.ebook-converter.com******

HEPARIN:
QUESTION: True or false: Heparin enhances the effect of antithrombin on thrombinmediated conversion of fibrinogen to fibrin
.
.
.
.
.
ANSWER: The answer is TRUE.
- The mechanism of action of Heparin is that it activates ATIII. (TOPIC)
.
.
.
.
.

QUESTION: What factors are inactivated by heparin?


.
.
.
.
.
ANSWER:
- The factors that Heparin inactives are factors 9, 10, 11, 12.

.
.
.
.
.
******ebook converter DEMO - www.ebook-converter.com*******

******Created by ebook converter - www.ebook-converter.com******


QUESTION: The lab is used to monitor heparin?
.
.
.
.
.
ANSWER:
- The lab test used to measure Heparin is PTT.

TOPIC: COMPLIC PROTAMINE REVERSAL HEPARIN

QUESTION: What is the management of a patient on heparin who actively bleeds into
the retroperitoneum?
.
.
.
.
.
ANSWER:
- If a patient on heparin develops active bleeding into the retroperitoneum, the treatment is conservative
management and protamine infusion.
.
.
.
.
.
QUESTION: What are the side effects of heparin?
.
.
.

******ebook converter DEMO - www.ebook-converter.com*******

******Created by ebook converter - www.ebook-converter.com******


.
.
ANSWER:
- The side effects of heparin include: HIT, thrombocytopenia, arterial thrombosis, skin necrosis, and
osteoporosis.

******ebook converter DEMO - www.ebook-converter.com*******

******Created by ebook converter - www.ebook-converter.com******

******ebook converter DEMO - www.ebook-converter.com*******

******Created by ebook converter - www.ebook-converter.com******

HEPARIN INDUCED THROMBOCYTOPENIA: HIT:


QUESTION: The diagnosis of heparin induced thrombocytopenia is made by what %
fall in platelet count, what assay, what number platelet, etc?
.
.
.
.
.
.
- Heparin induced thrombocytopenia (HIT) is a form of drug induced immune
thrombocytopenia.
- It is an immunologic disorder in which antibodies against PF4 formed during exposure
to heparin affect platelet activation and endothelial function with resultant
thrombocytopenia and intravascular thrombosis.
- The platelet count typically begins to fall 5 to 7 days after heparin has been started, but
if it is a re-exposure, the decrease in count may occur within 1 to 2 days.
- HIT should be suspected if the platelet count falls to <100,000/uL or if it drops by
50% from baseline in a patient receiving heparin.
- Although HIT is more common with full-dose unfractionated heparin (1 to 3%), it also
can occur with prophylactic doses or with low molecular weight heparins.
- Interestingly, approximately 17% of patients receiving unfractionated heparin and 8%
of those receiving low molecular weight heparin develop antibodies against PF4, yet a
much smaller percentage develop thrombocytopenia and even fewer clinical HIT.
- In addition to the mild to moderate thrombocytopenia, this disorder is characterized by
a high incidence of thrombosis, which may be arterial or venous.
- Importantly, the absence of thrombocytopenia in these patients does not preclude the
diagnosis of HIT.
- The diagnosis of HIT may be made by using either a serotonin release assay or an
enzyme-linked immunosorbent assay (ELISA).
- The serotonin release assay is highly specific but not sensitive, so that a positive test
******ebook converter DEMO - www.ebook-converter.com*******

******Created by ebook converter - www.ebook-converter.com******


result supports the diagnosis but a negative result does not exclude HIT.
- On the other hand, the ELISA has a low specificity, so although a positive ELISA
result confirms the presence of anti-heparin -PF4, it does not help in the diagnosis of
clinical HIT.
- A negative ELISA result, however, essentially rules out HIT.

.
.

******ebook converter DEMO - www.ebook-converter.com*******

******Created by ebook converter - www.ebook-converter.com******

QUESTION: In addition to stopping the heparin, a patient with heparin induced


thrombocytopenia (HIT) should be treated with what class of drugs?
.
.
- The initial treatment of suspected HIT is to stop heparin and begin an alternative
anticoagulant.
- Stopping heparin without adding another anticoagulant is not adequate to prevent
thrombosis in this setting.
- Alternative anticoagulants are primarily thrombin inhibitors.
- Those available in the United States are lepirudin, argatroban, and bivalirudin.
- In Canada and Europe, danaparoid also is available.
- Danaparoid is a heparinoid that has approximately 20% cross reactivity with HIT
antibodies in vitro but a much lower cross reactivity in vivo.
- Because of warfarin's early induction of a hypercoagulable state, only once full
anticoagulation with an alternative agent has been accomplished and the platelet count
has begun to recover should warfarin be instituted.

******ebook converter DEMO - www.ebook-converter.com*******

******Created by ebook converter - www.ebook-converter.com******

******ebook converter DEMO - www.ebook-converter.com*******

******Created by ebook converter - www.ebook-converter.com******

HEPATIC ADENOMA:
QUESTION: A 40 year old woman on no medications and without any past medical
problems presents with intermittent abdominal discomfort. She undergoes a contrasted
abdominal CT scan which shows a 4 cm hypervascular liver lesion. Sulfur colloid scan
shows no uptake. AFP is normal. There is no peripheral to central enhancement on the
contrasted CT scan. You diagnose her with a hepatic adenoma. How do you treat a
hepatic adenoma?
.
.
.
.
.
ANSWER:
- A liver mass that is hypervascular, is associated with a normal AFP (likely not
hepatocellular carcinoma), has no peripheral to central enhancement (likely not a
hemangioma), and no uptake on sulfur colloid scan (likely not focal nodular
hyperplasia) is most consistent with adenoma.
- Because she is not on any steroids, (i.e. oral contraceptive pills), the treatment for
the lesion is resection because of the significant risk of rupture and malignant
transformation.
.
.
.

******ebook converter DEMO - www.ebook-converter.com*******

******Created by ebook converter - www.ebook-converter.com******

******ebook converter DEMO - www.ebook-converter.com*******

******Created by ebook converter - www.ebook-converter.com******

HERNIAS:
QUESTION: What is the most common complication following inguinal hernia repair?
.
.
.
.
.
ANSWER:
- The most common complication following an inguinal hernia repair is urinary
retention.
- Risk factors include males, increased narcotic requirement and age
.

******ebook converter DEMO - www.ebook-converter.com*******

******Created by ebook converter - www.ebook-converter.com******

******ebook converter DEMO - www.ebook-converter.com*******

******Created by ebook converter - www.ebook-converter.com******

HIATAL HERNIA:
QUESTION: What are the indications for surgical repair of a hiatal hernia in a
symptomatic patient?
.
.
.
.
.
ANSWER: The indications for hiatal hernia repair include:
- Patient desires not to take pills to control reflux for a prolonged period
- Regurgitation and aspiration not controlled with medical therapy
- Persistent symptoms despite medical therapy
Barrett's is not an indication for hiatal hernia repair although the patient should be
placed on a proton pump inhibitor and have their esophagus evaluated for dysplasia and
cancer on a regular basis.

******ebook converter DEMO - www.ebook-converter.com*******

******Created by ebook converter - www.ebook-converter.com******

******ebook converter DEMO - www.ebook-converter.com*******

******Created by ebook converter - www.ebook-converter.com******

HISTAMINE:
QUESTION: What is the molecular mechanism of action of how histamine causes HCl
production?
.
.
.
ANSWER: Histamine acts on parietal cells via cAMP to increase HCL production
mnemonic: H for Happy cAMPers)

******ebook converter DEMO - www.ebook-converter.com*******

******Created by ebook converter - www.ebook-converter.com******

******ebook converter DEMO - www.ebook-converter.com*******

******Created by ebook converter - www.ebook-converter.com******

HYPERCOAGULABLE STATES:
QUESTION: What is the most common risk factor for spontaneous venous
thromboembolism?
.
.
.
.
.
ANSWER:
- The most common risk factor for spontaneous venous thromboembolism is factor V leiden.
.
.
.
.
.

QUESTION: What conditions lead to both arterial and venous clots?


.
.
.
.
.
ANSWER:
- Lupus anticoagulant (and hyperhomocysteinemia) are both associated with arterial and venous clots.
.
.
.
.
.

******ebook converter DEMO - www.ebook-converter.com*******

******Created by ebook converter - www.ebook-converter.com******

******ebook converter DEMO - www.ebook-converter.com*******

******Created by ebook converter - www.ebook-converter.com******

QUESTION: What deficiencies lead to hypercoagulable states and subsequent risk of


venous thromboembolisms?
.
.
.
.
.
ANSWER:
- The deficiencies that lead to venous thromboembolism are:
- Protein C or Protein S deficiency
- anti-thrombin III deficiency.
- Factor V Leiden (most common congenital hypercoagulability disorder)
- hyperhomocysteinemia (venous and arterial clots)
- prothrombin gene defect G20210A
- polycythemia vera
- lupus anticoagulant
- cardiopulmonary bypass (leads to increase in Hageman factor)
- acquired hypercoagulability (eg. tobacco), and others.

******ebook converter DEMO - www.ebook-converter.com*******

******Created by ebook converter - www.ebook-converter.com******

******ebook converter DEMO - www.ebook-converter.com*******

******Created by ebook converter - www.ebook-converter.com******

HYPERHOMOCYSTEINEMIA:
QUESTION: What is the treatment of hyperhomocysteinemia?
.
.
.
ANSWER:
- The deficiency associated with DVTs and arterial thrombosis (i.e. clots in arteries and veins) is
hyperhomocysteinemia (and lupus anticoagulant).
- Treatment of hyperhomocysteinemia is folic acid & B12.

******ebook converter DEMO - www.ebook-converter.com*******

******Created by ebook converter - www.ebook-converter.com******

******ebook converter DEMO - www.ebook-converter.com*******

******Created by ebook converter - www.ebook-converter.com******

HYPERKALEMIA:
QUESTION: What is the initial treatment of choice in a patient with hyperkalemia and
arrhythmias?
.
.
.
.
.
ANSWER: A patient that is having arrhythmias as a result of the hyperkalemia should
receive Ca-gluconate 1st to stabilize cardiac muscle cell membranes.

******ebook converter DEMO - www.ebook-converter.com*******

******Created by ebook converter - www.ebook-converter.com******

QUESTION: You bring an intubated 30 year old man with a 60% total body surface
burn back to the OR for his 3rd debridement in 3 days and the anesthesiologist gives
him succinylcholine instead of the pancuronium which he was receiving in the ICU.
Shortly after this, his EKG shows T wave abnormalities, then a widened QRS, and then
ventricular fibrillation. You cannot feel a pulse so you start CPR and shock him 3 times
without success. The 1st drug you should give for this problem is what?.
.
.
.
ANSWER:
- Burn patients are at increased risk for hyperkalemia because of myonecrosis and
leaking of potassium into the bloodstream.
- This, combined with succinylcholine which causes potassium release when it
depolarizes the cell membrane, results in hyperkalemia.
- The 1st treatment of choice when hyperkalemia occurs with arrhythmias is calcium
gluconate.
- This stabilizes the myocardial cell membrane and will help stabilize the arrhythmia.
- Dextrose 50% and 10 units of insulin, HCO3-, Kayexalate, dialysis, etc can be used to
help treat hyperkalemia.

******ebook converter DEMO - www.ebook-converter.com*******

******Created by ebook converter - www.ebook-converter.com******

QUESTION: What is the first treatment administered to a patient with a potassium level
of 6.3 and flattened P waves on their ECG?
.
.
.
.
.
.
ANSWER:
- The goals of therapy include reducing the total body potassium, shifting potassium
from the extracellular to the intracellular space, and protecting the cells from the effects
of increased potassium.
- For all patients exogenous sources of potassium should be removed, including
potassium supplementation in IV fluids and enteral and parenteral solutions.
- Potassium can be removed from the body using a cation-exchange resin such as
Kayexalate that binds potassium in exchange for sodium.
- It can be administered either orally, in alert patients, or rectally.
- Immediate measures also should include attempts to shift potassium intracellularly
with glucose bicarbonate infusion.
- Nebulized albuterol (10 to 20 mg) may also be used.
- Use of glucose alone will cause a rise in insulin secretion, but in the acutely ill this
response may be blunted, and therefore both glucose and insulin may be necessary.
- Circulatory overload and hypernatremia may result from the administration of
Kayexalate and bicarbonate, so care should be exercised when administering these
agents to patients with fragile cardiac function.
- When ECG changes are present, calcium chloride or calcium gluconate (5 to 10 mL of
10% solution) should be administered immediately to counteract the myocardial effects
of hyperkalemia.
******ebook converter DEMO - www.ebook-converter.com*******

******Created by ebook converter - www.ebook-converter.com******


- Calcium infusion should be used cautiously in patients receiving digitalis, because
digitalis toxicity may be precipitated.
- All of the aforementioned measures are temporary, lasting from 1 to approximately 4
hours.
- Dialysis should be considered in severe hyperkalemia when conservative measures
fail.

******ebook converter DEMO - www.ebook-converter.com*******

******Created by ebook converter - www.ebook-converter.com******

TREATMENT OF SYMPTOMATIC HYPERKALEMIA


Potassium removal
- Kayexalate
- oral administration is 15-30g in 50-100ml of 20% sorbitol
- Rectal administration is 50g in 200ml of 20% sorbitol
- Dialysis
Shift potassium
- Glucose 1 ampule of D50 and regular insulin 5-10 units IV
- Bicarbonate 1 ampule IV
Counteract cardiac effects
- Calcium gluconate 5-10ml of 10% solution
.

******ebook converter DEMO - www.ebook-converter.com*******

******Created by ebook converter - www.ebook-converter.com******

******ebook converter DEMO - www.ebook-converter.com*******

******Created by ebook converter - www.ebook-converter.com******

HYPERPARATHYROIDISM:
QUESTION: What are the typical lab results seen in primary hyperparathyroidism in
terms of Ca, PTH, and urine Ca levels?
.
.
.
.
ANSWER:
- Primary hyperparathyroidism is associated with:
- Autonomously elevated PTH (nl 10-60 pg/ml)
- elevated serum Ca (nl 8.5 - 10.5)
- Elevated (70%) or normal (30%) urine Ca
.
.
.
.
.
.
.

******ebook converter DEMO - www.ebook-converter.com*******

******Created by ebook converter - www.ebook-converter.com******

QUESTION: What are the indications for operative treatment of asymptomatic patients
with primary hyperparathyroidism?
.
.
.
.
.
ANSWER:
- Current National Institutes of Health conference guidelines for surgery in
asymptomatic patients include at initial evaluation:
(1) a serum calcium level more than 1 mg/dL above the upper limit of reference
value,
(2) reduced creatinine clearance of more than 30% compared with matched
controls,
(3) an increased urinary calcium excretion of more than 400 mg/day,
(3a) - Presence of kidney stones (nephrolithiasis) detected by abdominal imaging
(4) evidence of bone mass reduction more than 2.5 standard deviations below
matched controls, and
(5) unwillingness or inability to undergo continued follow-up.

******ebook converter DEMO - www.ebook-converter.com*******

******Created by ebook converter - www.ebook-converter.com******

******ebook converter DEMO - www.ebook-converter.com*******

******Created by ebook converter - www.ebook-converter.com******

HYPOCALCEMIA:
QUESTION: A 23 year old woman undergoes total thyroidectomy for carcinoma of the
thyroid gland. On the second postoperative day, she begins to complain of a tingling
sensation in her hands. She appears quite anxious and later complains of muscle cramps.
What is the most appropriate initial management strategy?
.
.
.
.
.
ANSWER:
- Intravenous calcium infusion is the treatment for severe, symptomatic hypocalcemia,
although, typically, oral calcium supplementation (up to 1 to 2 g every 4 hours) is
sufficient in patients with mild symptoms.
- Since post-thyroidectomy hypocalcemia is usually due to transient ischemia of the
parathyroid glands and is self-limited, in most cases the problem is resolved in several
days.
- In cases of persistent hypocalcemia, Vitamin D preparations may be necessary.
- There is no role for thyroid hormone replacement or magnesium sulfate in the
treatment of hypocalcemia.
.
.
.
QUESTION: What is the actual serum calcium level in a patient with an albumin of 2.0
and a serum calcium level of 6.6?
.
.
.
******ebook converter DEMO - www.ebook-converter.com*******

******Created by ebook converter - www.ebook-converter.com******


ANSWER: The answer is C.
- When total serum calcium levels are measured, the albumin concentration must be
taken into consideration: Adjust total serum calcium down by 0.8mg/dL for every 1g/dL
decrease in albumin.
- 0.8 X 2 = 1.6 + 6.6 = 8.2
.

******ebook converter DEMO - www.ebook-converter.com*******

******Created by ebook converter - www.ebook-converter.com******

HYPOKALEMIA:
QUESTION: A postoperative patient with a potassium of 2.9 is given 1 mEq/kg
replacement with KCL (potassium chloride). Repeat tests after the replacement show
the serum K to be 3.0. The most likely other electrolyte abnormality to cause this is
what?
.
.
.
.
.
ANSWER:
- The most likely other electrolyte abnormality in this case is hypomagnesemia.
- In cases in which potassium deficiency is due to magnesium depletion, potassium
repletion is difficult unless hypomagnesemia is first corrected.

******ebook converter DEMO - www.ebook-converter.com*******

******Created by ebook converter - www.ebook-converter.com******

******ebook converter DEMO - www.ebook-converter.com*******

******Created by ebook converter - www.ebook-converter.com******

INCIDENCE:
QUESTION: Define incidence
.
.
.
ANSWER: Incidence is the number of new cases diagnosed over a certain time frame in
a population. This time frame is usually per year. For example, the number of patients in
the US with newly diagnosed breast cancer in 2010.

******ebook converter DEMO - www.ebook-converter.com*******

******Created by ebook converter - www.ebook-converter.com******

******ebook converter DEMO - www.ebook-converter.com*******

******Created by ebook converter - www.ebook-converter.com******

INHALED ANESTHETICS:
QUESTION: Is the main side effect of Halothane hepatotoxicity or renal toxicity?
.
.
.
ANSWER: Halothane is hepatotoxic
.
.
.
QUESTION: What kind of toxicity is associated with Methoxyflurane?
.
.
.
ANSWER: Methoxyflurane is associated with renal toxicity

******ebook converter DEMO - www.ebook-converter.com*******

******Created by ebook converter - www.ebook-converter.com******

******ebook converter DEMO - www.ebook-converter.com*******

******Created by ebook converter - www.ebook-converter.com******

INR:
QUESTION: True or false: An INR of more than 1.5 is considered safe for operation
.
.
.
.
.
ANSWER: The answer is TRUE.
- With meticulous hemostatic technique, many operations can be performed on patients
with an INR greater than 1.5.
- Exceptions include operations on the eye or the prostate, neurosurgical procedures, or
a blind needle aspiration.
- In these cases, an INR of less than 1.2 is required.
- Patients who are receiving anticoagulant treatment with warfarin and who require
emergency surgery may be given plasma to immediately reverse the warfarin effect.
- Alternatively, vitamin K may be given orally or subcutaneously at least 6 hr
preoperatively to reverse the effect of warfarin on vitamin K dependent factors.
- The INR should be obtained again before surgery and, if it is not below 1.5, plasma
should be administered.

******ebook converter DEMO - www.ebook-converter.com*******

******Created by ebook converter - www.ebook-converter.com******

******ebook converter DEMO - www.ebook-converter.com*******

******Created by ebook converter - www.ebook-converter.com******

INTESTINAL ATRESIA:
PATHOPHYSIOLOGY
QUESTION: True or false: Jejunal atresia is due to a fetal mesenteric vascular
accident.
.
.
.
.
.
ANSWER: TRUE.
- Obstruction caused by intestinal atresia can occur at any point along the intestinal
tract.
- Most cases are believed to be caused by in utero mesenteric vascular accidents
leading to segmental loss of the intestinal lumen.
- They are classified into four types based on the severity.

******ebook converter DEMO - www.ebook-converter.com*******

******Created by ebook converter - www.ebook-converter.com******

PRESENTATION
QUESTION: Most infants with jejunal atresia present with nonbilious vomiting.
.
.
.
.
.
ANSWER: FALSE.
- Infants with jejunal or ileal atresia present soon after birth with bilious vomiting and
progressive abdominal distention.
- The more distal the obstruction, the more distended the abdomen and the greater the
number of loops on upright abdominal radiograph.
.
.
.
.
.
QUESTION: Jejunal atresia is the most common cause of surgical vomiting in infancy.
.
.
.
.
.
ANSWER: FALSE.

******ebook converter DEMO - www.ebook-converter.com*******

******Created by ebook converter - www.ebook-converter.com******

******ebook converter DEMO - www.ebook-converter.com*******

******Created by ebook converter - www.ebook-converter.com******

WORKUP AND TREATMENT


QUESTION: True or false: If the infant with jejunal atresia was born prematurely,
surgery should be delayed until the infant weighs more than 8 lb.
.
.
.
.
.
ANSWER: FALSE.
.
.
.
.
.
QUESTION: True or false: A barium enema is contraindicated in the workup of jejunal
atresia.
.
.
.
.
.
ANSWER: FALSE.
- In cases in which the diagnosis of complete intestinal obstruction is ascertained by the
clinical picture and the presence of staggered airfluid levels on plain abdominal films,
the child can be brought to the operating room after appropriate resuscitation.
******ebook converter DEMO - www.ebook-converter.com*******

******Created by ebook converter - www.ebook-converter.com******


- In these circumstances, there is little extra information that can be gained by a barium
enema.
- When the diagnosis is uncertain, a barium enema may be used.

******ebook converter DEMO - www.ebook-converter.com*******

******Created by ebook converter - www.ebook-converter.com******

TREATMENT
QUESTION: What is the management of jejunal atresia?
.
.
.
.
.
ANSWER:
- The initial treatment of jejunal atresia is nasogastric tube decompression and fluid
resuscitation.
- Definitive treatment involves surgical resection of the atretic loop and primary reanastomosis.

******ebook converter DEMO - www.ebook-converter.com*******

******Created by ebook converter - www.ebook-converter.com******

******ebook converter DEMO - www.ebook-converter.com*******

******Created by ebook converter - www.ebook-converter.com******

INTRA-AORTIC BALLOON PUMP: IABP:


QUESTION: What are the potential indications for the use of an intra-aortic balloon
pump?
.
.
.
.
.
ANSWER: include all of the following EXCEPT:
- The most frequent indication for the intra-aortic balloon pump is:
- to provide hemodynamic support during or after cardiac catheterization,
- in cardiogenic shock,
- for weaning from cardiac bypass,
- for preoperative use in high-risk patients, and
- for those with refractory unstable angina.

******ebook converter DEMO - www.ebook-converter.com*******

******Created by ebook converter - www.ebook-converter.com******

******ebook converter DEMO - www.ebook-converter.com*******

******Created by ebook converter - www.ebook-converter.com******

INTRINSIC PATHWAY:
QUESTION: Summarize the intrinsic pathway: include the lab test associated with it,
the factors involved, and the final product
.
.
.
ANSWER:
- Intrinsic path is tested with PTT.
- Exposed collagen + XII --> XI, IX --> X which activates thrombin to produce fibrin

******ebook converter DEMO - www.ebook-converter.com*******

******Created by ebook converter - www.ebook-converter.com******

******ebook converter DEMO - www.ebook-converter.com*******

******Created by ebook converter - www.ebook-converter.com******

INTUSSUSCEPTION:
QUESTION: What is the most common pathologic lead point for intussusception in
children?
.
.
.
.
.
ANSWER: Meckel diverticulum
- Infants 6 to 24 months of age with intussusception typically have no pathologic lead
point on pathologic examination, but instead have hypertrophied Peyer patches.
- Conversely, older children have a much higher likelihood of having a pathologic lead
point.
- As such, they have a much greater need for operative intervention to resect the segment
of bowel that includes the pathologic area.
- The most common pathologic lead point for intussusception in children is a Meckel
diverticulum.
- Other causes include polyps, appendicitis, intestinal neoplasms such as
lymphosarcoma, submucosal hemorrhage, foreign body, ectopic pancreatic or gastric
tissue, and intestinal duplication.

******ebook converter DEMO - www.ebook-converter.com*******

******Created by ebook converter - www.ebook-converter.com******

******ebook converter DEMO - www.ebook-converter.com*******

******Created by ebook converter - www.ebook-converter.com******

LACTIFEROUS DUCTS:
QUESTION: How many lactiferous ducts drain into the nipple of the mature female
breast?
.
.
.
.
.
ANSWER: 15-20
- The breast is composed of 15 to 20 lobes, which are each composed of several
lobules.
- Each lobe of the breast terminates in a major (lactiferous) duct (2 to 4 mm in
diameter), which opens through a constricted orifice (0.4 to 0.7mm in diameter) into the
ampulla of the nipple.

******ebook converter DEMO - www.ebook-converter.com*******

******Created by ebook converter - www.ebook-converter.com******

******ebook converter DEMO - www.ebook-converter.com*******

******Created by ebook converter - www.ebook-converter.com******

LAPAROSCOPY:
QUESTION: Compared to open procedures, do laparoscopic procedures result in
higher or lower serum cortisol levels?
.
.
.
.
.
ANSWER: Laparoscopic procedures result in higher serum cortisol levels.
- Endocrine responses to laparoscopic surgery are not always intuitive.
- Serum cortisol levels after laparoscopic operations are often higher than after the
equivalent operation performed through an open incision.

******ebook converter DEMO - www.ebook-converter.com*******

******Created by ebook converter - www.ebook-converter.com******

******ebook converter DEMO - www.ebook-converter.com*******

******Created by ebook converter - www.ebook-converter.com******

LIVER ABSCESS: PYOGENIC:


QUESTION: The most common cause of hepatic abscess in the US is what?
.
.
.
.
.
- Hepatic abscesses are rare, currently accounting for approximately 15 per 100,00
hospital admissions in the US.
- Pyogenic abscesses account for approximately 80% of cases, the remaining 20% being
equally divided among parasitic and fungal forms.
- Formerly, pyogenic liver abscesses were caused by pylephlebitis due to neglected
appendicitis or diverticulitis.
- Today, manipulation of the biliary tract to treat a variety of diseases has become a
more common cause, although in nearly 50% of patients no cause is identified.
.
.
.
.
.
QUESTION: A pyogenic abscess is best treated with:
.
.
.
.
******ebook converter DEMO - www.ebook-converter.com*******

******Created by ebook converter - www.ebook-converter.com******


.
A:
- A pyogenic abscess is best treated with broad spectrum antibiotics and
percutaneous drainage.
- GNR's are the most common organism in these cysts.
- You should also cover for anaerobes.
- Liver abscesses can arise weeks after an episode of diverticulitis or appendicitis or
other intra-abdominal infectious process.

******ebook converter DEMO - www.ebook-converter.com*******

******Created by ebook converter - www.ebook-converter.com******

LIVER CANCER: HEPATOCELLULAR CARCINOMA:


QUESTION: What are the criteria for liver transplantation in the setting of
hepatocellular carcinoma?
.
.
.
.
.
ANSWER:
- In a landmark 1996 study by Mazzaferro and colleagues at the University of Milan,
characteristics of patients with HCC that were good candidates for OLT were
described.
- These characteristics, now commonly referred to as the Milan criteria, included (a) a
single lesion <5 cm or 1 to 3 tumors, each <3 cm; and (b) absence of vascular or
lymphatic invasion.
- Patients meeting these criteria achieved an impressive 85% 4-year overall patient
survival, while those patients that exceeded the Milan criteria had only 50% 4 year
survival.
.
.

******ebook converter DEMO - www.ebook-converter.com*******

******Created by ebook converter - www.ebook-converter.com******

******ebook converter DEMO - www.ebook-converter.com*******

******Created by ebook converter - www.ebook-converter.com******

LUNG ABSCESS:
QUESTION: What is the most common cause of lung abscess?
.
.
.
.
.
ANSWER: Aspiration
- A lung abscess is usually the result of aspiration that results in a suppurative bacterial
infection, leading to localized pulmonary parenchymal necrosis.
- Patients with a history of alcohol abuse, those with poor dentition and gum disease,
and patients with seizure disorders are at greatest risk.

******ebook converter DEMO - www.ebook-converter.com*******

******Created by ebook converter - www.ebook-converter.com******

******ebook converter DEMO - www.ebook-converter.com*******

******Created by ebook converter - www.ebook-converter.com******

LUNG CANCER:
QUESTION: What is the most common type of lung cancer in a nonsmoker?
.
.
.
.
.
ANSWER: Adenocarcinoma
- Adenocarcinoma is the most common lung cancer in nonsmokers.

******ebook converter DEMO - www.ebook-converter.com*******

******Created by ebook converter - www.ebook-converter.com******

******ebook converter DEMO - www.ebook-converter.com*******

******Created by ebook converter - www.ebook-converter.com******

LUPUS ANTICOAGULANT:
QUESTION: What is the laboratory diagnosis of lupus anticoagulant?
.
.
.
.
.
ANSWER:
- The laboratory diagnosis of lupus anticoagulant is long Russel viper venom time.

.
.
.
.
.
QUESTION: What conditions are associated with both arterial and venous clots?
.
.
.
.
.
ANSWER:
- Lupus anticoagulant (and hyperhomocysteinemia) are both associated with arterial and venous clots.

******ebook converter DEMO - www.ebook-converter.com*******

******Created by ebook converter - www.ebook-converter.com******

******ebook converter DEMO - www.ebook-converter.com*******

******Created by ebook converter - www.ebook-converter.com******

MAINTENANCE FLUIDS:
QUESTION: A 30-kg child has an estimated daily fluid requirement of what ml/hr?
.
.
.
.
.
ANSWER: 70 ml/hr
- Daily maintenance fluids for children can be estimated using the 4-2-1 rule
4 mL/kg/hr for the first 10 kg,
2 mL/kg for the second 10 kg, and
1 mL/kg for any additional kilograms).
- For this child who weighs 30 kg, maintenance fluids calculate to be
4 mL/kg x 10 kg = 40 mL,
2 mL/kg x 10 kg = 20 mL,
1 mL/kg x 10 kg = 10 mL;
40 mL + 20 mL + 10 mL = 70 mL/hr.

******ebook converter DEMO - www.ebook-converter.com*******

******Created by ebook converter - www.ebook-converter.com******

******ebook converter DEMO - www.ebook-converter.com*******

******Created by ebook converter - www.ebook-converter.com******

MALIGNANT HYPERTHERMIA:
QUESTION: What are the symptoms of malignant hyperthermia?
.
.
.
ANSWER: The symptoms of malignant hyperthermia include: fever, tachycardia,
rigidity
.
.
.
QUESTION: If you suspect a patient has malignant hyperthermia, do you continue the
operation or stop it?
.
.
.
ANSWER: Part of the treatment of malignant hyperthermia is stopping the operation
.
.
.
.
.
.
QUESTION: What is the inheritance pattern of malignant hyperthermia?
.
.
.
.
.
ANSWER: Autosomal dominant.
******ebook converter DEMO - www.ebook-converter.com*******

******Created by ebook converter - www.ebook-converter.com******


- Malignant hyperthermia is a rare autosomal dominant disorder of skeletal muscle.
.
.
.
QUESTION:: What drugs can cause malignant hyperthermia?
.
.
.
.
.
ANSWER: inhalation anesthetics, succinylcholine
- The condition is characterized by a hypermetabolic state triggered by exposure to
certain inhalation anesthetics or succinylcholine.
- The older anesthetic agents associated with this reaction include halothane and
enflurane.
.
.
.
.
.
QUESTION: What is the pathophysiology of malignant hyperthermia?
.
.
.
.
.
ANSWER: Intracellular calcium accumulation.
******ebook converter DEMO - www.ebook-converter.com*******

******Created by ebook converter - www.ebook-converter.com******


- Malignant hyperthermia occurs when uncontrolled amounts of intracellular calcium
accumulate in skeletal muscle.
.
.
.
.
.

******ebook converter DEMO - www.ebook-converter.com*******

******Created by ebook converter - www.ebook-converter.com******

QUESTION: What are the symptoms of malignant hyperthermia?


.
.
.
.
ANSWER: muscle rigidity, increased end tidal CO2, tachycardia, increased temp
- Symptoms may develop as early as 30 minutes after anesthetic administration and as
late as 24 hours postoperatively.
- The initial clues occur in the operating room after induction.
- Rather than achieve complete paralysis, the anesthesiologist may notice rigidity in the
masseter muscle.
- Other findings include an increase in end tidal CO2, tachycardia, and an increase in
temperature.
.
.
.
.
.
QUESTION: What is the management of malignant hyperthermia?
.
.
.
.
.
ANSWER: stop anesthetics, dantrolene
- It is imperative that all anesthetics are immediately stopped and dantrolene given (2.5
******ebook converter DEMO - www.ebook-converter.com*******

******Created by ebook converter - www.ebook-converter.com******


mg/kg every 5 minutes) until resolution of symptoms.
- Dantrolene stabilizes muscle channels in the sarcoplasmic reticulum.
- The mortality rate approaches 30%.
.
.
.
.
.

******ebook converter DEMO - www.ebook-converter.com*******

******Created by ebook converter - www.ebook-converter.com******

QUESTION: What is the definitive diagnosis of malignant hyperthermia?


.
.
.
.
.
ANSWER: Biopsy.
- A functional test on skeletal muscle biopsy, the in vitro contracture test, is used for
diagnosis.
.
.
.
.
.
QUESTION: What is the genetic disorder associated with malignant hyperthermia?
.
.
.
.
.
ANSWER: Ryanodine receptor
- More than 50% of the families show linkage of the in vitro contracture test phenotype
to the gene encoding the skeletal muscle ryanodine receptor.
- The test requires a muscle biopsy with exposure of the muscle to halothane and
caffeine.
- A positive test will cause significant muscle contraction.
******ebook converter DEMO - www.ebook-converter.com*******

******Created by ebook converter - www.ebook-converter.com******


- The majority of cases occur in children or young adults.

******ebook converter DEMO - www.ebook-converter.com*******

******Created by ebook converter - www.ebook-converter.com******

******ebook converter DEMO - www.ebook-converter.com*******

******Created by ebook converter - www.ebook-converter.com******

MAMMOGRAPHY:
QUESTION: Routine mammography in women over 50 years of age decreases mortality
from breast cancer by approximately how many percentage points?
.
.
.
.
.
.
ANSWER: 33%
- Routine use of screening mammography in women >=50 years of age reduces mortality
from breast cancer by 33%.
- This reduction comes without substantial risks and at an acceptable economic cost.

******ebook converter DEMO - www.ebook-converter.com*******

******Created by ebook converter - www.ebook-converter.com******

******ebook converter DEMO - www.ebook-converter.com*******

******Created by ebook converter - www.ebook-converter.com******

MASTITIS:
QUESTION: What is the treatment of choice for Zuska's disease (recurrent periductal
mastitis)?
.
.
.
.
ANSWER: Antibiotics, incision, and drainage
- Zuska's disease, also called recurrent periductal mastitis, is a condition of recurrent
retroareolar infections and abscesses.
- This syndrome is managed symptomatically, by antibiotics coupled with incision and
drainage as necessary.

******ebook converter DEMO - www.ebook-converter.com*******

******Created by ebook converter - www.ebook-converter.com******

******ebook converter DEMO - www.ebook-converter.com*******

******Created by ebook converter - www.ebook-converter.com******

MEAN ARTERIAL PRESSURE: MAP:


QUESTION: In direct measurement of blood pressure, what effect, if any, will
overdamping have on the mean arterial pressure (MAP)?
.
.
.
.
.
- There is no effect on the MAP reading
- It is important to note that even in an under-damped or overdamped system, mean
pressure will be accurately recorded, provided the system has been properly calibrated.
- For these reasons, when using direct measurement of intra-arterial pressure to monitor
patients, clinicians should make clinical decisions based on the measured mean arterial
blood pressure.

******ebook converter DEMO - www.ebook-converter.com*******

******Created by ebook converter - www.ebook-converter.com******

******ebook converter DEMO - www.ebook-converter.com*******

******Created by ebook converter - www.ebook-converter.com******

MECONIUM ILEUS:
QUESTION: What is a common condition associated with meconium ileus?
.
.
.
.
.
ANSWER: Cystic fibrosis
- Meconium ileus is a result of cystic fibrosis, in which the meconium becomes thick
and viscous due to deficits in pancreatic enzymes.
.
.
.
.
.
QUESTION: What is the presentation of an infant with meconium ileus?
.
.
.
.
.
ANSWER: Small bowel obstruction, bilious emesis
- It creates a small bowel obstruction, and as such, the infant may present with bilious
vomiting.
- In the most severe forms, it can lead to intestinal perforation.
.
.
******ebook converter DEMO - www.ebook-converter.com*******

******Created by ebook converter - www.ebook-converter.com******


.
.
.

******ebook converter DEMO - www.ebook-converter.com*******

******Created by ebook converter - www.ebook-converter.com******

QUESTION: What radiographic findings are associated with meconium ileus?


.
.
.
.
.
ANSWER:
- The radiograph typically demonstrates a "ground glass" appearance, which represents
small pockets of gas trapped inside the thickened meconium.
.
.
.
.
.
.
QUESTION: What is the management of uncomplicated meconium ileus?
.
.
.
.
.
ANSWER: water-soluble contrast enemas.
- The treatment strategy depends on whether the patient has complicated or
uncomplicated meconium ileus.
- Patients with uncomplicated meconium ileus can be treated nonoperatively.
- Administering a water-soluble enema such as dilute gastrograffin per rectum allows
******ebook converter DEMO - www.ebook-converter.com*******

******Created by ebook converter - www.ebook-converter.com******


the meconium to soften as it takes on more water.
- Optimally, the contrast should be inserted in the dilated portion of the ileum under
fluoroscopic control.
- The enema may be repeated every 12 hours over several days as needed.
.
.
.
.
.
.
QUESTION: What is surgical indication for meconium enema?
.
.
.
.
.
ANSWER:
- Surgery is required if nonoperative management fails or if the patient already has
evidence of perforation.
- Complicated cases are usually amenable to bowel resection and primary anastomosis
provided there is no evidence of giant cystic meconium peritonitis.
.

******ebook converter DEMO - www.ebook-converter.com*******

******Created by ebook converter - www.ebook-converter.com******

******ebook converter DEMO - www.ebook-converter.com*******

******Created by ebook converter - www.ebook-converter.com******

MELANOMA:
QUESTION: A patient presents with a biopsy proven melanoma of the thigh which is
3mm thick on histologic examination. At the time of excision, how wide should the
margins be?
.
.
.
.
.
.
ANSWER:
- Regardless of tumor depth or extension, surgical excision is the management of choice.
- Lesions 1mm or less in thickness can be treated with a 1cm margin.
- For lesions 1 mm to 4mm thick, a 2 cm margin is recommended.
- Lesions of greater than 4mm may be treated with 3 cm margins.
- The surrounding tissue should be removed down to the fascia to remove all lymphatic
channels.
- If the deep fascia is not involved by the tumor, removing it does not affect recurrence
or survival rates, so the fascia is left intact.

******ebook converter DEMO - www.ebook-converter.com*******

******Created by ebook converter - www.ebook-converter.com******

******ebook converter DEMO - www.ebook-converter.com*******

******Created by ebook converter - www.ebook-converter.com******

META-ANALYSIS:
QUESTION: Does a meta-analysis use one study or several different studies?
.
.
.
ANSWER: A meta-analysis uses different studies
.
.
.
QUESTION: Does a meta-analysis combine or separate the data from different studies?
.
.
.
ANSWER: A meta-analysis combines data from different studies.

******ebook converter DEMO - www.ebook-converter.com*******

******Created by ebook converter - www.ebook-converter.com******

******ebook converter DEMO - www.ebook-converter.com*******

******Created by ebook converter - www.ebook-converter.com******

MITOCHONDRIA:
Q: Does the mitochondria have 1 or 2 membranes?
.
.
.
A: Mitochondria have 2 membranes
.
.
.
Q: Where in the mitochondria does the Krebs cycle occur?
.
.
.
A: The Krebs cycle occurs on the inner matrix
.
.
.

******ebook converter DEMO - www.ebook-converter.com*******

******Created by ebook converter - www.ebook-converter.com******

******ebook converter DEMO - www.ebook-converter.com*******

******Created by ebook converter - www.ebook-converter.com******

MOTILIN:
QUESTION: What is the effect of Motilin on the GI tract?
.
.
.
ANSWER: Motilin is a key stimulatory hormone of the GI tract
.
.
.
QUESTION: Where in the GI tract is the motilin receptor found?
.
.
.
ANSWER: The motilin receptor is found primarily in the stomach, duodenum, and colon
.
.

******ebook converter DEMO - www.ebook-converter.com*******

******Created by ebook converter - www.ebook-converter.com******

******ebook converter DEMO - www.ebook-converter.com*******

******Created by ebook converter - www.ebook-converter.com******

MUCOSA ASSOCIATED LYMPHOID TISSUE: MALT:


QUESTION MALT is a precursor to what kind of cancer?
.
.
.
ANSWER: MALT is a precursor to gastric lymphoma
.
.
.
QUESTION: What treatment causes regression of MALT?
.
.
.
ANSWER: Treatment of H. pylori causes regression of MALT
.
.
.
QUESTION: What is the initial treatment of MALT?
.
.
.
ANSWER: Initial treatment of MALT is triple therapy (amoxicillin, tetracycline, proton
pump inhibitor) for H. pylori. You can also substitute flagyl for tetracycline.

******ebook converter DEMO - www.ebook-converter.com*******

******Created by ebook converter - www.ebook-converter.com******

******ebook converter DEMO - www.ebook-converter.com*******

******Created by ebook converter - www.ebook-converter.com******

NECROTIZING FASCIITIS:
QUESTION: What physical exam finding is most suggestive of a necrotizing soft tissue
infection and would mandate immediate surgical exploration?
.
.
.
.
.
ANSWER:
- The physical exam finding that is most suggestive of a necrotizing soft tissue infection
includes grayish, turbid semipurulent material ("dishwater pus") that can be expressed,
as well as for the presence of skin changes (bronze hue or brawny induration), blebs, or
crepitus.
- The patient often develops pain at the site of infection that appears to be out of
proportion to any of the physical manifestations.
- Any of these findings mandates immediate surgical intervention, which should consist
of exposure and direct visualization of potentially infected tissue (including deep soft
tissue, fascia, and underlying muscle) and radical resection of affected areas.

******ebook converter DEMO - www.ebook-converter.com*******

******Created by ebook converter - www.ebook-converter.com******

******ebook converter DEMO - www.ebook-converter.com*******

******Created by ebook converter - www.ebook-converter.com******

NECROTIZING PANCREATITIS:
QUESTION: What type of nutrition has been shown to decrease the rate of pancreatic
abscess in patients with necrotizing pancreatitis?
.
.
.
.
.
ANSWER: Enteral nutrition
- In two small studies, enteral feedings initiated early, using nasojejunal feeding tubes
placed past the ligament of Treitz, have been associated with decreased development of
infected pancreatic necrosis, possibly due to a decrease in gut translocation of bacteria.
- Recent guidelines support the practice of enteral alimentation in these patients, with
the addition of parenteral nutrition if nutritional goals cannot be met by tube feeding
alone.

******ebook converter DEMO - www.ebook-converter.com*******

******Created by ebook converter - www.ebook-converter.com******

******ebook converter DEMO - www.ebook-converter.com*******

******Created by ebook converter - www.ebook-converter.com******

NEUROBLASTOMA:
QUESTION: A 2-year-old child presents with an abdominal mass, "raccoon eyes," and
"blueberry muffin" skin lesions. This most likely represents what disease?
.
.
.
.
.
ANSWER: Neuroblastoma.
- Neuroblastoma is the most common abdominal malignancy in children.
- The presenting symptoms depend on the site of the primary tumor, the presence of
metastatic disease, the age of the patient, and the metabolic activity of the tumor.
- The most common presentation is a fixed lobular mass extending from the flank toward
the midline.
- The tumor can also extend into the neural foramina and cause symptoms of spinal cord
compression.
- It tends to metastasize to cortical bones, bone marrow, and the liver, and patients may
present with localized swelling and tenderness, limp, or refusal to walk.
- Periorbital metastases account for proptosis and ecchymoses, resulting in "raccoon
eyes."
- In infants, liver metastases may expand, causing hepatomegaly.
- Metastatic lesions to the skin produce the blueberry muffin appearance.

******ebook converter DEMO - www.ebook-converter.com*******

******Created by ebook converter - www.ebook-converter.com******

******ebook converter DEMO - www.ebook-converter.com*******

******Created by ebook converter - www.ebook-converter.com******

NITRIC OXIDE: NO:


QUESTION: What is the effect of nitric oxide on the vascular system?
.
.
.
.
.
ANSWER:
- The NO-related vascular endothelial dysfunction is that nitric oxide causes vascular dilation.
.
.
.
.
.

QUESTION: What is the mechanism of action of nitrous oxide?


.
.
.
.
.
ANSWER:
- The mechanism of action of NO is that it increases cGMP, resulting in vascular smooth muscle dilation.

******ebook converter DEMO - www.ebook-converter.com*******

******Created by ebook converter - www.ebook-converter.com******

******ebook converter DEMO - www.ebook-converter.com*******

******Created by ebook converter - www.ebook-converter.com******

NITROUS OXIDE PNEUMOPERITONEUM: N2O


PNEUMOPERITONEUM:
QUESTION: Is N2O (nitrous oxide) pneumoperitoneum more or less analgesic than
CO2 pneumoperitoneum?
.
.
.
.
.
ANSWER: N2O pneumoperitoneum is more analgesic than CO2 pneumoperitoneum
- N2O had the advantage of being physiologically inert and rapidly absorbed.
- It also provided better analgesia for laparoscopy performed under local anesthesia
when compared with CO2 or air.
- Despite initial concerns that N2O would not suppress combustion, controlled clinical
trials have established its safety within the peritoneal cavity.

******ebook converter DEMO - www.ebook-converter.com*******

******Created by ebook converter - www.ebook-converter.com******

******ebook converter DEMO - www.ebook-converter.com*******

******Created by ebook converter - www.ebook-converter.com******

NUCLEOLUS:
Q: True or false: The nucleolus has no membrane
.
.
.
A: True

******ebook converter DEMO - www.ebook-converter.com*******

******Created by ebook converter - www.ebook-converter.com******

******ebook converter DEMO - www.ebook-converter.com*******

******Created by ebook converter - www.ebook-converter.com******

NUCLEUS:
Q: The nuclear membrane is continuous with what other organelle?
.
.
.
.
.
A: The nucleus outer membrane is continuous with the rough endoplasmic reticulum
.

******ebook converter DEMO - www.ebook-converter.com*******

******Created by ebook converter - www.ebook-converter.com******

******ebook converter DEMO - www.ebook-converter.com*******

******Created by ebook converter - www.ebook-converter.com******

OMEPRAZOLE:
QUESTION: What enzyme does omeprazole inhibit?
.
.
.
ANSWER: Omeprazole inhibits the H/K ATPase
.
.
.
QUESTION: Omeprazole inhibit acid secretion from which cell?
.
.
.
ANSWER: Omeprazole inhibits acid secretion from parietal cells

******ebook converter DEMO - www.ebook-converter.com*******

******Created by ebook converter - www.ebook-converter.com******

******ebook converter DEMO - www.ebook-converter.com*******

******Created by ebook converter - www.ebook-converter.com******

OMPHALOCELE:
QUESTION: The most common abnormality associated with omphalocele is what?
.
.
.
.
.
ANSWER: Cardiac.
- The incidence of other abnormalities in patients with omphalocele is approximately
60% to 70%.
- Cardiac anomalies are the most common, followed by musculoskeletal,
gastrointestinal, and genitourinary anomalies.

******ebook converter DEMO - www.ebook-converter.com*******

******Created by ebook converter - www.ebook-converter.com******

QUESTION: In omphalocele is the bowel covered by peritoneum?


.
.
.
.
.
ANSWER: Yes.
- Omphalocele refers to a congenital defect of the abdominal wall in which the bowel
and solid viscera are covered by peritoneum and the amniotic membrane.
- The abdominal wall defect can measure 4 cm or more in diameter.
- The size of the defect may be small or so large that it contains most of the abdominal
viscera.
- Omphalocele is less of a surgical emergency than gastroschisis because the bowel is
protected by the covering.
.
.
.
.
.
QUESTION: What conditions is omphalocele associated with?
.
.
.
.
.
ANSWER:
******ebook converter DEMO - www.ebook-converter.com*******

******Created by ebook converter - www.ebook-converter.com******


- Omphalocele is associated with many other congenital abnormalities that are not seen
with gastroschisis.
- Omphalocele occurs in association with exstrophy of the cloaca, BeckwithWiedemann syndrome, Cantrell pentalogy, ectopic cordis, and anterior midline
diaphragmatic hernia.
- There is an increased occurrence of cardiac and chromosomal abnormalities.
- Omphalocele is associated with premature and intrauterine growth retardation.

******ebook converter DEMO - www.ebook-converter.com*******

******Created by ebook converter - www.ebook-converter.com******

QUESTION: What is the treatment of omphalocele?


.
.
.
.
.
.
ANSWER:
- Immediate treatment of an infant with omphalocele consists of maintaining normal vital
signs and body temperature.
- The omphalocele should be covered with saline soaked gauze, and the trunk should be
wrapped circumferentially.
- No pressure should be placed in an effort to reduce the abdominal contents because
this maneuver may increase the risk of sac rupture or interfere with abdominal venous
return.
- Treatment is surgical.
- If the entire sac cannot be reduced, a temporary silo is placed.

******ebook converter DEMO - www.ebook-converter.com*******

******Created by ebook converter - www.ebook-converter.com******

******ebook converter DEMO - www.ebook-converter.com*******

******Created by ebook converter - www.ebook-converter.com******

OXALATE STONES:
QUESTION: What is the mechanism of renal oxalate stones after small bowel resection, specifically ileal
resection?

.
.
.
.
.
ANSWER:
- Ileal resection leads to decreased oxalate binding to calcium secondary to increased intraluminal fat.
- Oxalate then gets absorbed in colon, is released in urine, and can cause Ca oxalate kidney stones (hyperoxaluria)

- Ileal resection excessive

absorption of oxalate from the colon

- Normally, fatty acids are absorbed by the terminal ileum, and calcium and oxalate
combine to form an insoluble compound that is not absorbed.
- In the absence of the terminal ileum, unabsorbed fatty acids reach the colon, where
they combine with calcium, leaving free oxalate to be absorbed..

******ebook converter DEMO - www.ebook-converter.com*******

******Created by ebook converter - www.ebook-converter.com******

******ebook converter DEMO - www.ebook-converter.com*******

******Created by ebook converter - www.ebook-converter.com******

PALLIATIVE CARE:
QUESTION: The primary treatment for dyspnea ("air hunger") in a dying patient is what
medication?
.
.
.
.
.
ANSWER:
- The primary treatment of dyspnea (air hunger) in the dying is opioids, which should be
cautiously titrated to increase comfort and reduce tachypnea to a range of 15 to 20
breaths/min.
- Air movement across the face generated by a fan can sometimes be quite helpful.
- If this is not effective, empiric use of supplemental O2 by nasal cannula (2 to 3L/min)
may bring some subjective relief, independent of observable changes in pulse oximetry.
- Supplemental O2 should be humidified to avoid exacerbation of dry mouth.
- Typical starting doses of an immediate release opioid for breathlessness should be 1/2
- 2/3 of a starting dose of the same agent for cancer pain.
- For patients already on opioids for pain, a 25 to 50% increment in the dose of the
current immediate release agent for breakthrough pain often will be effective in
relieving breathlessness.

******ebook converter DEMO - www.ebook-converter.com*******

******Created by ebook converter - www.ebook-converter.com******

******ebook converter DEMO - www.ebook-converter.com*******

******Created by ebook converter - www.ebook-converter.com******

PARADOXICAL ACIDURIA:
TOPIC: COND ASSOC PARADOXICAL ACIDURIA

QUESTION: What conditions are associated with paradoxical aciduria? .


.
.
.
.
ANSWER:
- The conditions associated with paradoxical aciduria are pyloric stenosis and gastric outlet obstruction.

******ebook converter DEMO - www.ebook-converter.com*******

******Created by ebook converter - www.ebook-converter.com******

******ebook converter DEMO - www.ebook-converter.com*******

******Created by ebook converter - www.ebook-converter.com******

PARAESOPHAGEAL HERNIA:
QUESTION: What is the indication to operate in a patient with a paraesophageal
hernia?
.
.
.
ANSWER For paraesophageal hernias, you always operate since there is a risk of
incarceration and strangulation

******ebook converter DEMO - www.ebook-converter.com*******

******Created by ebook converter - www.ebook-converter.com******

******ebook converter DEMO - www.ebook-converter.com*******

******Created by ebook converter - www.ebook-converter.com******

PARALYTICS:
QUESTION: Which muscle in the body is the first muscle to recover from paralytics?
.
.
.
ANSWER: The diaphragm is the first muscle to recover from paralytics
.
.
.
QUESTION: Which muscles are the last to recover from paralytics?
.
.
.
ANSWER: The neck and face muscles are the last to recover from paralytics
.
.
.
QUESTION: Which antibiotic prolongs neuromuscular blockade?
.
.
.
ANSWER: Clindamycin prolongs neuromuscular blockade
- mnemonic. Clindamycin will "cling" to the "muscle" to make the blockade last longer.
.
.

******ebook converter DEMO - www.ebook-converter.com*******

******Created by ebook converter - www.ebook-converter.com******

******ebook converter DEMO - www.ebook-converter.com*******

******Created by ebook converter - www.ebook-converter.com******

PARATHYROID GLANDS:
QUESTION: From which branchial pouch does the inferior parathyroid glands arise?
.
.
.
.
.
ANSWER: The inferior parathyroid glands arise from the third branchial pouch.

******ebook converter DEMO - www.ebook-converter.com*******

******Created by ebook converter - www.ebook-converter.com******

******ebook converter DEMO - www.ebook-converter.com*******

******Created by ebook converter - www.ebook-converter.com******

PARATHYROID HORMONES: PTH:


QUESTION: What is the effect of PTH on bone?
.
.
.
.
.
ANSWER: PTH has a direct action on bone, stimulating osteoclastic activity.
- PTH has direct effects on the bone and kidneys and an indirect effect on the gut, all of
which result in an increased serum calcium concentration.
- In bone, PTH stimulates calcium release by enhancing resorption of bone matrix by
osteoclasts.
- In the kidneys, PTH increases tubular reabsorption of filtered calcium and decreases
tubular reabsorption of filtered phosphate (phosphaturic effect).
- Increased intestinal absorption of dietary calcium occurs indirectly through PTH
stimulation of renal vitamin D complex synthesis..

******ebook converter DEMO - www.ebook-converter.com*******

******Created by ebook converter - www.ebook-converter.com******

******ebook converter DEMO - www.ebook-converter.com*******

******Created by ebook converter - www.ebook-converter.com******

PARATHYROIDECTOMY:
QUESTION: What is the difference between subtotal parathyroidectomy and total
parathyroidectomy?
.
.
.
.
.
ANSWER:
- Subtotal parathyroidectomy typically involves resection of three and a half of the four
glands.
- Total parathyroidectomy consists of removing all four enlarged parathyroid glands and
auto transplanting part of one parathyroid gland into the forearm.
- Roughly equivalent results have been obtained with the two procedures, and debate
continues as to which is better.

******ebook converter DEMO - www.ebook-converter.com*******

******Created by ebook converter - www.ebook-converter.com******

******ebook converter DEMO - www.ebook-converter.com*******

******Created by ebook converter - www.ebook-converter.com******

PHEOCHROMOCYTOMA:
QUESTION: A 20 year old man comes to your office and has a blood pressure of
240/120. He states that he gets headaches sometimes when he lifts. Given the most
likely diagnosis, what is the best lab test to diagnose pheochromocytoma?
.
.
.
.
.
ANSWER:
- The best test for the diagnosis of pheochromocytoma is a 24 hour VMA and
metanephrine collection study.
.
.
.
.
.
QUESTION: What is the most sensitive test to diagnose a pheochromocytoma is
.
..
.
.
ANSWER: Plasma metanephrines.
- Recent studies have shown that plasma metanephrines are the most reliable tests to
identify pheochromocytomas, with sensitivity approaching 100%.
- Urinary metanephrines are 98% sensitive and are highly specific for
pheochromocytomas, whereas VMA measurements are slightly less sensitive and
specific.
******ebook converter DEMO - www.ebook-converter.com*******

******Created by ebook converter - www.ebook-converter.com******


.
.
.
.
.

******ebook converter DEMO - www.ebook-converter.com*******

******Created by ebook converter - www.ebook-converter.com******

QUESTION: What is the first drug to be started in a patient with symptomatic


pheochromocytoma?
.
.
.
.
.
ANSWER: Alpha blocker
- Alpha blockers such as phenoxybenzamine are started 1 to 3 weeks before surgery at
doses of 10 mg twice daily, which may be increased to 300 to 400 mg/day with
rehydration.
- Patients should be warned about orthostatic hypotension.
- Other alpha blockers such as prazosin and other classes of drugs such as ACE
inhibitors and calcium channel blockers are also useful.
.
.
.
.
.
QUESTION: What drug should you use to treat patient with preoperatively
pheochromocytoma who is already on an alpha blocker,and has persistent tachycardia?
.
.
.
.
.
******ebook converter DEMO - www.ebook-converter.com*******

******Created by ebook converter - www.ebook-converter.com******


ANSWER: Beta blockers
- Beta blockers such as propranolol at doses of 10 to 40 mg every 6 to 8 hours often
need to be added preoperatively in patients who have persistent tachycardia and
arrhythmias.
- Beta blockers should only be instituted after adequate alpha blockade and hydration to
avoid the effects of unopposed alpha stimulation, (i.e., hypertensive crisis and
congestive heart failure).
.
.
.
.
QUESTION: What can you do preoperative in order to avoid postoperative hypotension
in a patient with pheochromocytoma?
.
..
.
.
ANSWER: Replete volume with fluids preoperatively.
- Patients also should be volume repleted preoperatively to avoid postoperative
hypotension, which ensues with the loss of vasoconstriction after tumor removal.
.

******ebook converter DEMO - www.ebook-converter.com*******

******Created by ebook converter - www.ebook-converter.com******

******ebook converter DEMO - www.ebook-converter.com*******

******Created by ebook converter - www.ebook-converter.com******

PLASMA MEMBRANE:
QUESTION: What is the percent makeup of lipids and proteins in the plasma
membrane?
.
.
.
.
.
ANSWER:
. The percent breakdown of proteins and lipids in the plasma membrane is 40% lipid, 60% proteins.

******ebook converter DEMO - www.ebook-converter.com*******

******Created by ebook converter - www.ebook-converter.com******

******ebook converter DEMO - www.ebook-converter.com*******

******Created by ebook converter - www.ebook-converter.com******

PLASMIN:
QUESTION: What is the mechanism of plasmin?
.
.
.
.
ANSWER:
- The mechanism of action of plasmin is degradation of fibrinogen.

.
.
.
.
.
QUESTION: What drug inhibits plasmin?
.
.
.
.
ANSWER:
- The inhibitor of plasmin is aminocaproic acid (Amicar).

******ebook converter DEMO - www.ebook-converter.com*******

******Created by ebook converter - www.ebook-converter.com******

******ebook converter DEMO - www.ebook-converter.com*******

******Created by ebook converter - www.ebook-converter.com******

PORT WINE STAIN: CAPILLARY HEMANGIOMA:


QUESTION: What imaging is indicated in a patient with a capillary hemangioma (a
port-wine stain) present on the midface?
.
.
.
.
.
.
ANSWER: CT of the brain
- A capillary hemangioma (also known as a port-wine stain) present upon the midface
may signify Churg-Strauss syndrome, and computed tomography of the brain is
appropriate to rule out intracranial berry aneurysms.

******ebook converter DEMO - www.ebook-converter.com*******

******Created by ebook converter - www.ebook-converter.com******

******ebook converter DEMO - www.ebook-converter.com*******

******Created by ebook converter - www.ebook-converter.com******

POSTSPLENECTOMY SEPSIS:
QUESTION: A 3 year old undergoes splenectomy for hereditary spherocytosis. Six
weeks later, the child returns to the emergency room with a fever of 104, chills, rigors,
and a systolic blood pressure of 60. The child's white blood cell count is 20. Describe
the clinical features and risks of postsplenectomy sepsis.
.
.
.
.
ANSWER:
- The condition is more common in patients who undergo splenectomy for malignancy or
hematologic disease compared to trauma
- Children less than 5 years of age undergoing splenectomy are at higher risk
- The condition is due to a specific lack of immunity to encapsulated organisms
- Postsplenectomy sepsis syndrome is very rare after splenectomy.
- It occurs more commonly in patients undergoing splenectomy for non-traumatic
indications.
- The most common organism involved in PSSS is strep pneumoniae.
- Children <5 are at higher risk.
- The condition is due to a specific lack of immunity to encapsulated organisms (H.
influenzae, N. meningitidis, S. pneumoniae)

******ebook converter DEMO - www.ebook-converter.com*******

******Created by ebook converter - www.ebook-converter.com******

******ebook converter DEMO - www.ebook-converter.com*******

******Created by ebook converter - www.ebook-converter.com******

POSTTRANSPLANT LYMPHOPROLIFERATIVE
DISORDER: PTLD:
QUESTION: PTLD (posttransplant lymphoproliferative disorder) is caused by what
organism?
.
.
.
.
.
ANSWER: PTLD is caused by Epstein-Barr virus infection. The most severe form of
infection, PTLD can present as a localized tumor of the lymph nodes or GI tract, or
rarely as a rapidly progressive, diffuse, often fatal lymphomatous infiltration.

******ebook converter DEMO - www.ebook-converter.com*******

******Created by ebook converter - www.ebook-converter.com******

******ebook converter DEMO - www.ebook-converter.com*******

******Created by ebook converter - www.ebook-converter.com******

PREGNANCY:
QUESTION: Does the 2,3-DPG level increase or decrease during pregnancy? What is
physiologic effect of this?
.
.
.
.
.
ANSWER:
- In pregnancy, 2,3-DPG level is increased to enhance release of oxygen to the fetus.

.
.
.
.
.

******ebook converter DEMO - www.ebook-converter.com*******

******Created by ebook converter - www.ebook-converter.com******

QUESTION: What are the normal physiologic changes during pregnancy?


.
.
.
.
.
ANSWER:
- Pregnancy results in physiologic changes that may impact postinjury evaluation (TAble
7-9).
- Heart rate increases by 10-15 beats per minute during the first trimester and remains
elevated until delivery.
- Blood pressure diminishes during the first two trimesters due to a decrease in systemic
vascular resistance and rises again slightly during the third trimester (mean values: first
= 105/60, second = 102/55, third = 108/67)
- Intravascular volume is increased by up to 8L, which results in a relative anemia but
also a relative hypervolemia.
- Consequently, a pregnant woman may lose 35% of her blood volume before exhibiting
signs of shock.
- Pregnant patients have an increase in tidal volume and minute ventilation but a
decreased functional residual capacity; this results in a diminished PCO2 reading and
respiratory alkalosis.
- Also, pregnant patients may desaturate more rapidly, particularly in the supine
position and during intubation.
- Supplemental oxygen is always warranted in the trauma patient but is particularly
critical in the injured pregnant patient, because the oxygen dissociation curve is shifted
to the left for the fetus compared to the mother (i.e. small changes in maternal
oxygenation result in larger changes for the fetus because the fetus is operating in the
******ebook converter DEMO - www.ebook-converter.com*******

******Created by ebook converter - www.ebook-converter.com******


steep portions of the dissociation curve)
- As noted earlier, there is a relative anemia during pregnancy, but a hemoglobin level
of <11 is considered abnormal.
- Additional hematologic changes include a moderate leukocytosis (up to 20) and a
relative hypercoagulable state due to increased levels of factors VII, VII, IX, X, and XII
and decreased fibrinolytic activity.

******ebook converter DEMO - www.ebook-converter.com*******

******Created by ebook converter - www.ebook-converter.com******

******ebook converter DEMO - www.ebook-converter.com*******

******Created by ebook converter - www.ebook-converter.com******

PREVALENCE:
QUESTION: Define prevalence
.
.
.
ANSWER: Prevalence is the number of patients having the disease in the population
.
.
.
QUESTION: What is the effect of long standing disease on prevalence?
.
.
.
ANSWER: The prevalence is higher in diseases that last a long time
.
.

******ebook converter DEMO - www.ebook-converter.com*******

******Created by ebook converter - www.ebook-converter.com******

******ebook converter DEMO - www.ebook-converter.com*******

******Created by ebook converter - www.ebook-converter.com******

PROSTACYCLIN:
QUESTION: What is another term for prostacyclin?
.
.
.
ANSWER: Prostacyclin is PGI2
.
.
.
QUESTION: What is the effect of prostacyclin on platelet aggregation
.
.
.
ANSWER: Prostacyclin decreases platelet aggregation
.
.
.
QUESTION: What is the effect of prostacyclin on the vasculature?
.
.
.
ANSWER: Prostacyclin leads to vasodilation
.
.
.
QUESTION: What is the effect of prostacyclin on bronchial relaxation?
.
.
.
ANSWER: Prostacyclin causes bronchial relaxation
.

******ebook converter DEMO - www.ebook-converter.com*******

******Created by ebook converter - www.ebook-converter.com******

******ebook converter DEMO - www.ebook-converter.com*******

******Created by ebook converter - www.ebook-converter.com******

PROTAMINE:
QUESTION: True or false: Theoretically, 1.28mg of protamine neutralizes 1 mg of
heparin
.
.
.
.
.
ANSWER: The answer is TRUE.

******ebook converter DEMO - www.ebook-converter.com*******

******Created by ebook converter - www.ebook-converter.com******

******ebook converter DEMO - www.ebook-converter.com*******

******Created by ebook converter - www.ebook-converter.com******

PROTEIN C:
QUESTION: What coagulation factors does Protein C degrade?
.
.
.
ANSWER: Protein C degrades factors V and VIII and fibrinogen
.
.
.
.
.
QUESTION: What is the consequence of protein C or S deficiency?
.
.
.
.
.
ANSWER:
- The consequence of protein C deficiency (or protein S deficiency) is increased risk of spontaneous venous
thromboses.

******ebook converter DEMO - www.ebook-converter.com*******

******Created by ebook converter - www.ebook-converter.com******

******ebook converter DEMO - www.ebook-converter.com*******

******Created by ebook converter - www.ebook-converter.com******

PROTEIN S:
QUESTION: True or false: Protein S is a vitamin K dependent factor
.
.
.
ANSWER: True
.
.
.
.
.
QUESTION: What effect does Protein S have on Protein C?
.
.
.
.
.
ANSWER: Protein S helps protein C

******ebook converter DEMO - www.ebook-converter.com*******

******Created by ebook converter - www.ebook-converter.com******

******ebook converter DEMO - www.ebook-converter.com*******

******Created by ebook converter - www.ebook-converter.com******

PRUNE BELLY SYNDROME:


QUESTION: In addition to needing abdominal wall reconstruction, infants with prunebelly syndrome (Eagle-Barrett syndrome) will likely also require what additional
procedure?
.
.
.
.
.
ANSWER: Bilateral orchiopexy
- Prune-belly syndrome refers to a disorder that is characterized by a constellation of
symptoms including extremely lax lower abdominal musculature, a dilated urinary tract
including the bladder, and bilateral undescended testes.
- Despite ureteral dilatation, there is no role for ureteral surgery.
- The testes are intraabdominal.
- Bilateral orchipexy can be performed in conjunction with abdominal wall
reconstruction at 6 to 12 months of age.

******ebook converter DEMO - www.ebook-converter.com*******

******Created by ebook converter - www.ebook-converter.com******

******ebook converter DEMO - www.ebook-converter.com*******

******Created by ebook converter - www.ebook-converter.com******

PSEUDOMYXOMA PERITONEI OF APPENDICEAL


ORIGIN:
QUESTION: What type of cells cause pseudomyxoma peritonei of the appendix?
.
.
.
.
.
ANSWER:
- Pseudomyxoma is invariably caused by neoplastic mucus-secreting cells within the
peritoneum.
- These cells may be difficult to classify as malignant because they may be sparse,
widely scattered, and have a low-grade cytologic appearance.
.
.
.
.
.
QUESTION: What additional surgery is indicated in a patient with pseudomyxoma
peritonei of appendiceal origin?
.
.
.
.
.
ANSWER: hysterectomy with bilateral salpingo-oophorectomy
******ebook converter DEMO - www.ebook-converter.com*******

******Created by ebook converter - www.ebook-converter.com******


- Thorough surgical debulking is the mainstay of treatment.
- All gross disease and the omentum should be removed.
- If not done previously, appendectomy is routinely performed.
- Hysterectomy with bilateral salpingo-oophorectomy is performed in women.
.
.
.
.
QUESTION: What type of chemotherapy do these patients receive?
.
.
.
.
.
ANSWER: Intraperitoneal hyperthermic chemotherapy
- Because 5-year survival of mucinous appendiceal neoplasms is only 30%, adjuvant
intraperitoneal hyperthermic chemotherapy is advocated as a standard adjunct to radical
cytoreductive surgery.
- Abdominal XRT and systemic chemotherapy are not used in the treatment of
pseudomyxoma peritonei.
- Right hemicolectomy is not indicated as a routine procedure.

******ebook converter DEMO - www.ebook-converter.com*******

******Created by ebook converter - www.ebook-converter.com******

******ebook converter DEMO - www.ebook-converter.com*******

******Created by ebook converter - www.ebook-converter.com******

PT:
QUESTION: Does PT measure the intrinsic pathway or the extrinsic pathway?
.
.
.
.
.
ANSWER: PT measures the extrinsic pathway
.
.
.
.
.
.
QUESTION: What test is the single best test to evaluate synthetic function of the liver?
.
.
.
.
.
ANSWER: PT is the single best test to evaluate synthetic function of the liver
.
.
.
.
.

******ebook converter DEMO - www.ebook-converter.com*******

******Created by ebook converter - www.ebook-converter.com******

******ebook converter DEMO - www.ebook-converter.com*******

******Created by ebook converter - www.ebook-converter.com******

PTT:
QUESTION: Does PTT measure the intrinsic pathway or the extrinsic pathway?
.
.
.
.
.
.
ANSWER: PTT measures the intrinsic pathway
.
.
.
.
.
.
QUESTION: Collagen is in which state when the intrinsic pathway (PTT) is initiated?
.
.
.
.
.
ANSWER: The intrinsic pathway is activated when collagen is exposed
.
.
.
QUESTION: In the intrinsic pathway (PTT), exposed collagen reacts with which factor
to initiate the process that leads to the formation of fibrin?
.
.
.
ANSWER: Exposed collagen reacts with factor XII to initiate the coagulation process
that leads to the formation of fibrin
.
******ebook converter DEMO - www.ebook-converter.com*******

******Created by ebook converter - www.ebook-converter.com******


.
.

******ebook converter DEMO - www.ebook-converter.com*******

******Created by ebook converter - www.ebook-converter.com******

PULMONARY EMBOLISM:
QUESTION: What is the most common electrocardiographic change after pulmonary
embolism (PE)?
.
.
.
.
.
ANSWER: Sinus tachycardia
- The most common finding on electrocardiography after a PE is sinus tachycardia
(present in almost one half of patients).
- A heart rate greater than 100 beats per minute in the setting of suspected PE should
further raise concern.
- The classic finding on an electrocardiogram is the S1, Q3, T3 pattern, which consists
of a prominent S wave in lead I and a Q wave and inverted T wave in lead III.
- This electrocardiographic finding indicates right ventricular strain from a large PE,
but it is not commonly present.
- A large PE will lead to an enlargement of the right ventricle causing the
interventricular septum to deviate to the left.
- The right bundle branch stretches, leading to a right bundle branch block..

******ebook converter DEMO - www.ebook-converter.com*******

******Created by ebook converter - www.ebook-converter.com******

******ebook converter DEMO - www.ebook-converter.com*******

******Created by ebook converter - www.ebook-converter.com******

PULMONARY LYMPH NODES:


QUESTION: The lymphatic drainage from the pulmonary (N1) nodes of the lung
terminates in the lymphatic sump of Borrie which, on the right side, is located where?
.
.
.
.
.
ANSWER: Around the bronchus intermedius
- Lymph nodes that drain the lungs are divided into two groups according to the tumor,
node, and metastasis (TNM) staging system for lung cancer: the pulmonary lymph nodes,
N1; and the mediastinal nodes, N2.
- The N1 lymph nodes consist of the following:
a) intra-pulmonary or segmental nodes that lie at points of division of segmental bronchi
or in the bifurcations of the pulmonary artery
b) lobar nodes that lie along the upper, middle, and lower lobe bronchi
c) interlobar nodes that are located in the angles formed by the bifurcation of the main
bronchi into the lobar bronchi
d) hilar nodes that are located along the main bronchi
- The interlobar lymph nodes lie in the depths of the interlobar fissure on each side and
constitute a lymphatic sump for each lung, referred to as the lymphatic sump of Borrie.
- All of the pulmonary lobes of the corresponding lung drain into this group of nodes.
- On the right side, the nodes of the lymphatic sump lie around the bronchus intermedius
(bounded above by the right upper lobe bronchus and below by the middle lobe and
superior segmental bronchi).
- On the left side, the lymphatic sump is confined to the interlobar fissure, with the
lymph nodes in the angle between the lingular and lower lobe bronchi and in apposition
to the pulmonary artery branches.
******ebook converter DEMO - www.ebook-converter.com*******

******Created by ebook converter - www.ebook-converter.com******

.
source .

******ebook converter DEMO - www.ebook-converter.com*******

******Created by ebook converter - www.ebook-converter.com******

******ebook converter DEMO - www.ebook-converter.com*******

******Created by ebook converter - www.ebook-converter.com******

PYLORIC STENOSIS:
QUESTION: What medication, when given in early infancy, is linked to pyloric
stenosis?
.
.
.
.
.
ANSWER: Erythromycin
- Administration of erythromycin in early infancy has been linked to the development of
hypertrophic pyloric stenosis.
- It is linked with early administration of erythromycin.
.
.
.
.
.
QUESTION: True or false: Pyloric stenosis has a familial link in some cases.
.
.
.
.
ANSWER: True
.
.
.
.
******ebook converter DEMO - www.ebook-converter.com*******

******Created by ebook converter - www.ebook-converter.com******


.

******ebook converter DEMO - www.ebook-converter.com*******

******Created by ebook converter - www.ebook-converter.com******

QUESTION: How is pyloric stenosis diagnosed?


.
.
.
.
.
ANSWER: ultrasound
- Ultrasonography can establish the diagnosis accurately in 95% of cases by detecting
pyloric thickness (>4 mm thick for a length of 16 mm).
- It is readily diagnosed with ultrasonography.
.
.
.
.
.
QUESTION: What is the treatment of pyloric stenosis?
.
.
.
.
.
ANSWER: Pyloromyotomy
- The key element of initial management is the adequate replacement of fluids and
electrolytes.
- Pyloromyotomy is performed by making an incision in the serosa and muscle layers
******ebook converter DEMO - www.ebook-converter.com*******

******Created by ebook converter - www.ebook-converter.com******


and separating the muscle until the submucosa bulges out.
- Repair can be achieved laparoscopically.
.
.
.
.

******ebook converter DEMO - www.ebook-converter.com*******

******Created by ebook converter - www.ebook-converter.com******

QUESTION: True or false: Surgery requires a full-thickness division of the pylorus into
the mucosa.
.
.
.
.
.
ANSWER: FALSE.
- The submucosa and mucosa are not entered.

******ebook converter DEMO - www.ebook-converter.com*******

******Created by ebook converter - www.ebook-converter.com******

******ebook converter DEMO - www.ebook-converter.com*******

******Created by ebook converter - www.ebook-converter.com******

RADIATION THERAPY:
QUESTION: What phase of the cell cycle is most sensitive to radiation therapy?
.
.
.
ANSWER: The M phase of the cell cycle is the most sensitive to radiation therapy
.
.
.
QUESTION: True or false: Radiation therapy is most effective with high O2 levels
.
.
.
ANSWER: True
.
.
.
QUESTION: If you want to minimize skin damage, do you use higher or lower energy
radiation therapy?
.
.
.
ANSWER: Higher energy radiation therapy leads to less skin damage
.
.
.
QUESTION: What is the pathology seen on histology of tissue that has undergone
radiation therapy?
.
.
.
ANSWER: Pathology of tissue that underwent radiation therapy shows obliterative
endarteritis
.
.
.
******ebook converter DEMO - www.ebook-converter.com*******

******Created by ebook converter - www.ebook-converter.com******


QUESTION: Why is there decreased healing in areas that have undergone radiation
therapy?
.
.
.
ANSWER: Radiation therapy leads to decreased healing due to impaired fibroblasts
.

******ebook converter DEMO - www.ebook-converter.com*******

******Created by ebook converter - www.ebook-converter.com******

******ebook converter DEMO - www.ebook-converter.com*******

******Created by ebook converter - www.ebook-converter.com******

RECOMBINANT FACTOR VIIa: NOVOSEVEN:


QUESTION: What are the indications for recombinant factor VIIa (Novoseven)?
.
.
.
.
.
ANSWER:
- The indications for recombinant factor VIIa (Novoseven) are:
- life threatening bleeding in pt with Hemophilia A and B
- life threatening bleeding in a trauma patient, and
- factor VII deficiency.

.
.
..
QUESTION: What is the side effect of recombinant factor VIIa (Novoseven)?
.
.
.
.
ANSWER:
- Side effects recombinant factor VIIa: thromboembolic events.
.
.
.
.

QUESTION: In what clinical situation is recombinant factor VIIa (Novoseven)


******ebook converter DEMO - www.ebook-converter.com*******

******Created by ebook converter - www.ebook-converter.com******


contraindicated?
.
.
.
.
.
ANSWER:
Recombinant factor VIIa is contraindicated in DIC.

******ebook converter DEMO - www.ebook-converter.com*******

******Created by ebook converter - www.ebook-converter.com******

******ebook converter DEMO - www.ebook-converter.com*******

******Created by ebook converter - www.ebook-converter.com******

RECTUS SHEATH HEMATOMA:


QUESTION: What are the indications for surgery in a patient with a rectus sheath
hematoma?
.
.
.
.
.
ANSWER: The primary indications for operating on a patient with a rectus sheath
hematoma are hemodynamic instability and an expanding hematoma despite
embolization. .

******ebook converter DEMO - www.ebook-converter.com*******

******Created by ebook converter - www.ebook-converter.com******

******ebook converter DEMO - www.ebook-converter.com*******

******Created by ebook converter - www.ebook-converter.com******

REFEEDING SYNDROME:
QUESTION: Refeeding syndrome is associated with which electrolyte abnormalities?
..
.
.
ANSWER:
- Refeeding syndrome occurs when excess calories are given to a starved person
(anorexia).
- Refeeding syndrome is a potentially lethal condition that can occur with rapid and
excessive feeding of patients with severe underlying malnutrition due to starvation,
alcoholism, delayed nutritional support, anorexia nervosa, or massive weight loss in
obese patients.
- With refeeding, a shift in metabolism from fat to carbohydrate substrate stimulates
insulin release, which results in the cellular uptake of electrolytes and therefore low
serum levels of phosphate, magnesium, potassium, and calcium.
- In refeeding syndrome, the electrolyte abnormalities you should expect to see are:

- Hypophosphatemia
- Hypomagnesemia
- Hypokalemia
- Hypocalcemia
.
.
QUESTION: What is the process that results in the hypophosphatemia seen in refeeding
syndrome?
.
.
.
.
******ebook converter DEMO - www.ebook-converter.com*******

******Created by ebook converter - www.ebook-converter.com******


.
ANSWER:
- The cause of hypophosphatemia in refeeding syndrome is that the metabolism shifts from fat to carbohydrates,
stimulating insulin release, which results in cellular uptake of electrolytes, especially phosphate, magnesium,
potassium, and calcium, leading to low levels of these.

******ebook converter DEMO - www.ebook-converter.com*******

******Created by ebook converter - www.ebook-converter.com******

******ebook converter DEMO - www.ebook-converter.com*******

******Created by ebook converter - www.ebook-converter.com******

REJECTION:
QUESTION: Graft vs host disease is mediated by which immune cells?
.
.
.
ANSWER: Graft vs host disease is mediated by CD8+ T cells
.
.
.
QUESTION: Hyperacute rejection is due to which part of the immune system?
.
.
.
ANSWER: Hyperacute rejection is due to antibodies
.
.
.
QUESTION: Is hyperacute rejection due to preformed antibodies, antibodies that are
formed after the transplantation, or immune cells?
.
.
.
ANSWER: Hyperacute rejection is due to preformed antibodies.
.
.
.
QUESTION: How do you prevent hyperacute rejection occurring?
.
.
.
ANSWER: You can avoid hyperacute rejection by not transplanting when the
crossmatch is positive
.
.
.
******ebook converter DEMO - www.ebook-converter.com*******

******Created by ebook converter - www.ebook-converter.com******


QUESTION: The ischemia time of a harvested lung should ideally be less than how
many hours?
.
.
.
.
.
ANSWER: The ischemia time of a harvested lung should ideally be less than 6 hours.
Hearts and lungs tolerate preservation poorly; ideally, ischemia times should be below
6 hours.

******ebook converter DEMO - www.ebook-converter.com*******

******Created by ebook converter - www.ebook-converter.com******

******ebook converter DEMO - www.ebook-converter.com*******

******Created by ebook converter - www.ebook-converter.com******

RENIN:
QUESTION: What is the mechanism of action of renin?
.
.
.
.
.
ANSWER:
- The mechanism of action of renin is conversion of angiotensinogen to angiotensin I.

******ebook converter DEMO - www.ebook-converter.com*******

******Created by ebook converter - www.ebook-converter.com******

******ebook converter DEMO - www.ebook-converter.com*******

******Created by ebook converter - www.ebook-converter.com******

RESPIRATORY QUOTIENT: RQ:


QUESTION: What is the RQ of pure carbohydrate, protein, and fat metabolism?
.
.
.
.
.
ANSWER:
- The respiratory quotient of pure fat metabolism is 0.7.
- RQs for pure metabolism are carbohydrates 1.0, protein 0.8, fat 0.7.

.
.
.
.
.
QUESTION: What RQ signifies overfeeding? What RQ signifies starvation?
.
.
.
.
.
ANSWER:
- The respiratory quotient (RQ) is the ratio of CO2 produced to O2 consumed.
- An RQ>1.0 means too much carbohydrates, overfeeding.
- An RQ<1.0 means too little carbohydrates, starving.

******ebook converter DEMO - www.ebook-converter.com*******

******Created by ebook converter - www.ebook-converter.com******

******ebook converter DEMO - www.ebook-converter.com*******

******Created by ebook converter - www.ebook-converter.com******

RHABDOMYOLYSIS:
QUESTION: How do you decrease the risk of renal failure in a patient with
myoglobinuria?
.
.
.
.
.
ANSWER:
- The treatment of renal failure due to myoglobinuria in severe trauma patients has
shifted away from the use of sodium bicarbonate for alkalinizing the urine, to merely
maintaining brisk urine output of 100 mL/h with crystalloid fluid infusion.
- Mannitol and furosemide are not recommended as long as the IV fluid achieves the
goal rate of urinary output.
.
.
.
.
QUESTION: Treatment of patients with severe burn injuries and myoglobinuria consists
of what?
.
.
.
.
.
.
ANSWER: The best treatment for patients with burn injuries and myoglobinuria is fluid
resuscitation and HCO3- to prevent precipitation of the myoglobin.
******ebook converter DEMO - www.ebook-converter.com*******

******Created by ebook converter - www.ebook-converter.com******

******ebook converter DEMO - www.ebook-converter.com*******

******Created by ebook converter - www.ebook-converter.com******

******ebook converter DEMO - www.ebook-converter.com*******

******Created by ebook converter - www.ebook-converter.com******

RNA:
Q: Where in the cell is ribosomal RNA made?
.
.
.
A: Ribosomes RNA is made in the nucleolus

******ebook converter DEMO - www.ebook-converter.com*******

******Created by ebook converter - www.ebook-converter.com******

******ebook converter DEMO - www.ebook-converter.com*******

******Created by ebook converter - www.ebook-converter.com******

SARCOMAS:
QUESTION: For extremity sarcoma biopsy, when do you make an excisional biopsy,
and when do you make a longitudinal incisional biopsy?
.
.
.
ANSWER: You can get an excisional biopsy if the sarcoma is <4 cm. Otherwise do a
longitudinal incisional biopsy.
.
.
.
QUESTION: What is the benefit of a longitudinal incisional biopsy?
.
.
.
ANSWER: With a longitudinal incisional biopsy, you have less lymphatic disruption
and it is easier to excise scar if the biopsy positive
.
.
.
QUESTION: In treating soft tissue sarcomas, when is postoperative radiation therapy
indicated?
.
.
.
ANSWER: Postoperative radiation therapy is indicated for: high grade sarcoma, close
margins, or tumor >5 cm
.
.
.
QUESTION: How do soft tissue sarcomas spread?
.
.
.
ANSWER: Sarcomas spread hematogenously, not via lymphatics. Metastases to nodes
is rare.
******ebook converter DEMO - www.ebook-converter.com*******

******Created by ebook converter - www.ebook-converter.com******


.
QUESTION:: What characteristics is staging based on?
.
.
.
ANSWER: Staging is based on grade, NOT on the size or the nodes.
.
.
.
.

******ebook converter DEMO - www.ebook-converter.com*******

******Created by ebook converter - www.ebook-converter.com******

******ebook converter DEMO - www.ebook-converter.com*******

******Created by ebook converter - www.ebook-converter.com******

SECRETIN:
QUESTION: What is the effect of secretin on the pancreas?
.
.
.
ANSWER: Secretin is the primary stimulus of pancreatic bicarbonate secretion

******ebook converter DEMO - www.ebook-converter.com*******

******Created by ebook converter - www.ebook-converter.com******

******ebook converter DEMO - www.ebook-converter.com*******

******Created by ebook converter - www.ebook-converter.com******

SENSITIVITY:
QUESTION: What does the sensitivity of a test reflect?
.
.
.
ANSWER:
- Sensitivity reflects ability to detect disease
- [true positives/(true positive + false negatives)]
.
.
QUESTION:: True or false: Sensitivity is the ability of the test to detect disease
.
.
.
ANSWER: True
.
.
.
QUESTION: True or false: Sensitivity is # with disease and positive test result/ # that
have disease
.
.
.
ANSWER: True

******ebook converter DEMO - www.ebook-converter.com*******

******Created by ebook converter - www.ebook-converter.com******

******ebook converter DEMO - www.ebook-converter.com*******

******Created by ebook converter - www.ebook-converter.com******

SHORT GUT SYNDROME: SHORT BOWEL


SYNDROME:
QUESTION: What is short gut syndrome? What is the minimum length of bowel if there
is no ileocecal valve? What is the length of bowel if there is an ileocecal valve present?
.
.
.
.
.
ANSWER:
- Short gut syndrome is a clinical diagnosis of inability to absorb enough water and
nutritional elements to be off TPN
- The length of bowel in general needs to be at least 75cm if there is no ileocecal valve
- The length of bowel in general needs to be at least 50cm if the ileocecal valve is
present
- Short gut syndrome is a clinical diagnosis of inability to maintain appropriate
hydration and nutrition without the use of TPN.
- In general though, the length of bowel needs to be at least 50cm with the ileocecal
valve and at least 75cm without the ileocecal valve to avoid TPN.
.
.
.
.

******ebook converter DEMO - www.ebook-converter.com*******

******Created by ebook converter - www.ebook-converter.com******

QUESTION: What factors are associated with successful weaning of TPN in patients
with short bowel syndrome?
.
.
.
.
ANSWER
- Length of small bowel >200cm
- Presence of ileocecal valve
- Presence of colon
- Pediatric patients adapt better than adult patients.
TABLE - Risk factors for development of short bowel syndrome after massive small
bowel resection
- small bowel length <200cm
- Absence of ileocecal valve
- Absence of colon
- Diseased remaining bowel (eg. Crohn's disease)
- Ileal resection

******ebook converter DEMO - www.ebook-converter.com*******

******Created by ebook converter - www.ebook-converter.com******

******ebook converter DEMO - www.ebook-converter.com*******

******Created by ebook converter - www.ebook-converter.com******

SILVADENE:
QUESTION:What is the side effect of silvadene?
.
.
.
ANSWER: The side effect of Silvadene is neutropenia
.
.
.
.
.
.
QUESTION: Which one of the burn creams has good activity against candida?
.
.
.
ANSWER: Silvadene has good activity against Candida
.
.
.
QUESTION: Which one of the burn creams has poor eschar penetration?
.
.
.
ANSWER: Silvadene has poor eschar penetration

******ebook converter DEMO - www.ebook-converter.com*******

******Created by ebook converter - www.ebook-converter.com******

******ebook converter DEMO - www.ebook-converter.com*******

******Created by ebook converter - www.ebook-converter.com******

SILVER NITRATE:
QUESTION: What is the side effect of Silver nitrate?
.
.
.
ANSWER: The side effect of Silver nitrate is hyponatremia and hypochloremia due to
leaching of NaCl

******ebook converter DEMO - www.ebook-converter.com*******

******Created by ebook converter - www.ebook-converter.com******

******ebook converter DEMO - www.ebook-converter.com*******

******Created by ebook converter - www.ebook-converter.com******

SKIN GRAFTS:
QUESTION: Skin grafts survive in the 1st 48 hours primary by which mechanism?:
.
.
.
.
.
ANSWER:
- Skin grafts survive by imbibition (osmotic exchange of nutrients) for the 1st 48 hours.
- After that, neovascularization takes over.

******ebook converter DEMO - www.ebook-converter.com*******

******Created by ebook converter - www.ebook-converter.com******

******ebook converter DEMO - www.ebook-converter.com*******

******Created by ebook converter - www.ebook-converter.com******

SOMATOSTATIN:
QUESTION: Somatostatin inhibits the secretion of what molecules?
.
.
.
ANSWER: Somatostatin inhibits gastrin, insulin, secretin, and acetylcholine
.
.
.
QUESTION: What is the effect of somatostatin on pancreatic and biliary output?
.
.
.
ANSWER: Somatostatin decreases pancreatic and biliary output
.
.
.
QUESTION:: What stimulates the release of somatostatin?
.
.
.
ANSWER: Somatostatin is stimulated by acid in duodenum
.
.
.
QUESTION:: What hormone analog is octreotide?
.
.
.
ANSWER: Octreotide is a long acting somatostatin analog

******ebook converter DEMO - www.ebook-converter.com*******

******Created by ebook converter - www.ebook-converter.com******

******ebook converter DEMO - www.ebook-converter.com*******

******Created by ebook converter - www.ebook-converter.com******

SPECIFICITY:
QUESTION True or false: specificity is the ability to state no disease is present
.
.
.
ANSWER: True
.
.
.
QUESTION: True or false: Specificity is # with no disease and negative test result / #
without disease
.
.
.
ANSWER: True

******ebook converter DEMO - www.ebook-converter.com*******

******Created by ebook converter - www.ebook-converter.com******

******ebook converter DEMO - www.ebook-converter.com*******

******Created by ebook converter - www.ebook-converter.com******

SULFAMYLON:
QUESTION: What is the side effect of Sulfamylon?
.
.
.
ANSWER: The side effect of Sulfamylon is acidosis due to carbonic anhydrase
inhibition (less H2CO3 -> H2O + CO2)
.
.
.
.
.
.
QUESTION: Which one of the burn creams is painful on application?
.
.
.
ANSWER: Sulfamylon is painful on application

******ebook converter DEMO - www.ebook-converter.com*******

******Created by ebook converter - www.ebook-converter.com******

******ebook converter DEMO - www.ebook-converter.com*******

******Created by ebook converter - www.ebook-converter.com******

SUPERIOR VENA CAVA SYNDROME: SVC


SYNDROME:
QUESTION: Superior vena cava syndrome most frequently occurs with which lung
cancer type?
.
.
.
.
.
ANSWER: SCLC.
..
.
.
.
.
QUESTION: A 65 year old cachectic man with an apical lung cancer presents with
swollen face, arms and hands. The best treatment for this patient's SVC syndrome, is
what?
.
.
.
.
ANSWER
- This patient most likely has superior vena cava syndrome (SVC syndrome) due to
cancer in-growth into the SVC.
- Treatment of this problem requires XRT.
******ebook converter DEMO - www.ebook-converter.com*******

******Created by ebook converter - www.ebook-converter.com******


- Resection is possible if just a very small portion of the SVC is involved with the
lesion (usually found at the time of thoracotomy).
- Resection is not indicated if the patient has SVC syndrome.
- Invasion of the SVC is considered a T4 tumor.

******ebook converter DEMO - www.ebook-converter.com*******

******Created by ebook converter - www.ebook-converter.com******

******ebook converter DEMO - www.ebook-converter.com*******

******Created by ebook converter - www.ebook-converter.com******

SURGICAL SITE INFECTIONS: SSI: SSIs:


QUESTION: What are the risk factors developing a surgical site infection (SSI)?
.
.
.
ANSWER:
TABLE: Risk factors for development of surgical site infections
PATIENT FACTORS
- older age
- immunosuppression
- obesity
- diabetes mellitus
- chronic inflammatory process
- malnutrition
- peripheral vascular disease
- anemia
- radiation
- chronic skin disease
- carrier state (chronic staphylococcal carriage)
- recent operation
LOCAL FACTORS
- poor skin preparation
- contamination of instruments
- inadequate antibiotic prophylaxis
******ebook converter DEMO - www.ebook-converter.com*******

******Created by ebook converter - www.ebook-converter.com******


- prolonged procedure
- local tissue necrosis
- hypoxia, hypothermia
MICROBIAL FACTORS
- prolonged hospitalization (leading to nosocomial organisms)
- toxin secretion
- resistance to clearance (eg. capsule formation)

******ebook converter DEMO - www.ebook-converter.com*******

******Created by ebook converter - www.ebook-converter.com******

SYSTEMIC VASCULAR RESISTANCE: SVR:


QUESTION:. What is the equation for SVR?
.
.
.
.
.
ANSWER:
Systemic vascular resistance (SVR) is calculated by the following equation:SVR =
MAP - CVP/CO x 80;
MAP = DBP + (SBP -DBP) where MAP is mean arterial pressure, CVP is central
venous pressure, CO is cardiac output, DBP is diastolic blood pressure, and SBP is
systolic blood pressure.

******ebook converter DEMO - www.ebook-converter.com*******

******Created by ebook converter - www.ebook-converter.com******

******ebook converter DEMO - www.ebook-converter.com*******

******Created by ebook converter - www.ebook-converter.com******

TENSION PNEUMOTHORAX:
QUESTION: A 21-year-old man who was the driver in a head-on collision has a pulse
rate of 140 beats per minute, respiratory rate of 36 breaths per minute, and blood
pressure of 60 mm Hg palpable. His trachea is deviated to the left, with palpable
subcutaneous emphysema and poor breath sounds in the right hemithorax. The most
appropriate initial treatment is to do what?
.
.
.
.
.
ANSWER: Needle decompression
- Advanced trauma life support protocol states that the initial management is to place a
needle in the second intercostal space, midclavicular line, just above the rib using a
4.5-cm (2-inch) catheter (5-cm needle).
- This should be immediately followed by a tube thoracostomy.

******ebook converter DEMO - www.ebook-converter.com*******

******Created by ebook converter - www.ebook-converter.com******

******ebook converter DEMO - www.ebook-converter.com*******

******Created by ebook converter - www.ebook-converter.com******

TESTICULAR CANCER:
QUESTION: AFP is a marker for which type of testicular cancer?
.
.
.
ANSWER: AFP is a marker for non-seminomatous testicular cancer
.
.
.
QUESTION: beta-HCG is a marker for which type of testicular cancer?
.
.
.
ANSWER: beta-HCG is a marker for non-seminomatous testicular cancer
.
.
.
QUESTION: In the workup of a testicular mass, do you biopsy the mass via
orchiectomy via inguinal incision or via a trans-scrotal incision?
.
.
.
ANSWER: In the workup of a testicular mass, the biopsy is orchiectomy via inguinal
incision. It is NEVER via trans-scrotal.
.
.
.
QUESTION: Which testicular cancer is very radiosensitive? Seminoma or
nonseminoma?
.
******ebook converter DEMO - www.ebook-converter.com*******

******Created by ebook converter - www.ebook-converter.com******


.
.
ANSWER: Seminoma are very radiosensitive.
.
.
.
QUESTION: True or false: There is a role for radiation therapy in Stage I seminomas
.
.
.
ANSWER: True. Even stage I seminomas get radiation therapies. It turns out that 25%
of patients with seminomas have occult metastasis.
.
.
.
QUESTION: When is chemotherapy indicated when treating a seminoma?
.
.
.
ANSWER: If a patient is node positive, the patient gets platinum chemotherapy.
.
.
.
QUESTION: Is testicular cancer more common in children, teenagers, adults 20-40,
older adults (40-60), or elderly (>65) years of age?
.
.
.
ANSWER: Testicular cancer is usually found in men ages 20-35.
.
.
******ebook converter DEMO - www.ebook-converter.com*******

******Created by ebook converter - www.ebook-converter.com******


.
QUESTION: Is testicular cancer more commonly benign or malignant?
.
.
.
ANSWER: Testicular cancer is most often malignant
.
.
.

******ebook converter DEMO - www.ebook-converter.com*******

******Created by ebook converter - www.ebook-converter.com******

QUESTION: In which Ethnic group would you not expect to find testicular cancer?
.
.
.
ANSWER: Testicular cancer is rare in African-Americans
.
.
.

******ebook converter DEMO - www.ebook-converter.com*******

******Created by ebook converter - www.ebook-converter.com******

******ebook converter DEMO - www.ebook-converter.com*******

******Created by ebook converter - www.ebook-converter.com******

TETRALOGY OF FALLOT:
QUESTION: What is the most common congenital heart defect causing a right-to-left
shunt?
.
.
.
.
.
ANSWER: Tetralogy of Fallot
- Tetralogy of Fallot is the most common congenital heart defect presenting as a "blue
baby" (cyanosis from a right-to-left shunt).
- The four features include VSD, dextroposition of the heart (overriding aorta), right
ventricular outflow tract obstruction, and right ventricular hypertrophy.
- Most surgeons recommend repair during infancy, but if the patient is unstable and is
younger than 6 months old, Blalock-Taussig shunt placement may be performed.

******ebook converter DEMO - www.ebook-converter.com*******

******Created by ebook converter - www.ebook-converter.com******

******ebook converter DEMO - www.ebook-converter.com*******

******Created by ebook converter - www.ebook-converter.com******

THROMBIN:
QUESTION: What is the mechanism of action of thrombin?
.
.
.
.
.
ANSWER:
- The mechanism of action of thrombin is conversion of fibrinogen to fibrin and fibrin split products, and
activation of platelets.

******ebook converter DEMO - www.ebook-converter.com*******

******Created by ebook converter - www.ebook-converter.com******

******ebook converter DEMO - www.ebook-converter.com*******

******Created by ebook converter - www.ebook-converter.com******

THROMBOXANE:
QUESTION: What is the other word for thromboxane?
.
.
.
ANSWER: Thromboxane and TXA2 are interchangeable terms
.
.
.
QUESTION: What cells or cell components secrete thromboxane?
.
.
.
ANSWER: Platelets secrete thromboxane
.
.
.
QUESTION: Does thromboxane increase or decrease aggregation of platelets?
.
.
.
ANSWER: Thromboxane increases platelet aggregation
.
.
.
QUESTION: Does thromboxane cause vasoconstriction or vasodilation?
.
.
.
ANSWER: Thromboxane causes vasoconstriction
.
.
.

******ebook converter DEMO - www.ebook-converter.com*******

******Created by ebook converter - www.ebook-converter.com******

QUESTION: What is the effect of thromboxane on the calcium level inside platelets?
.
.
.
ANSWER: Thromboxane triggers release of calcium in platelets
.
.
.
QUESTION: What is the effect of increased calcium levels inside platelets on the
GpIIb/IIIa receptor?
.
.
.
ANSWER: Release of calcium in platelets exposes GpIIb/IIIa receptor on platelets
.
.
.
QUESTION: What is the effect of exposure of the GpIIb/IIIa receptor on platelets?
.
.
.
ANSWER: Exposure of the GpIIb/IIIa receptor on platelets causes platelet-to-platelet
binding and platelet to collagen binding

******ebook converter DEMO - www.ebook-converter.com*******

******Created by ebook converter - www.ebook-converter.com******

******ebook converter DEMO - www.ebook-converter.com*******

******Created by ebook converter - www.ebook-converter.com******

THYROGLOSSAL CYSTS:
QUESTION: What is the treatment for thyroglossal ysts?
.
.
.
.
.
ANSWER: Sistrunk procedure
- The classic treatment of thyroglossal duct cyst is the Sistrunk procedure, which
involves complete excision of the cyst in continuity with its tract, the central portion of
the hyoid bone, and the tissue above the hyoid bone extending to the base of the tongue.
.

******ebook converter DEMO - www.ebook-converter.com*******

******Created by ebook converter - www.ebook-converter.com******

******ebook converter DEMO - www.ebook-converter.com*******

******Created by ebook converter - www.ebook-converter.com******

THYROID STORM:
QUESTION: What is the most appropriate initial treatment for thyroid storm?

.
.
.
ANSWER:
- Beta-blockers are used in the initial treatment of thyroid storm.
- Treatment of thyroid storm includes:
- Beta blockers (esmolol drip usually)
- Lugol's (KI)
- Propylthiouracil
- Cooling blankets
- Oxygen
- Glucose
- Fluid
- Steroids (Cortisol 100 mg Q8H)

******ebook converter DEMO - www.ebook-converter.com*******

******Created by ebook converter - www.ebook-converter.com******

******ebook converter DEMO - www.ebook-converter.com*******

******Created by ebook converter - www.ebook-converter.com******

TRACHEA:
QUESTION: What is the approximate length of the trachea distal to the subglottic
space?
.
.
.
.
.
ANSWER: 10-13 cm
- The trachea is composed of cartilaginous and membranous portions, beginning with
the cricoid cartilage, the first complete cartilaginous ring of the airway.
- The cricoid cartilage consists of an anterior arch and a posterior broad-based plate.
- Articulating with the posterior cricoid plate are the arytenoid cartilages.
- The vocal cords originate from the arytenoid cartilages and then attach to the thyroid
cartilage.
- The subglottic space, the narrowest part of the trachea with an internal diameter of
approximately 2 cm, begins at the inferior surface of the vocal cords and extends to the
first tracheal ring.
- The remainder of the distal trachea is 10.0 to 13.0 cm long, consists of 18 to 22 rings,
and has an internal diameter of 2.3 cm.

******ebook converter DEMO - www.ebook-converter.com*******

******Created by ebook converter - www.ebook-converter.com******

******ebook converter DEMO - www.ebook-converter.com*******

******Created by ebook converter - www.ebook-converter.com******

TRACHEOESOPHAGEAL FISTULAS: TE FISTULAS:


QUESTION: Describe the surgical treatment of TE fistulas.
.
.
.
.
.
ANSWER:
- The surgical treatment for the most common type of TEF involves an extrapleural
thoracotomy through the right fourth intercostal space, although more recently it has been
performed thoracoscopically.
- The azygos vein is a key landmark because once it is divided, the TEF is usually
underneath.
- The TEF is dissected circumferentially and ligated with interrupted nonabsorable
sutures.
- The proximal esophageal pouch is then mobilized as high as possible to afford a
tension-free esophageal anastomosis.
- The blood supply to the upper esophageal pouch is based on arteries derived from the
thyrocervical trunk.
- However, the blood supply to the lower esophagus is more tenuous and segmental,
originating from the intercostal vessels.
- As such, significant mobilization of the lower esophagus should not be done to avoid
ischemia of the anastomosis.
- The extrapleural approach is preferred, although the transpleural one is easier to
perform.

******ebook converter DEMO - www.ebook-converter.com*******

******Created by ebook converter - www.ebook-converter.com******

******ebook converter DEMO - www.ebook-converter.com*******

******Created by ebook converter - www.ebook-converter.com******

TRACHEOSTOMY:
QUESTION: If you see massive bleeding from a trach, what artery is involved?
.
.
.
ANSWER: If you see a massive bleed from a tracheostomy site, the innominate artery
is involved.
.
.
.
QUESTION: If you see massive bleeding from a trach, what kind of fistula could the
patient have developed?
.
.
.
ANSWER: Bleeding from a trach site could represent a tracheo-innominate fistula
.
.
.
QUESTION: When making a tracheostomy, how can you avoid creating a tracheoinnominate fistula or injuring the innominate artery?
.
.
.
ANSWER: You can avoid injuring the innominate artery by making the tracheostomy no
lower than 3rd tracheal ring.
.

******ebook converter DEMO - www.ebook-converter.com*******

******Created by ebook converter - www.ebook-converter.com******

******ebook converter DEMO - www.ebook-converter.com*******

******Created by ebook converter - www.ebook-converter.com******

TRANSFUSION REACTIONS:
QUESTION: What is the mechanism of acute hemolysis?
.
.
.
.
.
ANSWER:
- The mechanism of acute hemolysis is ABO incompatibility.
.
.
.
.
.

QUESTION: What is the most common cause of transfusion related deaths?


.
.
.
.
.
ANSWER:
- The most common cause of transfusion related deaths is ABO incompatibility.
.
.
.
.
.

QUESTION: What is likely occurring in an anesthetized patient who develops diffuse


bleeding after transfusion?
******ebook converter DEMO - www.ebook-converter.com*******

******Created by ebook converter - www.ebook-converter.com******


.
.
.
.
.
ANSWER:
- The transfusion reaction in an anesthetized patient who develops diffuse bleeding and hypotension is likely to be
acute hemolysis due to ABO compatibility causing DIC.

QUESTION: What are the complications of acute hemolysis?


.
.
.
.
.
ANSWER:
- The complications from acute hemolysis include hypotensive shock, ATN, DIC, and hemoglobinuria.
.
.
.
.
.

QUESTION: What is the treatment of acute hemolysis?


.
.
.
.
.
ANSWER:
- The treatment of acute hemolysis is to

******ebook converter DEMO - www.ebook-converter.com*******

******Created by ebook converter - www.ebook-converter.com******


1) stop transfusion
2) restore blood pressure and renal perfusion by giving fluids and using vasopressors
3) give mannitol and bicarb to alkaline urine to prevent precipitation of hemoglobin in renal
tubules
) Benadryl.
.
.
.
.
.

******ebook converter DEMO - www.ebook-converter.com*******

******Created by ebook converter - www.ebook-converter.com******

QUESTION: What is the mechanism of delayed hemolysis?


.
.
.
.
.
ANSWER:
- The mechanism of delayed hemolysis is antibodies against minor antigens.
.
.
.
.
.

QUESTION: What is the most common transfusion reaction?


.
.
.
.
.
ANSWER:
- The most common transfusion reaction is febrile nonhemolytic transfusion reaction.
.
.
.
.
.

QUESTION: What is the mechanism of febrile nonhemolytic transfusion reaction?


******ebook converter DEMO - www.ebook-converter.com*******

******Created by ebook converter - www.ebook-converter.com******


.
.
.
.
.
ANSWER:
- The mechanism of febrile nonhemolytic transfusion reaction is recipient antibodies react against donor WBCs.
.
.
.
.
.

QUESTION: What is the treatment of febrile nonhemolytic transfusion reaction?


.
.
.
.
.
ANSWER:
- The treatment of febrile nonhemolytic transfusion reaction is stop transfusion; WBC filters for subsequent
transfusions.
.
.
.
.
.

QUESTION: What is the mechanism of urticaria after transfusion?


.
.
.
******ebook converter DEMO - www.ebook-converter.com*******

******Created by ebook converter - www.ebook-converter.com******


.
.
ANSWER:
- The mechanism of urticaria after transfusion is a reaction against plasma proteins or IgA in transfused blood.
.
.
.
.
.

QUESTION: What is the treatment of anaphylaxis after transfusion?


.
.
.
.
.
ANSWER:
- The treatment for anaphylaxis after transfusion is fluids, pressors, steroids, epinephrine, benadryl, lasix.
.
.
.
.
.

QUESTION: What is the most common organism to contaminate blood products?


.
.
.
.
.
ANSWER:
******ebook converter DEMO - www.ebook-converter.com*******

******Created by ebook converter - www.ebook-converter.com******


- The most common contaminant organism in blood products is E. coli.
.
.
.
.
.

QUESTION: What is the most commonly contaminated blood product ?


.
.
.
.
.
ANSWER:
- The most common blood product source of contamination is platelets.
.
.
.
.
.

QUESTION: What is the mechanism of TRALI?


.
.
.
.
.
ANSWER:
- The mechanism of TRALI is the donor antibodies attack recipient WBCs in pulmonary vasculature.
.
.
.

******ebook converter DEMO - www.ebook-converter.com*******

******Created by ebook converter - www.ebook-converter.com******


QUESTION: What is the ventilator setting in TRALI?
.
.
.
.
.
ANSWER:
- The ventilator setting often needed during treatment of TRALI is high PEEP.

******ebook converter DEMO - www.ebook-converter.com*******

******Created by ebook converter - www.ebook-converter.com******

******ebook converter DEMO - www.ebook-converter.com*******

******Created by ebook converter - www.ebook-converter.com******

TRANSLATION:
Q: Where in the cell does translation of mRNA into proteins occur?
.
.
.
.
.
A: The process of decoding information on mRNA to synthesize proteins is called
translation. Translation takes place in ribosomes composed of rRNA and ribosomal
proteins.

******ebook converter DEMO - www.ebook-converter.com*******

******Created by ebook converter - www.ebook-converter.com******

******ebook converter DEMO - www.ebook-converter.com*******

******Created by ebook converter - www.ebook-converter.com******

ULCERATIVE COLITIS:
QUESTION: The risk of colon cancer in a patient who was diagnosed with ulcerative
colitis 20 years ago is approximately what percentage?
.
.
.
.
.
.
ANSWER
- Risk of malignancy increases with pancolonic disease and the duration of symptoms is
approximately 2% after 10 years, 8% after 20 years, and 18% after 30 years.
- Unlike sporadic colorectal cancers, carcinoma developing in the context of ulcerative
colitis is more likely to arise from areas of flat dysplasia and may be difficult to
diagnose at an early stage.
.
.
.
.
.

******ebook converter DEMO - www.ebook-converter.com*******

******Created by ebook converter - www.ebook-converter.com******

QUESTION: Elective proctocolectomy should be advised for which types of patients


with ulcerative colitis?
.
.
.
.
.
ANSWER: Elective proctocolectomy in UC patient should be done in these situations:
- Low grade dysplasia on biopsy
- Moderate dysplasia on biopsy
- Pancolonic disease for >20 years, independent of biopsy results
- Although low grade dysplasia was long thought to represent minimal risk, more recent
studies show that invasive cancer may be present in up to 20% of patients with low
grade dysplasia.
- For this reason, any patient with dysplasia should be advised to undergo
proctocolectomy.
- Controversy exists over whether prophylactic proctocolectomy should be
recommended for patients who have had chronic ulcerative colitis for >10 years in the
absence of dysplasia.
- Proponents of this approach note that surveillance colonoscopy with multiple
biopsies, samples only a small fraction of the colonic mucosa and dysplasia and
carcinoma are often missed.
- Opponents cite the relatively low risk of progression to carcinoma if all biopsies have
lacked dysplasia (approximately 2.4%)
-Neither approach has been definitively shown to decrease mortality from colorectal
cancer.
- Risk of malignancy increases with pancolonic disease and the duration of symptoms is
approximately 2% after 10 years, 8% after 20 years, and 18% after 30 years.
******ebook converter DEMO - www.ebook-converter.com*******

******Created by ebook converter - www.ebook-converter.com******

******ebook converter DEMO - www.ebook-converter.com*******

******Created by ebook converter - www.ebook-converter.com******

******ebook converter DEMO - www.ebook-converter.com*******

******Created by ebook converter - www.ebook-converter.com******

UMBILICAL HERNIAS:
QUESTION: What is the most important initial therapy for a patient with portal
hypertension, ascites, and a tense umbilical hernia?
.
.
.
.
.
.
ANSWER:
- Medical therapy to control the ascites.
- Treatment and control of the ascites with diuretics, dietary management, and
paracentesis is the most appropriate initial therapy.
- Patients with refractory ascites may be candidates for transjugular intrahepatic
portocaval shunting or eventual liver transplantation.
- Umbilical hernia repair should be deferred until after the ascites is controlled.
.
.
.
.
.
QUESTION: True or false: Most umbilical hernias in adults are congenital.
.
.
.
.

******ebook converter DEMO - www.ebook-converter.com*******

******Created by ebook converter - www.ebook-converter.com******


ANSWER: False.
- Unlike in children, umbilical hernias in adults are usually acquired.
- Risk factors are any conditions that increase intra-abdominal pressure, such as
pregnancy, obesity, and ascites.
.
.
.
.
.
QUESTION: True or false: repair of umbilical hernias is contraindicated in patients
with cirrhosis.
.
.
.
.
.
ANSWER: False
- In patients with cirrhosis and ascites, the markedly increased pressure causes the skin
overlying the hernia to become thin and eventually ischemic.
- One of the most catastrophic complications in this setting is rupture of the hernia
through the ischemic skin, leading to peritonitis and death.
- Thus, patients with cirrhosis and ascites should undergo repair if there is evidence that
the skin overlying the hernia is thinning or becoming ischemic.
- However, repair should be delayed until after medical management of the ascites.
- If medical management fails and the skin over the hernia is thinned and tense, then a
transjugular portosystemic shunt should be considered before repair.
- Alternatively, if the patient is a transplantation candidate, the hernia can be repaired
during the transplantation.

******ebook converter DEMO - www.ebook-converter.com*******

******Created by ebook converter - www.ebook-converter.com******


.
.
.
.
.
QUESTION: True or false: Strangulation in the setting of umbilical hernias is less
common in adults than in children.
.
.
.
ANSWER: False
- Overall strangulation of umbilical hernias is uncommon, but occurs more often than in
children.
QUESTION: True or false: In regards to umbilical hernias in adults, small,
asymptomatic hernias can be clinically observed.
.
.
.
.
.
ANSWER: True.
- Small barely palpable and asymptomatic hernias can be followed clinically.
- Larger or symptomatic hernias should be repaired.
.
.
.
******ebook converter DEMO - www.ebook-converter.com*******

******Created by ebook converter - www.ebook-converter.com******


QUESTION: True or false: In regards to umbilical hernias in adults, primary closure
has recurrence rates similar to those of mesh repair.
.
.
.
.
.
ANSWER: False.
- Umbilical hernias have historically all been repaired by primary closure.
- Borrowing from the low recurrence rates using mesh for inguinal hernias, umbilical
hernias are now more frequently being repaired using mesh, particularly those with
large defects.
- A recent prospective, randomized study compared primary closure with mesh repair.
- The early complication rates such as seroma, hematoma, and wound infection were
similar in the two groups.
- However, the hernia recurrence rate was significantly higher after primary suture
repair (11%) than after mesh repair (1%).
- Some authors are now advocating routine use of mesh for all adult umbilical hernias in
the absence of bowel strangulation.
.
.

******ebook converter DEMO - www.ebook-converter.com*******

******Created by ebook converter - www.ebook-converter.com******

QUESTION: When should you offer operative repair to a patient with ascites secondary
to cirrhosis who presents for elective umbilical hernia repair?
.
.
.
.
.
ANSWER: You should offer repair to this patient if they are leaking ascites from the
hernia or at risk of doing so.
- One criterion supporting the surgical repair of an umbilical hernia is that patients are
leaking ascites, and thereby,placing them at higher risk of peritonitis..
- It is of utmost importance to minimize ascites preoperatively.
- Repair of an umbilical hernia without adequate control of ascites preoperatively
contributes to a 73% recurrence rate.
.
.
.
.
.
QUESTION: How do you manage ascites prior to umbilical hernia repair in a patient
with cirrhosis and ascites?
.
.
.
.
.
ANSWER: Medical control of ascites entails fluid and salt restriction, diuretics, and
******ebook converter DEMO - www.ebook-converter.com*******

******Created by ebook converter - www.ebook-converter.com******


possibly paracentesis.
.
.
.
.
.

******ebook converter DEMO - www.ebook-converter.com*******

******Created by ebook converter - www.ebook-converter.com******

QUESTION: What operation can you perform prior to umbilical hernia repair in patient
with cirrhosis and ascites to reduce their ascites?
.
.
.
.
.
ANSWER: You can perform TIPS. Transjugular intrahepatic portosystemic shunt may
be used preoperatively to reduce the production of ascites.
.
.
.
.
.
.
QUESTION: What are the complications of umbilical hernia repair in patient with
cirrhosis and ascites?
.
.
.
.
ANSWER: Surgery on these patients is complicated by the risk of hemorrhage
secondary to variceal disruption, peritonitis, postoperative ascites leak, and hepatic
decompensation.
- Therefore,such a surgical undertaking should be considered only in select patients.

******ebook converter DEMO - www.ebook-converter.com*******

******Created by ebook converter - www.ebook-converter.com******

******ebook converter DEMO - www.ebook-converter.com*******

******Created by ebook converter - www.ebook-converter.com******

QUESTION: True or false: The repair of umbilical hernias in patients with cirrhosis is
associated with a high rate of recurrence.
..
.
.
ANSWER: True.
- The high rate of recurrence is secondary to the production of ascites and nutritional
deficiencies, resulting in muscular wasting and fascial thinning.
- Repair of an umbilical hernia without adequate control of ascites preoperatively
contributes to a 73% recurrence rate..
.
.
.
.
QUESTION: What percentage of patients with cirrhosis develop an umbilical hernia?
.
.
.
.
.
ANSWER: Up to 20% of patients with cirrhosis develop a hernia of the anterior
abdominal wall.

.
******ebook converter DEMO - www.ebook-converter.com*******

******Created by ebook converter - www.ebook-converter.com******

******ebook converter DEMO - www.ebook-converter.com*******

******Created by ebook converter - www.ebook-converter.com******

******ebook converter DEMO - www.ebook-converter.com*******

******Created by ebook converter - www.ebook-converter.com******

UREMIA:
QUESTION: What is the treatment for uremic bleeding in a patient with chronic renal
failure?
.
.
.
.
.
ANSWER:
- The initial treatment for coagulopathy in chronic renal failure is DDAVP, which takes only 1 hour to take effect.
- The definitive treatment, if you have more time to spare (i.e. you do not need to perform an emergency operation and
there is no active bleeding), is hemodialysis.
- Other options include cryoprecipitate (has lots of vWF and factor VIII), erythropoietin, and estrogen.

.
.
.
.
.
QUESTION: What is the best test to assess risk of bleeding in patients with uremia?
.
.
.
.
ANSWER:
- The most useful laboratory test to assess both risk of bleeding and response to therapy in patients with uremia is
bleeding time.

******ebook converter DEMO - www.ebook-converter.com*******

******Created by ebook converter - www.ebook-converter.com******

******ebook converter DEMO - www.ebook-converter.com*******

******Created by ebook converter - www.ebook-converter.com******

URINARY TRACT INFECTIONS: UTI: UTIs:


QUESTION: What organism is the most common cause of UTIs?
.
.
.
.
.
ANSWER:
- The most common cause of urinary tract infections are E.coli. Other causes include proteus, and klebsiella.

******ebook converter DEMO - www.ebook-converter.com*******

******Created by ebook converter - www.ebook-converter.com******

QUESTION: The appropriate duration of antibiotic therapy for nosocomial urinary tract
infection is how many days?
.
.
.
.
ANSWER:
The presence of postoperative UTI should be considered based on urinalysis
demonstrating WBCs or bacteria, a positive test for leukocyte esterase, or a
combination of these elements.
- The diagnosis is established after more than 10^4 CFU/mL of microbes are identified
by culture techniques in symptomatic patients, or more than 10^5 CFU/mL in
asymptomatic individuals.
- Treatment for 3-5 days with a single antibiotic that achieves high levels in the urine is
appropriate.
- Postoperative surgical patients should have indwelling urinary catheters removed as
quickly as possible, typically within 1 to 2 days, as long as they are mobile.

******ebook converter DEMO - www.ebook-converter.com*******

******Created by ebook converter - www.ebook-converter.com******

******ebook converter DEMO - www.ebook-converter.com*******

******Created by ebook converter - www.ebook-converter.com******

VASODILATION:
QUESTION: What are the mediators of vasodilation?
.
.
.
.
ANSWER:
- The mediators associated with vasodilation are nitric oxide, prostacyclin, adenosine.

******ebook converter DEMO - www.ebook-converter.com*******

******Created by ebook converter - www.ebook-converter.com******

******ebook converter DEMO - www.ebook-converter.com*******

******Created by ebook converter - www.ebook-converter.com******

VENTRICULAR SEPTAL DEFECTS: VSDs:


QUESTION: What is the most common isolated congenital cardiac deformity?
.
.
.
.
.
ANSWER: Ventricular septal defects.
- A VSD is a hole between the right and left ventricles.
- It is the most common isolated congenital cardiac deformity.

******ebook converter DEMO - www.ebook-converter.com*******

******Created by ebook converter - www.ebook-converter.com******

******ebook converter DEMO - www.ebook-converter.com*******

******Created by ebook converter - www.ebook-converter.com******

VITAMIN D:
QUESTION: The production of Vitamin D starts in which part of the body?
.
.
.
ANSWER: Vitamin D is made initially made in the skin
.
.
.
QUESTION: Once Vitamin D is made in the skin, where is it next processed and what
change is made to it?
.
.
.
ANSWER: Vitamin D is initiated in the skin, and then processed in the liver with the
modification of 25-OH.
.
.
.
QUESTION: After it leaves the liver, where is Vitamin D finalized, and what is the
modification done to it?
.
.
.
ANSWER: Vitamin D is made into its active form in the kidney. The modification is 1OH.
.
.

******ebook converter DEMO - www.ebook-converter.com*******

******Created by ebook converter - www.ebook-converter.com******

******ebook converter DEMO - www.ebook-converter.com*******

******Created by ebook converter - www.ebook-converter.com******

VITAMIN K:
QUESTION: True or false: The effects of vitamin K reversal take 48hr
.
.
.
.
.
ANSWER: The answer is FALSE.
- Patients who are receiving anticoagulant treatment with warfarin and who require
emergency surgery may be given plasma to immediately reverse the warfarin effect.
- Alternatively, vitamin K may be given orally or subcutaneously at least 6 hr
preoperatively to reverse the effect of warfarin on vitamin K dependent factors.
- The INR should be obtained again before surgery and, if it is not below 1.5, plasma
should be administered.
.
.
.
.
.
QUESTION: What are the vitamin K dependent factors?
.
.
.
ANSWER: Vitamin K dependent factors are II, VII, IX, and X; protein C and S. The
mnemonic is 1972.
.
.
.
QUESTION: True or false: Protein C is vitamin K dependent factor
.
.
******ebook converter DEMO - www.ebook-converter.com*******

******Created by ebook converter - www.ebook-converter.com******


.
ANSWER: True
.
.
QUESTION: True or false: Protein S is a vitamin K dependent factor
.
.
.
ANSWER: True

******ebook converter DEMO - www.ebook-converter.com*******

******Created by ebook converter - www.ebook-converter.com******

******ebook converter DEMO - www.ebook-converter.com*******

******Created by ebook converter - www.ebook-converter.com******

VON WILLEBRAND DISEASE: VON WILLEBRANDS


DISEASE: VWD:
QUESTION: Does von Willebrand's disease have a normal, high, or low PTT?
.
.
.
.
.
ANSWER: von Willebrand disease has a prolonged PTT
.
.
.
.
.
QUESTION: Does von Willebrand disease have a normal, short, or prolonged bleeding
time?
.
.
.
.
.
ANSWER: von Willebrand's disease has a long bleeding time
.
.
.
QUESTION: What special funny named test is used in the diagnosis of von Willebrand
disease? Is a positive or negative result suggestive of the disease?
.
.
.
ANSWER: von Willebrand disease has positive ristocetin test
.
.
******ebook converter DEMO - www.ebook-converter.com*******

******Created by ebook converter - www.ebook-converter.com******


.
QUESTION: von Willebrand disease has 3 types. Which type has low amounts of vWF
and which type has qualitatively poor vWF?
ANSWER: There are 3 types of von Willebrand disease. Type I and III have low
amounts of vWF. Type II is qualitatively poor vWF
.
.
.
.
QUESTION: What is the treatment for vWD?
.
.
.
.
.
ANSWER:
- Von Willebrand disease can be treated with DDAVP (causes release of vWF), vWF, factor VIII, or
cryoprecipitate.

.
.
.
.
.
QUESTION: Which type of von Willebrand disease responds to ddAVP?
.
.
.
ANSWER: Type I and III have low amounts of vWF, responds to ddAVP
.
.
.
QUESTION: What is the other name for DDAVP?
.
******ebook converter DEMO - www.ebook-converter.com*******

******Created by ebook converter - www.ebook-converter.com******


.
.
ANSWER: DDAVP is desmopressin
.
.
.

******ebook converter DEMO - www.ebook-converter.com*******

******Created by ebook converter - www.ebook-converter.com******

QUESTION: What is the effect of DDAVP on vWF?


.
.
.
ANSWER: DDAVP causes release of vWF
.
.
.
QUESTION: In what patients can you use DDAVP?
.
.
.
ANSWER: You can use DDAVP (desmopressin) in patients with ASA or uremic
platelets.

******ebook converter DEMO - www.ebook-converter.com*******

******Created by ebook converter - www.ebook-converter.com******

******ebook converter DEMO - www.ebook-converter.com*******

******Created by ebook converter - www.ebook-converter.com******

VON WILLEBRAND FACTOR: VWF:


QUESTION: What is the mechanism of vWF?
.
.
.
.
.
ANSWER:
- The mechanism of action of vWF is that it links GpIb receptor on platelets to collagen.
- Von Willebrand factor also serves as a carrier for factor VIII.

******ebook converter DEMO - www.ebook-converter.com*******

******Created by ebook converter - www.ebook-converter.com******

******ebook converter DEMO - www.ebook-converter.com*******

******Created by ebook converter - www.ebook-converter.com******

WARFARIN: COUMADIN:
QUESTION: True or false: Warfarin (coumadin) inhibits the activity of vitamin Kdependent factors (II, VII, IX, and X)
.
.
.
.
.
ANSWER: The answer is TRUE.
- With meticulous hemostatic technique, many operations can be performed on patients
with an INR greater than 1.5.
- Exceptions include operations on the eye or the prostate, neurosurgical procedures, or
a blind needle aspiration.
- In these cases, an INR of less than 1.2 is required.
- Patients who are receiving anticoagulant treatment with warfarin and who require
emergency surgery may be given plasma to immediately reverse the warfarin effect.
- Alternatively, vitamin K may be given orally or subcutaneously at least 6 hr
preoperatively to reverse the effect of warfarin on vitamin K dependent factors.
- The INR should be obtained again before surgery and, if it is not below 1.5, plasma
should be administered.

******ebook converter DEMO - www.ebook-converter.com*******

******Created by ebook converter - www.ebook-converter.com******

QUESTION: Which deficiency leads to skin necrosis after starting warfarin therapy?
.
.
.
.
.
ANSWER:
- The deficiency that leads to skin necrosis after starting warfarin is protein C deficiency.

.
.
.
.
.
QUESTION: Which drugs increase the effect of coumadin?
.
.
.
.
ANSWER:
- Drugs that affect coumadin metabolism by increasing its effect are cephalosporins, fluoroquinolones,
amiodarone, quinidine, etc.

.
.
.
.
.
******ebook converter DEMO - www.ebook-converter.com*******

******Created by ebook converter - www.ebook-converter.com******


QUESTION: Which drugs decrease the effect of coumadin?
.
.
.
.
ANSWER:
- Drugs that affect coumadin metabolism by decreasing its effect are corticosteroids, estrogen, OCPs, etc.

******ebook converter DEMO - www.ebook-converter.com*******

******Created by ebook converter - www.ebook-converter.com******

******ebook converter DEMO - www.ebook-converter.com*******

******Created by ebook converter - www.ebook-converter.com******

WOUND HEALING:
QUESTION: The peak number of fibroblasts in a healing wound occurs at what day
after injury?
.
.
.
.
ANSWER: The peak number of fibroblasts in a healing wound occurs 6 days after
injury.
.
.
.
.
.
.
QUESTION: What is the order of cell migration to a wound? Put these cells in the right
order: Fibroblasts, Macrophages, Platelets, PMNs
.
.
.
ANSWER: Cells to wound (in order): platelets, PMNs, macrophages, fibroblasts
(dominant by day 5)
.

******ebook converter DEMO - www.ebook-converter.com*******

******Created by ebook converter - www.ebook-converter.com******

******ebook converter DEMO - www.ebook-converter.com*******

******Created by ebook converter - www.ebook-converter.com******

ZENKERS DIVERTICULUM:
QUESTION: The most important step in treatment of a patient with a Zenker's
diverticulum is:
.
.
.
.
.
ANSWER:
- The most important step in treatment of Zenker's diverticulum is performing a
cricopharyngomyotomy.
- The anatomic problem with a Zenker's diverticulum is failure of the UES to relax with
swallowing.
- The diverticulum is usually resected but in some situations when the diverticulum
would be too hard to remove, it can be suspended upward such that it drains into the
esophagus.
.
.
.
.
QUESTION: What is the name of the triangle that Zenker's diverticulum occurs in?
.
.
.
ANSWER: Zenker's diverticulum occurs in Killians triangle
.
.
.
QUESTION: Is Zenker's diverticulum due to decreased pressure or increased pressure?
.
******ebook converter DEMO - www.ebook-converter.com*******

******Created by ebook converter - www.ebook-converter.com******


.
.
ANSWER: Zenker's diverticulum is due to increased pressure
.
QUESTION#: What is the treatment for Zenker's diverticulum?
.
.
.
ANSWER: The treatment for Zenker's diverticulum is need myotomy and
diverticulectomy/pexy.
.
.
.
QUESTION: What is the surgical approach (i.e. where do you make the incision) to
treat a Zenker's diverticulum?
.
.
.
ANSWER: Your surgical approach in this situation is via a left cervical incision

******ebook converter DEMO - www.ebook-converter.com*******

******Created by ebook converter - www.ebook-converter.com******

REFERENCES
Schwartz's Principles of Surgery .
Rush Review of Surgery .
The Practice ABSITE Question Book .

PURCHASE KEY
DATE

KEY

10/07/13

723KY2023W-3262303JS-272-4MDY2-371-3N2-172-3WY1-3M

12/31/13

37452937363-37203S731-373-27937-312-3MD-382-3829-33

******ebook converter DEMO - www.ebook-converter.com*******

You might also like